CFP module 5 estate planning quizzes

Ace your homework & exams now with Quizwiz!

A trust that is created in a decedent's will and does not become effective until the decedent's death is A) a testamentary trust. B) a complex trust. C) an inter vivos trust. D) a simple trust. Explanation A trust that is created in a decedent's will and that does not become effective until the decedent's death is a testamentary trust.

a

As part of their retirement plan, Stefon and his spouse, Addy, jointly purchased a commercial deferred annuity. This annuity paid income to Stefon and Addy on a joint and survivor basis. Stefon has now died. Which of the following statements regarding this commercial annuity is CORRECT? A) The amount of the annuity includible in Stefon's estate is half of the replacement cost of a single life annuity on Addy at the time of Stefon's death. B) Stefon's estate must include half of the present value of the survivorship benefits. C) Since this annuity had a spousal survivorship feature, none of the benefits qualify for the marital deduction. D) The amount of the annuity includible in Stefon's estate is the amount of the original investment that had not been fully recovered at his death. Explanation The remaining options are incorrect because the amount includible in Stefon's estate is the replacement cost of a single life annuity on Addy at the time of Stefon's death, which is proportionate to the purchase price contributed by the decedent. The present value of survivorship payments is used only for a private annuity. Further, half of the value of the survivorship benefits will qualify for the marital deduction.

a

Marie and Elsa are nonspouse domestic partners. They currently own all of their property in their individual names. They both have qualified retirement plans and IRAs but have not named beneficiaries for them. Each wants to ensure that the other receives her assets when she dies. Which of the following estate planning techniques would be useful to Marie and Elsa? Convert their individually owned property into JTWROS. Establish a revocable living trust naming the other partner as beneficiary at death. Rely on intestate succession laws to pass property to the other partner at death. Name the other partner as beneficiary of their qualified retirement plans and IRAs. A) I, II, and IV B) III only C) II and IV D) I, II, III, and IV Explanation Statement III is incorrect because intestate succession laws generally do not recognize the rights of unmarried domestic partners to inherit property from one another.

a

Ronald and Karen are spouses. Karen is a U.S. citizen, and Ronald is a citizen of Scotland. Karen has a sizable estate, and when she dies she wants to leave it all to Ronald in a way that qualifies for the estate tax marital deduction. Which of the following marital trusts can Karen use to accomplish her goal? A) Qualified domestic trust (QDOT) B) Bypass (B) trust C) Qualified terminable interest property (QTIP) trust D) Disclaimer trust Explanation A qualified domestic trust (QDOT) is used to qualify a bequest for the marital deduction when the surviving spouse is not a U.S. citizen.

a

What is the maximum gift that Bob and Stan, a married couple, can give to one donee in 2020 without paying any gift tax, assuming they have not made any previous taxable gifts? A) $23,190,000 B) $30,000 C) $22,800,000 D) $11,430,000 Explanation For 2020, the answer is $23,160,000: ($11,580,000 applicable exclusion amount × 2 donors) + ($15,000 annual exclusion × 2 donors).

a

Which of the following are characteristics of a qualified disclaimer of assets from a decedent's estate? It must be irrevocable and stated in writing. It must direct the bequest to another person selected by the disclaimant. It must be received by the estate's personal representative. The disclaimant may refuse the bequest after accepting its benefits. A) I and III B) I only C) III and IV D) I, II, and III Explanation A disclaimant can have no say in where the disclaimed property goes, nor can the disclaimant have derived any benefit from the disclaimed property.

a

Which of the following are factors that should be considered in selecting a trustee for a trust that will last for an extended period of time? Appointing of co-trustees Appointing a contingent trustee Providing a method for the appointment of a successor trustee The age of the potential trustee A) I, II, III, and IV B) I and III C) II and IV D) I, II, and IV Explanation All of these are factors that should be considered in selecting a trustee for a trust that will last for an extended period of time.

a

Which of the following characteristics are required for a power of appointment trust ("A" trust) to qualify for the marital deduction? Authorization to the trustee to split trust income between the surviving spouse and other family members Mandatory distribution of all income earned by the trust to the surviving spouse at least annually Control by the surviving spouse over the ultimate disposition of the trust assets Inclusion of the trust corpus in the surviving spouse's gross estate A) II, III, and IV B) I and IV C) I, II, and III D) II and IV Explanation Statement I is false because the spouse must be the sole income recipient in a marital trust, which is what a power of appointment trust ("A" trust) is.

a

Which of the following features apply to both the federal gift tax model and the federal estate tax model? Unlimited marital deduction for qualifying transfers Unlimited charitable deduction for qualifying transfers Use of an applicable credit amount Allowance of an annual exclusion A) I, II, and III B) I, II, III, and IV C) III only D) I and II Explanation Statement IV is incorrect because only the gift tax model allows for an annual exclusion.

a

Diane and Clara are spouses. They own their home as a joint tenancy with right of survivorship (JTWROS). They both worked during their marriage, and Diane contributed 30% of the purchase price for the home, while Clara contributed 70%. The home is currently valued at $1 million. If Clara dies, what amount will be included in her gross estate for estate tax purposes? A) $1,000,000 B) $500,000 C) $700,000 D) $0 Explanation Because Diane and Clara are spouses, 50% of the value of any property they own as JTWROS is included in the gross estate of the first spouse to die, regardless of how much each spouse contributed to the acquisition of the property.

b

Erica owns a house that is not her personal residence and has a fair market value of $575,000. Erica's basis in the house is $400,000. She sells the house to her daughter for $450,000. Which of the following statements regarding this transaction is CORRECT? Erica has a taxable gain of $50,000. The property will not be included in Erica's gross estate when she dies. A) Neither I nor II B) Both I and II C) I only D) II only Explanation This is a bargain sale, and Erica has a taxable gain equal to the difference between the sales price and her basis. The property sold in a bargain sale is not included in the seller's gross estate; the taxable gift portion of the transaction is included as an adjusted taxable gift when calculating the seller's tentative tax base.

b

Erwin makes a gift of his vacation home to his friend, Winnie. Erwin paid $200,000 for the home 20 years ago, and the home has a fair market value of $1.5 million on the date of the gift. What is the value of the gift for gift tax purposes? A) $200,000 B) $1.5 million C) $1.3 million D) $1.7 million Explanation The value of a gift for gift tax purposes is the fair market value of the property on the date of the gift less any consideration paid by the donee. The value of the gift is not affected by the donor's basis in the gifted property.

b

Minnie has made a gift of all her common stock in a closely held corporation to her son and daughter. The gifted shares constitute 20% of the issued and outstanding common stock of the corporation. Minnie will retain 30% of the preferred shares of the corporation, which will pay her a fixed, cumulative annual dividend. Which of the following would be relevant in arriving at the value of the gifted shares for gift tax purposes? The fair market value (FMV) of Minnie's interest in the corporation prior to the gift The FMV of Minnie's preferred shares in the corporation at the time of the gift A lack of marketability discount A blockage discount A minority interest discount A) I and III B) I, II, III, and V C) II, IV, and V D) I, II, III, IV, and V Explanation When a donor gives less than all of his or her interest in the asset, the amount of the gift is determined by subtracting the value of what is retained from the value of the donor's interest prior to the gift (option I). Since Minnie's retained interest in the corporation is qualified for Chapter 14 purposes, the FMV of the preferred shares (option II) becomes relevant. If her retained interest were not qualified, the Chapter 14 rules would require that it be valued at zero. A lack of marketability discount (option III) is possible since this is a closely held corporation for which there is no established market to trade the shares. Blockage discounts (option IV) apply only to publicly traded stock. A minority discount (option V) is possible since the gifted shares cannot control the corporation.

b

Of the following actions taken last year by Joan, which transfers must be included in calculating her total gifts for last year? Purchase of a certificate of deposit (CD) that is payable to her daughter on Joan's death Writing a check to her mother for $3,600 to assist her in paying for recent surgery Placement of her brother's name jointly with her own on the deed to a commercial office building that she purchased Cancellation of an $25,000 debt owed to her by her best friend A) I and IV B) II, III, and IV C) II and III D) I only Explanation Statement I is false because the gift has not been completed. The daughter only has a future interest in the CD.

b

The purpose of including Crummey powers in a trust is to A) allow the grantor to revoke the trust upon 30 days' notice. B) ensure that gifts to the trust qualify for the gift tax annual exclusion. C) protect trust assets against claims by the grantor's creditors. D) prevent the trust from having to pay tax on the trust income. Explanation The purpose of including Crummey powers in a trust is to ensure that gifts to the trust

b

Which of the following is a method by which assets can be transferred both during life and at death? A) Right of survivorship B) Irrevocable trust C) Beneficiary designation D) Testamentary trust Explanation An irrevocable trust can be used to transfer assets both while the grantor is alive (inter vivos) and when the grantor is dead (testamentary).

b

Which of the following is the only exception(s) to an estate transfer being subject to the generation-skipping transfer tax (GSTT) when a gratuitous completed inter vivos transfer is a generation-skipping transfer? The transferor makes payments directly to the recipient for medical expenses. The transferor makes direct payments of medical expenses on behalf of the recipient. The transferor makes payments directly to the recipient for educational expenses. The transferor makes direct payments for tuition expenses on behalf of the recipient. A) II only B) II and IV C) I and III D) I and IV Explanation The transferor can qualify within the limited exception if direct payment is made for medical expenses in addition to making direct payments for tuition expenses.

b

Which of the following planning techniques allow the donor to make deductible charitable contributions without drafting a private trust agreement? Charitable gift annuity Charitable remainder trust Charitable lead trust Pooled income fund A) I, II, and IV B) I and IV C) II and IV D) II and III Explanation Statements I and IV are correct; charitable gift annuities and pooled income funds do not involve the use of private trust agreements. Statements II and III are incorrect because charitable remainder trusts and charitable lead trusts involve the use of private trust agreements.

b

Which of the following property transfers between family members are subject to the special zero valuation rules under Chapter 14? Corporate recapitalizations Partnership capital freezes Buy-sell agreements A) II and III B) I, II, and III C) I and II D) III only Explanation All of these property transfers between family members are subject to the special zero valuation rules under Chapter 14. In other words, these types of transfers must comply with the rules under Chapter 14 of the Tax Code or the transfer will have negative gift tax consequences.

b

Which of the following statements regarding an entity-purchase buy-sell agreement is CORRECT? Under the entity approach, the surviving partners purchase the interest of a partner who dies. Under the entity approach, the number of life insurance policies required to fund the agreement is one per partner or shareholder. A) Both I and II B) II only C) Neither I nor II D) I only Explanation Statement I is incorrect because it is the partnership entity that becomes the purchaser in the buy-sell agreement. The partnership buys the interest of a partner who dies.

b

Which of the following statements regarding donor-advised funds is CORRECT? A donor-advised fund is an arrangement in which the donor makes a gift to charity and then makes future recommendations regarding who should receive grants or future monies from the charity. The major advantage of a donor-advised fund to the donor is the ability to name several charitable recipients. The costs to establish a donor-advised fund are high, as is the minimum required contribution. A) I, II, and III B) I and II C) I only D) II and III Explanation Statement III is incorrect. The costs to establish a donor-advised fund and the minimum required contribution are both low.

b

Which of the following statements regarding general powers of appointment is CORRECT? If a person owns a general power of appointment when she dies, the property that is subject to that power is not included in the holder's gross estate for federal estate tax purposes. The exercise, release, or lapse of a general power of appointment during the holder's lifetime is a gift to the person who receives the property. A) Both I and II B) II only C) I only D) Neither I nor II Explanation Statement I is incorrect, as a general power of appointment held by the decedent-holder at death is included in the holder's gross estate for federal estate tax purposes.

b

Which of the following statements regarding joint tenancy with right of survivorship is CORRECT? Property owned as a joint tenancy with right of survivorship avoids probate when the first owner dies. When a joint tenancy is created in real estate a joint tenant who contributes more than his or her share of the purchase price makes a gift to the other joint tenant. A) I only. B) Both I and II. C) Neither I nor II. D) II only. Explanation Both statements are correct.

b

Which of the following statements regarding revocable living trusts in dealing with potential legal incapacity issues is(are) CORRECT? Revocable living trusts can be used to plan for the grantor's possible legal incapacity. A revocable living trust designed to address incapacity issues typically is not funded until the grantor becomes incapacitated. Assets in a revocable living trust do not avoid probate if the trust is funded during the grantor's lifetime. A) II and III B) I and II C) I, II, and III D) III only Explanation Statements I and II are correct. Statement III is incorrect because assets in a revocable living trust will avoid probate if the trust is funded during the grantor's lifetime.

b

Which of the following statements regarding the generation-skipping transfer tax (GSTT) is CORRECT? The GSTT applies to both inter vivos and testamentary transfers. A transfer that is subject to the GSTT may also be subject to gift tax or estate tax. A) II only B) Both I and II C) I only D) Neither I and II Explanation Both of these statements are correct.

b

Which of the following typically represent liquidity needs of a decedent's estate? The decedent's funeral and burial expenses The decedent's outstanding debts Taxes Payment of cash bequests A) III only B) I, II, III, and IV C) I, II, and III D) II and IV Explanation These are all typical liquidity needs of a decedent's estate.

b

Which one of the following is a CORRECT statement regarding the advantages and disadvantages of using a conservatorship to manage property left to a child? A) The actions of the conservator are not subject to court supervision. B) Establishing the conservatorship can be costly and time consuming. C) A conservator must be the same person who makes decisions regarding the minor's day-to-day personal care. D) A conservator typically has a broader range of powers than does a trustee. Explanation A conservator has a narrower range of powers than a trustee and the actions most definitely are supervised by a court. A guardian manages the day-to-day personal care of a minor while a conservator manages the finances.

b

Zeke is 85 years old. He feels he will need nursing home care within the next 2 to 3 years. In an attempt to become eligible for Medicaid, Zeke gives his home and most of his assets to his grandchildren. Which of the following statements regarding these transfers is(are) CORRECT? The transfers are subject to a look-back period of 36 months. Because of the transfers, Zeke may be subject to a waiting period before he becomes eligible for Medicaid. A) Neither I nor II B) II only C) I only D) Both I and II Explanation Statement I is incorrect because transfers of assets for less than adequate consideration to gain eligibility for Medicaid are subject to a 60 month look-back period.

b

A grandfather is considering making gifts to his grandchildren in 2020. Assuming the grandfather has made no previous generation-skipping transfers, the generation-skipping transfer tax (GSTT) lifetime exemption amount available to the grandfather in 2020 is A) $1,000,000. B) $4,577,800. C) $11,580,000. D) $15,000. Explanation The GSTT lifetime exemption amount for 2020 is $11,580,000.

c

Assume that in 2020 Marleen incurs substantial medical bills at the local hospital and Tasha pays $100,000 directly to the hospital in payment of Marleen's medical expenses. What is the amount of Tasha's taxable gift as a result of this transaction? A) $30,000 B) $100,000 C) $0 D) $85,000 Explanation The payment of another person's medical expenses directly to the medical provider is a qualified transfer and is not considered a gift for gift tax purposes.

c

Case Study Question Assume that in 2020, Grant pays $30,000 directly to State University in payment of Alex's tuition. What is the amount of the taxable gift, assuming Grant makes no other gifts to Alex in 2020? A) $5,000 B) $20,000 C) $0 D) $15,000 Explanation The payment of tuition directly to a qualified educational institution is a qualified transfer and is not considered a gift for gift tax purposes.

c

David wants to draft a document advising his physician that he does not want to receive life-sustaining medical care if he ever becomes terminally ill and is unable to make decisions for himself. Which of the following documents best meets David's needs? A) Letter of personal instruction (side letter) B) Nondurable power of attorney C) Living will D) Nuncupative will Explanation A living will will best meet David's needs. A nuncupative will (another name for an oral will) would not take effect until David's death. A nondurable power of attorney would become ineffective if David ever became incapacitated. A letter of personal instruction (side letter) is an informal document advising the writer's survivors of his wishes concerning post-mortem matters such as funeral arrangements or the disposition of personal effects.

c

Generally, the due date of the federal estate tax return (Form 706) is A) 6 months after the date of death. B) 12 months after the date of death. C) 9 months after the date of death D) April 15 of the year following the date of death. Explanation The federal estate tax return is due nine months after the decedent's date of death, although a six-month extension may be requested.

c

Lucius died in 2020, and he was survived by his spouse, Bedelia, and his son, Dwight. Prior to his death, the only taxable lifetime gifts Lucius made, and the resulting gift tax paid out of pocket, are shown in the table below: 2017:Paid gift taxes in the amount of $100,000 on his 2016 gifts2016:Gifted his hunting lodge to Dwight, retaining a life estate. (The lodge was valued at $300,000 at this time and at $525,000 at Lucius's death; the life estate for gift tax purposes was $180,000.) Bedelia split this gift.2016:Set up an irrevocable unfunded life insurance trust, naming Bedelia as sole income beneficiary at the discretion of the corporate trustee and Dwight as remainder beneficiary, and assigned a policy with a face value of $350,000 on his life to the trust. The replacement cost of the policy at the time of transfer to the trust was $100,000.2016:Established and funded an irrevocable trust for the benefit of his brother, Rufus. (The trust assets had a value of $5,400,000 in 2016 and at Lucius's death). Lucius retained no interest in or powers over this trust, either as grantor or trustee. Other assets owned by Lucius at his death and their values for estate tax purposes were as follows: Sole assets:Cash and investments$6,220,000Other assets$150,000Total sole assets$6,370,000Assets held as tenants by the entirety (TBE):House and personal property (Lucius's share only)$1,200,000 Lucius's will, executed in 2010, left his entire estate to Dwight if he survived him. Lucius's will also exercise a special power of appointment to name Bedelia the sole remainder beneficiary of a trust established by Lucius's father valued at $400,000 at the time of Lucius's death. Additional Information Adjusted taxable gifts* Funeral and administrative expenses Debts of decedent, mortgages, and liens State death taxes paid $5,400,000 $57,000 $43,000 $85,000 *Adjusted taxable gifts are the final adjusted taxable gifts for 2020 after subtracting taxable gifts on any property or properties eventually included in the gross estate. Which of the following most closely approximates the net estate tax due for Lucius's estate? A) $0 B) $915,864 C) $292,000 D) $4,969,800 Explanation See the following computation. The hunting lodge must be included as an asset of the gross estate at its date of death value because of Lucius's retained life estate. The gift of the hunting lodge in 2016 resulted in a taxable gift for Lucius of $45,000 ($300,000 - $180,000 = $120,000) as a future interest gift. This is cut in half by Bedelia splitting the gift ($60,000). $60,000 - $15,000 in 2020 = $45,000. Since the value of the hunting lodge actually ends up being included in the gross estate, this $45,000 taxable gift is subtracted from the final adjusted taxable gifts. However, the footnote said the $5.4 million of adjusted taxable gifts had already taken account of this adjustment. The amount paid in 2017 for gift taxes paid out of pocket for gifts made within three years of death must be included in the gross estate under the gross-up rule. The credit for gift taxes payable on post-1976 gifts is deductible from the tentative tax. Normally, only the amount the decedent would have paid out of pocket if everything (including the maximum credit allowed during the period of the gifts) had been taxed using tax rates for the year of death is the credit amount. However, since tax rates for 2020 (year of death) are the same rates used in 2016 (year of the gifts), the actual gift taxes paid out of pocket ($100,000) can be taken as a gift taxes payable credit. The life insurance must be included as an asset of the gross estate under the three-year rule since incidents of ownership in the policy were released by Lucius less than three years prior to his death. The transfer of the life insurance into the trust was a taxable gift at the time because the spouse's interest would not have qualified for a marital deduction. This is due to her not having exclusive unlimited control of the life insurance proceeds according to the terms of the trust or even a terminable interest that would qualify as a QTIP. However, the footnote for adjusted taxable gifts indicated this was accounted for. Neither the trust for Lucius's brother nor the funds given to Bedelia by exercise of the special power of appointment enter into the computation in any way. Lucius retained no interest in or control over the trust for his brother's benefit, so the trust assets are not brought back into his estate under the three-year rule or the transfer sections. The funds given to Bedelia are not included in Lucius's gross estate because he had a special—rather than a general—power of appointment. The correct computation is as follows: Gross estate:$ 6,370,000Sole assets$ 1,200,000TEE assets$ 100,000Gross up$ 525,000Hunting lodge$ 350,000$ 8,545,000DeductionF&A$ (57,000)Debts(43,000)(100,000)Adjusted Gross Income$ 8,445,000Less:State death taxes(85,000)MDE TBE assets$ (1,200,000)Taxable estate(1,285,000)Plus adjustable taxable gifts$ 7,160,000Tax base5,400,000Tentative Tax:$ 12,560,000Base amount345,800Excess4,624,0004,969,800Credits:Credit Amount$ 4,577,800Gift taxes payable on post-1976 gifts100,0004,677,800Net estate tax due$ 292,000

c

Peter and Ann are married, and each of them has children from a prior marriage. Most of their assets are solely owned. Peter's solely owned assets are worth $11.4 million, and Ann's are worth $9.4 million. Peter and Ann also hold their residence, automobiles, and other personal property (valued at $1 million) as joint tenants. Peter, 68, is in good health and has made no lifetime taxable gifts. Ann, 63, is in poor health and has made taxable gifts of $1 million (all of which were more than three years ago). Peter willed his solely owned assets to his children from his prior marriage, and Ann did the same to her children. Some friends have suggested that they modify their estate plan by placing their solely owned assets in QTIP trusts and having their personal representative elect the marital deduction for all assets placed in the trust. Their respective children will still receive the assets eventually under the QTIP arrangements. Which of the following statements correctly describes the federal estate tax implications of establishing the QTIP trusts and making the election, assuming both deaths occur in 2020 and the estate of the first to die allows the surviving spouse to use his or her deceased spousal unused exclusion (DSUE) amount under the existing or proposed plan? A) Regardless of who dies first, the tax on their combined estates would be less if they implement the proposed QTIP trusts and elect the marital deduction. B) If Ann dies first, her estate tax would be less under the existing plan than under the proposed QTIP trust plan, but Peter's estate tax would be more. C) Implementing the QTIP trusts as proposed would cost Peter and Ann the same amount in federal estate tax on their combined estates as it would under their existing plan. D) If Peter dies first, both his estate and Ann's estate would have to pay less estate tax if the proposed QTIP strategy is elected than they would under their existing plan. Explanation The first step is to determine their current estate tax situation. If Peter died first, his gross estate would be $11.9 million (his solely owned assets of $11.4 million plus $500,000 of the JTWROS property with Ann). However, his taxable estate would be the $11.4 million that went to his children. The remaining $500,000 would receive the marital deduction. Thus, his estate would not owe any estate tax after his entire applicable credit was used at his death. Next, Ann's gross estate would grow from $9.9 million (her $9.4 million of solely owned assets, plus $500,000 of JTWROS property) to $10.4 million with the addition of the remaining $500,000 of formerly JTWROS property. With her $1 million of adjusted taxable gifts, her tentative tax base would be $11.4 million. This is fully covered by her applicable credit, so she would not owe any estate taxes, either. Thus, if Peter dies first, neither of them would owe estate taxes. If Ann died first, her gross estate would be $9.9 million ($9.4 million + $500,000 of JTWROS). However, the $500,000 would be a marital deduction, reducing her taxable estate to $9.4 million. Then her $1 million of adjusted taxable gifts would make her tentative tax base $10.4 million. Thus, her estate would not owe estate taxes, and Peter would receive a $1 million deceased spousal unused exclusion (DSUE). His estate would increase by the $500,000 of JTWROS property he would receive from Ann at death. Thus, his tentative tax base would be $12.4 million (his current $11.4 million of solely owned assets + $1 million that was formerly JTWROS). His exclusion amount would be $12.58 million (his own $11.58 million for 2020 + $1 million DSUE). Thus, his estate will not owe estate taxes, and therefore,their current plan has no estate taxes. The proposed plan would also not owe any estate taxes. If Peter's $11.4 million of solely owned assets received a marital deduction from the proposed QTIP, then 100% of his estate would be eligible for the marital deduction, and thus, his entire $11.58 million exemption would be available for Ann's estate. That would give her a $23.16 million exemption. After Peter's death, her gross estate is $21.8 million (her original $9.4 million of solely owned assets + her original $500,000 of JTWROS assets + his original $11.4 million of solely owned assets + his $500,000 of formerly JTWROS assets). When her $1 million of adjusted taxable gifts are added, her tentative tax base becomes $22.8 million. This is covered by her augmented exclusion, so no estate tax would be owed if Peter died first. If Ann died first under the new arrangement, her gross estate of $9.9 million ($9.4 million + $500,000) would be eligible for the marital deduction. Thus, her taxable estate would be zero. However, she would have a tentative tax base of $1 million because of her adjusted taxable gifts. Her estate tax bill would be reduced to zero by using $1 million of her exemption amount. That would leave Peter $10.4 million of DSUE. When added to his $11.4 million applicable exclusion, he would not owe any net estate taxes until his tentative tax base went over $22.16 million. His tentative tax base is $21.8 million ($11.4 million of his originally solely owned property + $1 million of formerly JTWROS property + $9.4 million of her solely owned property), so he would not owe estate taxes. In all, there are no estate taxes under either the current arrangement or the proposed plan. Does that make the plans equal? Not if you were her children and expected to inherit from her soon due to her failing health. Here is another way to work the question. Peter and Ann own $21.8 million in property, plus Ann has made $1 million in adjusted taxable gifts. Therefore, they have a total of $22.8 million that is subject to estate tax. However, they also have two 2020 applicable exclusion amounts of $11.58 million, for a total of $23.16 million. Portability of the DSUE amount guarantees that this amount, and its resulting credit, will be totally used in one estate or the other. Since they will incur tax on $11.4 million in both estates and have a credit amount that will pay the tax on $11.4 million, there is no net estate tax to be paid in either estate under either the current plan or the proposed plan. Regardless of who dies first and what plan is employed, neither estate will owe any esta

c

Rolando owned a parcel of real estate as an equal tenant in common (TIC) with his wife, Liz, and his brother, Sam. Rolando and Liz each contributed $50,000 to the original purchase price, and Sam contributed $20,000. Rolando recently died and is survived by Liz and Sam. Which of the following statements are CORRECT concerning a tax implication of this form of property ownership? Rolando's estate must include one-third of the property's fair market value (FMV) as of the date of death. When they took title as TIC, both Rolando and Liz made a gift to Sam. Rolando's estate must include 41.66% of the property's FMV at the date of death, unless his personal representative can prove contribution by Sam. After Rolando's death, Liz will be entitled to receive 83.33% of the income from the property because she will receive Rolando's interest by right of survivorship. A) III only B) I, II, III, and IV C) I and II D) II and IV Explanation Rolando's estate must include one third of the date of death FMV of the property because that is his percentage share of ownership. Contribution by the parties would be relevant only if the property were owned in joint tenancy. Sam obtained a one-third ownership interest, but paid less than one-third of the purchase price; thus, Rolando and Liz each have made a gift to Sam. Liz will not automatically receive Rolando's interest, as tenancy in common has no right of survivorship feature.

c

The Chapter 14 zero valuation rules focus on proper valuation of assets at the time of transfer for purposes of determining gift tax. Which of the following statements regarding the Chapter 14 valuation rules are CORRECT? An estate freeze involving the intrafamily transfer of corporate stock or partnership interests generally results in an immediate gift tax based on the entire value of the business held by the senior family member. In the case of buy-sell agreements, the Chapter 14 rules do not apply to transfers between non-family members. A) Neither I nor II B) II only C) I only D) Both I and II Explanation Statement II is incorrect because in the case of buy-sell agreements, the Chapter 14 valuation rules apply to transfers between nonfamily members as well as transfers between family members.

c

When do the Chapter 14 rules of estate valuation generally apply? A) To determine the value of real property owned by a closely held business B) To arrive at the actuarial value of a life estate or remainder interest C) To determine the amount of the taxable gift in certain estate freeze transactions D) To discount the listed value of publicly held securities Explanation The Chapter 14 rules of estate valuation generally apply in certain estate freeze transactions, such as corporate recapitalizations, between family members. These rules also apply in the valuation of interests in certain trusts, such as grantor retained income trusts (GRITs).

c

Which of the following is NOT a way that a person can voluntarily transfer estate assets to another person or entity at death? A) By will substitute B) By probate C) By gift D) By testamentary trust Explanation Gifting is one of the two ways that a person can voluntarily transfer estate assets to another person or entity during life, not at death.

c

Which of the following is a skip beneficiary for purposes of the generation-skipping transfer tax (GSTT)? A related person who is at least two generations below that of the transferor. A trust in which the beneficiaries are skip persons and from which no non-skip person will benefit. An unrelated person who is younger than the transferor by 37½ years or more. A) I and II B) I and III C) I, II, and III D) III only Explanation These are all skip persons for purposes of the GSTT.

c

To qualify for the marital deduction, property must pass to the surviving spouse. How can property pass and still qualify for the deduction? By will By survivorship By intestacy By power of appointment A) I and IV. B) I and II. C) I and III. D) I, II, III, and IV. Explanation All of these methods allow for property to pass and still qualify for the marital deduction.

d

Which of the following statements regarding bypass planning is CORRECT? One purpose of bypass planning is to take full advantage of the applicable credit amount when the first spouse dies. Assets in a bypass (B) trust are not included in the surviving spouse's gross estate. A) II only B) Neither I nor II C) I only D) Both I and II Explanation B Trusts are subject to estate taxes when the first spouse dies. Therefore, B Trusts will not be included in the estate of the spouse when the surviving spouse dies. This makes B Trusts especially effective with appreciating assets.

d

Which of the following statements regarding charitable gifts is CORRECT? Gifts to qualified charities by U.S. citizens or residents are deductible for gift tax purposes. The gift tax charitable deduction is not limited to a percentage of the donor's adjusted gross income (AGI). A) II only B) I only C) Neither I nor II D) Both I and II Explanation Both statements are correct.

d

Which of the following statements regarding cross-purchase buy-sell agreements funded with life insurance is CORRECT? The death benefits under the life insurance policies are generally subject to income tax. The increase of the basis of the surviving owner(s) in the purchased interest(s) will equal the purchase price paid under the cross-purchase buy-sell agreement. The number of policies required may become cumbersome as the number of businessowners increases. A) III only B) I only C) I, II, and III D) II and III Explanation Statement I is incorrect because the death benefits received under the life insurance policies are generally not subject to income tax.

d

Which of the following statements regarding the generation-skipping transfer tax (GSTT) for 2020 is CORRECT? The annual exclusion is allowed. Gift splitting is permitted. Qualified transfers are excluded from GSTT. Each transferor is allowed a lifetime exemption of $11,580,000. A) II and IV. B) I and IV. C) I, II, and III. D) I, II, III, and IV. Explanation All the statements are correct.

d

Which of the following statements regarding the role of guardians in estate planning is(are) CORRECT? A guardian manages the personal care and well-being of a ward. A guardian must account annually to the court until released from his fiduciary duties by the judge. Empathy and caretaking ability are the most important considerations when selecting a guardian. A) I and II B) I and III C) II only D) I, II, and III Explanation All of these statements are correct.

d

Which of the following statements should be considered to minimize the probability of a successful will contest? Leaving every heir a bequest, even if it is small Executing a codicil that appoints a new executor of the estate Establishing a trust during lifetime to provide for testamentary disposition to the heirs Including an in terrorem clause in the will if these clauses are generally enforced in the testator's state A) II and III B) I, II, and III C) I and IV D) I, III, and IV Explanation Appointing a new executor by codicil is a good idea if the originally appointed executor has died or no longer is able to serve in this capacity. This, however, will generally not minimize the probability of a successful will contest.

d

Which one of the following actions would probably not constitute the unauthorized practice of law by a nonattorney financial planner? A) Drafting a power of attorney for a client B) Advising a client to change from sole ownership of property to joint tenancy with right of survivorship (JTWROS). C) Advising a client to conduct business as a partnership rather than a corporation D) Telling a client that property that is titled in joint tenancy with right of survivorship will pass outside of probate at his or her death Explanation This statement merely recognizes a well-established fact and does not constitute the unauthorized practice of law.

d

Which one of the following is an advantage of all will substitutes? A) They can never be revoked. B) They permit you to pass any property by making a payable on death (P.O.D.) designation. C) They are private and are not governed by state law. D) They avoid the probate process. Explanation Will substitutes, by definition, avoid probate.

d

Which one of the following objectives cannot be achieved with an unfunded irrevocable life insurance trust that does not have a Crummey power? A) Naming the trust as beneficiary of the policy with annual income payments to the grantor's spouse and children B) Avoiding transfer tax on the death benefit, as long as the grantor is not the trustee C) Avoiding probate of death benefits D) Sheltering premium payments gifted to the trust from gift tax with the annual exclusion Explanation Sheltering of premium payments gifted to the trust from gift tax by using the annual exclusion cannot be accomplished unless the beneficiaries of the unfunded irrevocable life insurance trust (ILIT) have Crummey powers, which gives them a present interest in the premium payments.

d

Which one of the following statements describes a basic feature of one of the special valuation rules under IRC Chapter 14? A) For a transfer in trust to be subject to these rules, the transferor must be related to the transferee in a specified manner. B) These rules will apply if the transferor sells some of his or her interest in a closely held corporation to a son or a daughter at fair market value. C) These rules will apply if the transferor transfers all of his or her interest in a closely held corporation to a son or a daughter. D) These rules cannot apply to a buy-sell agreement between two unrelated partners. Explanation Chapter 14 rules are best remembered as a set of "anti-shenanigans" laws requiring intrafamily transfers to meet certain restrictions. Unrelated partners would not fall under Chapter 14 rules.

d

Which one of the following statements is CORRECT concerning income earned by spouses in a community property state? A) Income earned by each spouse after marriage is considered community property only if it is commingled. B) Income earned by each spouse before and after marriage is considered community property. C) Income earned by each spouse is considered separate property. D) Income earned by each spouse after marriage is considered community property. Explanation The answer is income earned by each spouse after marriage is considered community property. Even though earned by only one spouse, such earnings are considered community property.

d

Which one of the following statements regarding the estate tax marital deduction is NOT correct? A) The deduction is unlimited in amount. B) Only the amount that passes to the spouse from the decedent will qualify for the deduction. C) If the spouse is given a terminable interest in property as well as a general power of appointment over the same property, the decedent's estate will be allowed to take a marital deduction. D) Use of the deduction is elective for all property that qualifies for the deduction. Explanation If property qualifies for the marital deduction, it must be taken. Under certain circumstances, terminable interest property can be made to qualify for the marital deduction by making an election, such as one made for a QTIP trust.

d

Which one of the following states a basic feature of the IRC Chapter 14 special valuation rules? A) These rules apply to any intrafamily transfer of assets for which there is no established market if the interest retained by the transferor is of the same type as the interest transferred. B) In a transfer of property in trust to which these rules apply, they make any income interest retained by the transferor that is a unitrust right subject to gift tax. C) These rules apply to a transaction between family members to acquire, use, or sell at fair market value property for which there is no established market. D) In a transfer of a closely held business interest to which these rules apply, they make any distribution right retained by the transferor that is not fixed in time and amount subject to gift tax. Explanation A retained distribution right that is not fixed in time and amount must be valued at zero, which requires the transferor to pay gift tax not only on the value of what was given away, but also on the value of what the transferor retained. Retention of a unitrust right by the transferor will allow this right to have a value greater than zero, which can then be deducted from what the transferor owned prior to the transfer. Therefore, the transferor will have to pay gift tax only on the value of what was transferred to others. The Chapter 14 rules do not apply to any transfer for which there is an established market, or where the interest transferred is of the same type as the interest retained. Any transaction to acquire, use, or sell property at fair market value, even between family members, is not subject to the Chapter 14 rules.

d

Which of the following statements regarding a springing power of attorney is CORRECT? With a springing power of attorney, the attorney-in-fact's authority to act is delayed until the principal actually becomes incapacitated or incompetent. A springing power of attorney can be used in planning for the principal's possible incapacity. A) Neither I nor II B) II only C) Both I and II D) I only Explanation Both of these statements correctly define a springing power of attorney.

x

A pre-marital agreement should not be considered by individuals contemplating marriage in which one of the following situations? (CFP® Certification Examination, released 12/96) A) when one or both parties are unwilling to make a full disclosure of all their income and assets to the other party. B) when each party has significant wealth and wishes to protect his/her financial independence. C) when one or both parties have ongoing obligations, rights and/or children from a previous marriage. D) when there is a significant difference in the wealth of each party. Explanation Premarital agreements require full disclosure by both parties as to their financial matters before marriage.

a

Abby is a businessowner, and she wants to provide her key employee with an additional benefit in light of her stellar work performance. Abby is interested in securing a life insurance policy for her key employee. Which type of policy should Abby give to her employee? A) A split-dollar life insurance policy B) A key-person life insurance policy C) A second-to-die life insurance policy D) A first-to-die life insurance policy Explanation A split-dollar life insurance policy can be used to retain the services of key employees by providing them with more insurance protection than the employee could otherwise afford.

a

Adam, age 72, is in the final stages of a terminal illness and wants to ensure that he has a continued fixed income to take care of his medical needs. He also wants to receive a lifetime charitable income tax deduction. Adam is considering making a charitable gift to the American Kidney Fund, so long as his stated objectives are met. As his financial planner, which of the following charitable gifting techniques should you recommend? A) A charitable remainder annuity trust (CRAT) B) A charitable remainder unitrust (CRUT) C) A charitable lead trust D) An outright charitable gift Explanation A charitable remainder annuity trust (CRAT) is the only option that would provide Adam with a fixed income amount. The unitrust form would provide lifetime income to Adam, but it will be a variable income stream. An outright gift to a charity when the client needs income from the asset will never be the correct answer.

a

All of the following items of property would be considered community property in a community property state except A) real estate received by one spouse during marriage as an inheritance from her mother. B) stock purchased during marriage by one spouse using her self-employment earnings. C) a business in which both spouses participate and which is formed during the marriage. D) salary earned by one spouse during marriage. Explanation In a community property state, property acquired by one of the spouses by gift or inheritance during their marriage is considered to be separate property. Items owned prior to the marriage can also be separate property.

a

All of the following statements about fee simple ownership are correct EXCEPT: A) Property owned at death is not eligible for a step-up in basis to fair market value. B) Property owned at death is subject to probate administration. C) Property owned at death is 100% includible in the gross estate. D) The owner in fee simple has complete control and dominion over the property. Property owned in fee simple is entitled to a step-up in basis to fair market value at death.

a

All of the following statements about the major forms of property ownership are correct EXCEPT: A) A community property interest is not subject to probate. B) A tenancy by the entireties is only available to married spouses. C) A tenants in common interest is subject to probate. D) Property held as joint tenant with right of survivorship is partitionable without the consent of the other joint tenant. Explanation A community property interest is subject to probate administration.

a

All of the following statements regarding direct skips for purpose of the generation-skipping transfer tax (GSTT) are correct except A) gift-splitting by spouses is not permitted for direct skips. B) a direct skip is an outright transfer to a skip person or a transfer of property to a trust exclusively for the benefit of one or more skip persons. C) an annual exclusion of $15,000 is available in 2020 for lifetime direct skips. D) the transferor or the transferor's estate is liable for any GSTT that is due on a direct skip. Explanation Gift-splitting is allowed for direct skips if both spouses elect it. All of the other statements are correct. LO 7.1.1

a

Although Grant and Rose are currently in good health, they are concerned that they may become ill in the future and not be able to manage their own affairs. They want to ensure that Marie will be able to manage their financial affairs if either of them becomes incapacitated. Which of the planning devices best meets their needs? A) Durable power of attorney (DPOA) B) Living will C) Special needs trust D) Guardianship Explanation A durable power of attorney (DPOA) best meets their needs because a DPOA allows the agent to continue acting on the principal's behalf even if the principal becomes incapacitated. A living will provides for the suspension of medical care if the drafter becomes terminally ill. A guardianship is a court-supervised proceeding that allows the guardian to manage the personal care and well-being of the ward. Special needs trusts are established to provide supplemental benefits for beneficiaries who are receiving governmental assistance. LO 9.2.1

a

Andrea gifted property with a market value of $2 million to her daughter 18 months before she died. Andrea paid gift tax of $700,000 on the gift. At Andrea's death, the property had a market value of $2.5 million. What amount is included in Andrea's gross estate? A) $700,000 B) $2,500,000 C) $2,000,000 D) $500,000 Explanation The gifted property itself is not included in Andrea's gross estate, but the gift tax paid on the gift ($700,000) is included in Andrea's gross estate under the gross-up rule. The adjusted taxable gift from this transaction is not a part of her gross estate, but the adjusted taxable gifts are a portion of the total estate tax calculation further down in the process. The appreciation on the property after the gift is not subject to transfer taxation.

a

Andy, age 68, has a gross estate currently valued at $2,500,000 that consists primarily of highly appreciated growth securities. Within the last six months, Andy transferred $500,000 worth of these securities to his wife, Harriet. His cost in these securities was $200,000. Harriet recently died. The fair market value of the transferred securities at the time of her death was $500,000. The securities passed to Andy under the terms of Harriet's will. Which one of the following is an income tax implication of the transfer of stock? A) Andy's basis in the stock is $200,000. B) Andy must recognize $300,000 in capital gain on the stock as of Harriet's death. C) If Andy sells the stock he received from Harriet immediately after her death, his gain, if any, will be deemed to be short-term capital gain. D) Andy's basis in the stock is $500,000. Explanation The basis Harriet received from the lifetime gift from Andy would be the same as Andy's. Because she did not live for more than one year, the property does not qualify as a reverse gift and get a stepped-up basis. The basis will remain $200,000.

a

Assume that Grant places assets valued at $1 million into a grantor retained annuity trust (GRAT) with a trust term of 10 years. The remainder beneficiary of the GRAT is Marie. Which of the following statements regarding this GRAT is(are) CORRECT? Grant will receive a fixed annuity payment from the trust each year during the 10-year trust term. If Grant survives the 10-year trust term, the FMV of the trust assets will not be included in his gross estate. A) Both I and II B) I only C) Neither I nor II D) II only Explanation Both of these statements are correct.

a

Assume that Marie's parents give Michael and Marie 1,000 shares of XYZ stock on July 8, 2020, which is a Wednesday. The only trades of XYZ stock that week are as follows: DateHighLowClosing PriceJuly 7$32$28$29.50July 10$35$31$31.50 What is the value of the gift for gift tax purposes? A) $31,000 B) $30,000 C) $32,000 D) $31,500 Explanation The value of the gift is determined as follows: FMV for last trade date before gift: ($32 + $28) ÷ 2 = $30 FMV for first trade date after gift: ($35 + $31) ÷ 2 = $33 Business days from last trade date to gift date: 1 Business days from gift date to first trade date: 2 Prorated before gift FMV: $30 × 2 = $60 Prorated after gift FMV: $33 × 1 = $33 FMV for gift tax purposes: ($60 + $33) ÷ 3 = $31 Total value of gift: 1,000 × $31 = $31,000

a

Assuming that the special valuation rules under IRC Chapter 14 apply to each of the following situations, which one of the following statements is CORRECT? A) In a transfer of corporate or partnership interests, the retention of an interest that will pay a fixed cumulative dividend by the donor will constitute a qualified interest. B) In a transfer in trust, the retention of an annuity or unitrust interest by the grantor is deemed to be a nonqualified interest. C) The Chapter 14 special valuation rules negate the impact of IRC Code sections 2035-2038, otherwise known as the "three-year rule" and the "transfer sections." D) In a transfer in trust, the grantor of the trust must pay gift tax on the entire value of the trust assets if she retains a qualified interest. Explanation Chapter 14 rules do not negate other regulations. A retained interest will reduce the value of the gift because the done is not receiving the entire gift. The retention of an annuity or unitrust interest by the grantor is deemed to be a qualified interest.

a

Because of her financial stability and sizable net worth, Marleen intends to leave the funds in her IRA untouched. When she dies, she believes this asset will get a step-up in basis for her heirs. Which of the following statements regarding Marleen's IRA is CORRECT? She must receive minimum distributions after attaining age of 72, but any remaining amounts in the IRA at her death will receive a step-up in basis. She is correct in her belief, and this is a great strategy. The heirs will not receive a step-up in basis, and she will be penalized if she does not receive distributions when required by the Internal Revenue Code. A) III only B) I and III C) I, II, and III D) I only Explanation There are a couple of problems with Marleen's plan. She is required to take distributions from the IRA once she reaches her required beginning date. Retirement accounts are considered income in respect of a decedent assets and do not receive a step-up in basis at the owner's death.

a

Brielle, a widow, had been in poor health for several years prior to her death. In the two years preceding her death, Brielle took the following actions: Made several lifetime cash gifts to her children Exercised a special power of appointment over property included in a trust Changed the designated beneficiary on a life insurance policy she owned on her life from her deceased spouse to her estate Gave up a life estate in a vacation cabin inherited from her mother Which of the following is included in Brielle's gross estate? A) The life insurance proceeds payable to her estate B) The full fair market value of the vacation cabin C) The trust property over which she held a special power of appointment D) The cash gifts made to her children within three years of her death Explanation If something is given away during life, it will be included in the donor's gross estate only if it is subject to the three-year rule (IRC 2035) or the transfer sections (IRC 2036-2038). None of the actions described would require application of the transfer sections, as Brielle did not retain an interest in or control over something that she irrevocably gifted to another person. While all described actions were taken within three years of death, none of the actions fit the description of the three actions that will cause the three-year rule to apply. Giving up the life estate in the vacation cabin did not constitute the release of a retained right since the life estate was not retained—it was the only interest in the property that Brielle ever had. Brielle owned and retained the right to change the beneficiary of the life insurance policy, which shows that she really did not gift this asset prior to her death, and therefore, it must be included in her gross estate, as she had incidents of ownership in the policy at death.

a

Bruce, age 80, wants to ensure that he has a continued fixed income to take care of him for the rest of his life. Because Bruce is in a high income tax bracket he also wants to receive a lifetime charitable income tax deduction. He is considering making a charitable gift to the American Cancer Society as long as his stated objectives are met. As his financial planner, which of the following charitable gifting techniques do you recommend? A) Charitable remainder annuity trust (CRAT) B) Charitable lead trust C) Charitable remainder unitrust (CRUT) D) Outright charitable gift Explanation A charitable remainder annuity trust (CRAT) is the only option that would provide Bruce with a fixed income amount. The unitrust form would provide lifetime income to Bruce, but the income stream would be variable.

a

Case Study Question If Grant decides to help fund Max's and Sam's college educations using Section 529 plans, what is the maximum amount Grant can contribute to the plans without making a taxable gift in 2020, assuming no gift splitting? A) $150,000 B) $75,000 C) $30,000 D) $300,000 Explanation Under Section 529, a donor can contribute up to $75,000 for each beneficiary without making a taxable gift. The total contribution he can make to both grandchildren without making a taxable gift is $150,000.

a

Consider the following statement of financial position for Robert and Amy, a married couple with no children. They reside in a common-law property state. (Note: JT means property held as joint tenants with right of survivorship between Robert and Amy; client name = separate property.) Robert and Amy's Statement of Financial Position AssetsLiabilitiesCash/checking account (JT)$55,000Mortgage note (JT)$1,571,190Life Insurance cash value (Amy)1$2,393Automobile loan (JT)$20,000Common stock (JT)$710,000Mutual fund (Robert)$816,000Collectibles (Robert)$13,500Pension plan (Amy)2$10,800IRA (Robert)3$755,430Residence (JT)$12,700,000Automobiles (JT)$28,800Personal property (JT)$65,000Total assets$15,156,923Total liabilities$1,591,190Net worth$13,565,733Liabilities and net worth$15,156,923 1Robert is the named beneficiary; Amy's estate is contingent beneficiary. 2Robert is the named beneficiary; Amy's mother is contingent beneficiary. 3Amy is the named beneficiary; Robert's estate is contingent beneficiary. If Robert dies first, what will be the amount of his probate estate? A) $829,500 B) $1,584,930 C) $15,156,923 D) $0 Explanation Robert's probate estate consists of the mutual fund ($816,000) and collectibles ($13,500), which are titled in his name alone. The assets held as joint tenants with right of survivorship pass to Amy by operation of law and avoid probate. Robert's IRA passes by contract directly to Amy because she is the designated beneficiary.

a

Darwin, age 60, has an estate valued at $15 million. Included in the valuation of his estate are the following: A small U.S. Treasury note that is subject to changes in market value One-half ownership in Coeur d'Alene Estates, a private real estate development owned as a joint tenant with rights of survivorship with his sister, Anne 800,000 shares of Untell Inc., a public corporation traded on a major exchange; 50,000 shares of this stock are traded daily A joint and last survivor annuity that names his daughter, Ruthie, as the surviving annuitant When Darwin dies, which one of the following valuation techniques would most effectively reduce the value of his gross estate, and why? A) A blockage discount on the publicly traded stock, because the stock is difficult to sell on a public exchange at one time and in such a large quantity B) A co-ownership discount on the real estate development, because it would be difficult to find a buyer for the real estate six months after the date of Darwin's death C) The alternate valuation date on the joint and last survivor annuity, because the value of the annuity will be less in six months than it is on the date of Darwin's death D) The date-of-death fair market value for the U.S. Treasury note, because it would reflect the current market price if the value of the Treasury note decreases Explanation With 800,000 shares of stock in a company that only trades 50,000 shares daily, this is the rare example of when a blockage discount would be available.

a

Derik Levine recently established a revocable living trust and funded it with several parcels of income-producing real estate. The trust provides that the trustee has discretion to distribute all trust income at least annually in equal shares to Derik's three adult children, who are also to receive the remainder of the trust at Derik's death. Derik named his brother as trustee. Which of the following statements about the tax implications of this trust are CORRECT? Derik will not owe a gift tax on the value of the assets placed in the trust. Derik will be able to take three annual exclusions in computing the gift tax due from funding the trust. Derik will have to report all trust income on his personal income tax return. The taxable value of the assets placed in trust will be included in Derik's estate tax calculation as an adjusted taxable gift. A) I and III B) II and III C) I, III, and IV D) I and II Explanation The answer is I and III only. Establishment and funding of a revocable trust are not deemed to constitute a completed gift, as the donor does not give up dominion and control over the assets. Because no gift is made, no gift tax calculation is necessary, and no gift tax is due. Therefore, the trust assets remain part of the grantor's gross estate. A right to revoke a trust triggers one of the grantor trust rules and thus makes all trust income taxable to the grantor whether or not he ever receives any of the income.

a

Desiree just met with her financial adviser and discovered that her estate substantially exceeds the estate tax exclusion amount. She would like to reduce her potential taxable estate without giving up control of her property during her life. Which of the following premortem planning techniques would achieve Desiree's goals? A) Desiree should make a testamentary transfer of property that will qualify for the charitable deduction. B) Desiree should make lifetime gifts up to the annual exclusion amount. C) Desiree should reduce or eliminate property that is difficult to appraise or administer by sale. D) Desiree should establish and fund a revocable living trust. Explanation A testamentary transfer of property that will qualify for the charitable deduction is a method to reduce the potential taxable estate. Making lifetime gifts up to the annual exclusion amount is a method to reduce the taxable estate without incurring gift tax. However, Desiree does not want to give up control of her property before death. A revocable living trust allows her to retain control, but does nothing to reduce her estate tax liability. Reducing or eliminating property that is difficult to appraise or administer by sale will reduce the administrative fees of an estate, but it will not necessarily reduce the potential taxable estate. Also, she does not want to give up control of her assets.

a

Donna received a gift of rental real estate with an adjusted basis of $75,000 to the donor and a fair market value (FMV) of $50,000 on the date of gift. The donor paid gift tax of $3,000. Donna subsequently sold the property for $60,000. What is her recognized gain or loss? A) No gain or loss is recognized. B) $12,000 loss is recognized. C) $10,000 gain is recognized. D) $15,000 loss is recognized. Explanation The double basis rule applies to this gift. Donna's basis for gain is $75,000, and her basis for loss is $50,000. Because the sales price is between $50,000 and $75,000, no gain or loss is recognized. The gift tax paid is irrelevant because there was no unrealized appreciation as of the date of the gift. If FMV is less than the adjusted basis at date of gift, no basis adjustment is made for gift tax paid.

a

Edward and Dennis are brothers who share the ownership of a farm they purchased together. Edward owns an undivided 40% interest in the property, and Dennis owns an undivided 60% interest. They both have the right to sell their interest in the farm or to leave their interest in the farm to anyone they choose under their will. Which this form of ownership do they have? A) Tenancy in common B) Tenancy by the entirety C) Joint tenancy with right of survivorship (JTWROS) D) Community property Explanation Edward and Dennis own the farm as a tenancy in common because each party owns an undivided interest in the farm and their interests are unequal. The property is not held as JTWROS because each owner can dispose of their interest by will. Community property and tenancy by the entirety can only be owned by spouses.

a

Fred and Ethel live in a community property state. They acquired property during their marriage and classified it as separate property pursuant to a spousal agreement. What is the effect of the spousal agreement? A) The property is separate property so long as the spousal agreement is valid (i.e., recognized by local law and entered into with the requisite intent). B) Community property laws supersede a spousal agreement. C) The property is community property. Property classification cannot be changed pursuant to a spousal agreement. D) The property is separate property so long as the spousal agreement is valid (i.e., recognized by local and federal law and entered into with the requisite intent). Explanation Federal law does not apply.

a

Gil and Tina are newlyweds who live in a community property state. Assuming no titling changes were made, which of the following assets would be separate property? A parcel of land owned by Gil prior to marriage. An antique desk Tina inherits from her aunt during the marriage. A money market account owned by Gil before marriage that has not been commingled with community assets. A) I, II, and III B) II only C) I and II D) I and III Explanation The assets in statements I, II, and III are all separate property.

a

Ginger sells some antique jewelry with a fair market value of $50,000 to her daughter for $10,000. Ginger's basis in the jewelry is $5,000. What is the value of Ginger's gift for gift tax purposes? A) $40,000 B) $45,000 C) $0; this is a sale, not a gift D) $25,000 Explanation This is a gift for gift tax purposes because it is a transfer of property in exchange for less than full and adequate consideration. The value of the gift is the fair market value of the property on the date of the gift ($50,000) reduced by any consideration paid by the donee ($10,000). The adjusted taxable gift in this situation was $25,000 in 2020 ($40,000 - $15,000), but the question asked about the value for gift tax purposes, not the adjusted taxable gift.

a

Hal has a gross estate currently valued at $11.5 million. He intends for his wife, Leslie, and two minor children to be the primary beneficiaries of his estate. Hal wants to provide their minor children with current income from some of his investments until they reach the age of majority without having any of the income taxable to him. His primary tax goal is to reduce his federal estate tax liability when he dies. Which one of the following is the most appropriate trust arrangement for Hal to use to accomplish all of his objectives regarding property to be used to benefit the children? A) A Section 2503(b) trust with his children as both income and remainder beneficiaries B) A 15-year grantor-retained income trust (GRIT) C) A revocable living trust D) A qualified domestic trust Explanation An irrevocable transfer to a Section 2503(b) trust in which Hal has no retained interest (or discretion as trustee) will achieve both objectives of not having income taxable to Hal and not having the assets included in his gross estate. Hal must make sure that none of the income from the trust is used to satisfy his legal obligation of support to the children to avoid the grantor trust rules. The other options would not accomplish Hal's objectives. A qualified domestic trust is used to transfer property to a noncitizen spouse at death. A revocable living trust would be included in Hal's gross estate. Income from a GRIT would be taxable to Hal under the grantor trust rules.

a

Henry dies on June 30 of the current year. His gross estate consists mostly of rental real estate with a market value of $20 million on his date of death. Within a few weeks after he dies, the real estate market takes a severe downturn, and the value of his property declines to $17 million by December 31. Which of the following postmortem estate planning techniques might be useful to Henry's estate? A) Alternate valuation date (AVD) election B) QTIP election C) Qualified disclaimer D) Special use valuation Explanation An election to use the AVD would be useful because it allows the estate to value estate assets for estate tax purposes at their value six months after the date of death rather than on the date of death. Special use valuation allows the estate to value qualifying real property on the basis of its actual use rather than its highest and best use. A QTIP election allows the estate to take the marital deduction for bequests of certain terminable interests, and a qualified disclaimer allows a beneficiary to refuse a bequest or inheritance.

a

If included as part of your married client's gross estate, which of the following property interests qualify for the marital deduction? A life estate interest in, and a general power of appointment over, the family residence (titled in his name only) to his wife A trust with income distributable at least annually to his wife and his children, with the remainder to the children at his wife's death A life income interest in a testamentary charitable remainder trust to his wife as the only noncharitable beneficiary A stock portfolio owned in joint tenancy with right of survivorship by the client and his brother A) I and III B) I, III, and IV C) I, II, and IV D) II and III Explanation Statement II is false because income is also paid to the children and to qualify for the marital deduction, income must only be payable to the spouse. Statement IV is false because JTWROS property owned with his brother will not qualify for the marital deduction. The spouse is completely left out of that property, so no deduction is available.

a

In 2020, Ron gives $20,000 in cash to each of his four children. What amount of taxable gifts, if any, must Ron report for 2020? A) $20,000 B) $80,000 C) $0 D) $65,000 Explanation Because all of the gifts are present interests and they are made to four different donees, each gift is eligible for the annual exclusion of $15,000 (in 2020). Therefore, total taxable gifts are $20,000: 4 × ($20,000 − $15,000).

a

In 2020, Walter gave his 20-year-old son, Rufus, stock valued at $320,000. Walter's spouse, Frances, consented to split this gift with Walter for gift tax purposes. What amount of taxable gift must Walter report on his gift tax return for 2020? A) $145,000 B) $320,000 C) $160,000 D) $305,000 Explanation Walter must report a taxable gift of $145,000 on his 2020 gift tax return [($320,000 ÷ 2) − $15,000 annual exclusion]. The annual exclusion is considered after the gift is split between spouses.

a

In light of the multiple possible estate goals which of the following would be plausible candidates for estate planning? A) All of these B) A recently married couple expecting their first child. C) A Pennsylvania domiciled client with a condo in Vail, Colorado. D) A client whose wealth is concentrated in a real estate business. Explanation All are potential clients to deal with issues of liquidity, need for a guardian and avoidance of ancillary probate.

a

In which of the following situations must the donor file a federal gift tax return? The donor makes a gift of a present interest valued at $10,000 to one donee. The donor makes a gift of a future interest valued at $1,000 to one donee. The donor makes a gift of $10,000 to one donee and the donor's spouse agrees to gift splitting. A) II and III B) I, II, and III C) I only D) I and II Explanation Statement I is incorrect; a gift tax return is not required because the gift does not exceed the annual exclusion and is not a gift of a future interest. Statements II and III are correct. A gift tax return is required when a gift of a future interest in any amount has been made or when gift splitting has been elected by spouses.

a

In which one of the following situations would a state's intestate succession laws be applied? A) The decedent owned a residence that is community property in a community property state, and the decedent died without a will. B) The decedent transferred all of his property to an inter vivos revocable trust, and died without a will. C) The decedent owned real estate in fee simple, and left a valid holographic will that leaves his property to his sister. D) The decedent owned property as a tenant in common, and the decedent's will contains a residuary clause naming his son as beneficiary. Explanation Intestacy applies when a person dies without a will or with a will that fails to include all probate property.

a

Jacob and Wendy have been married for nine years and live in a common law state. They have two children, ages 4 and 2. In the same year they were married, Wendy insisted that they should each have wills. Since they had no children at the time, the wills they executed gave everything to the survivor, or if there was no survivor, to that person's brothers and sisters. Recently, the state in which Jacob and Wendy live has passed a statute allowing wills to be self-proved if they include language specified in the statute. Which of the following correctly state why Jacob and Wendy's current wills do or do not need to be amended? Their wills need to be amended to provide for their minor children's personal care if there is no survivor. Their wills do not need to be amended to provide for the children because of intestacy statutes. Their wills do not need to be amended to be self-proving, as they will be grandfathered since they were already validly executed. Their wills need to be amended to provide for the financial care of their children if there is no survivor. A) I and IV B) I and III C) I, III, and IV D) II, III, and IV Explanation Statements II and III are false because the wills should be amended to account for the new language required for self-proving wills.

a

James leases an office building for use in his business. The lease gives James the right to use the building for 10 years in exchange for annual lease payments of $100,000. What type of ownership interest does James have in this building? A) Term of years B) Life estate C) Future interest D) Remainder Explanation James's interest under the lease is a term of years because it gives him the right to possess and use the building for a given period. It is not a future interest because the right to enjoy the benefits of the property occurs immediately.

a

James was gifted a house during the current year. At the date of the gift, the house had a fair market value (FMV) of $175,000, and the donor's adjusted basis was $105,000. The donor paid a gift tax of $12,000 on the gift. The donor did not have the annual exclusion available for this gift. What is James's basis in the house? A) $109,800 B) $175,000 C) $187,000 D) $117,000 Explanation Because the property was appreciated property as of the date of the gift, a portion of the gift tax paid is allocated to the donee's basis in the property. Therefore, James's basis is calculated as follows: donor's adjusted basis + [(unrealized appreciation ÷ FMV at date of gift) × gift tax paid]. Or, here, $105,000 + [($70,000 ÷ 175,000) × $12,000] = $109,800. $105,000 + $4,800 = $109,800.

a

Jill, Sherry, and Peggy are each one-third owners of a closely held business and they have executed a buy-sell agreement. The agreement requires that Jill, Sherry, and Peggy each purchase and pay the premiums on an insurance policy that insures each co-owner and names the policyowner as the beneficiary. Which of the following correctly states an advantage or disadvantage of this buy-sell agreement? A) The premiums paid are not deductible by the policyowner. B) The premiums paid are income to the insured under the policy. C) The premiums are considered a gift from the owner to the insured. D) The replacement cost of the policy must be included in the owner's gross estate. Explanation Premiums paid are not income or gifts and the death benefit is what is included in the owner's gross estate.

a

Joaquim is contemplating the sale of his solely owned business to his son in the form of a private annuity transaction. Which one of the following statements is CORRECT regarding the disadvantages of this type of transaction? A) The transaction will not allow Joaquim to realize the full value of the business if he dies prior to his actuarial life expectancy. B) The son's obligation to make the annuity payments will cease when Joaquim reaches his actuarial life expectancy. C) The son must pay a premium for the right to cease making payments whenever his father dies. D) This transaction would be subject to the IRC Chapter 14 rules. Explanation Annuities, even private ones, are based on life expectancy and if he does not reach that age, the full value will not be paid out. Additionally, if Joaquim lives beyond life expectancy, he will continue to be paid. Private annuities are based on FMV so they would not be subject to Chapter 14 rules. The payment of a premium for the right to cease making payments whenever his father dies applies to self-cancelling installment notes (SCINs), not private annuities.

a

Jose and Alfonso are brothers. They own a rental property as joint tenants with right of survivorship. The current value of the property is $1 million. Jose can prove that he contributed 80% of the acquisition price for the property. If Alfonso dies, what amount will be included in his gross estate for estate tax purposes? A) $200,000 B) $1,000,000 C) $0 D) $500,000 Explanation Because Jose and Alfonso are not spouses, the consideration furnished rule determines how much of the property is included in the gross estate of the first joint tenant to die. Because Jose can prove he contributed 80% of the acquisition price, only 20% ($200,000) is included in Alfonso's gross estate.

a

Juan Valentino's will leaves his half of his probate estate to a testamentary trust in which his spouse and children are income beneficiaries, and his children the remainder beneficiaries. The will gives the remainder of his probate estate outright to his children. Juan wanted to be assured that both his spouse and children will receive some part of his estate while incurring minimal estate administration fees. Juan has a gross estate of $2 million. Since the will was drafted, Juan has had second thoughts about the way he decided to distribute the assets of his estate. Does Juan need to consider amending his will? A) Yes, because the existing will does not include a residuary clause and thus could be subject to the intestacy statutes. B) Yes, because if his spouse predeceases him, her portion of the estate will pass through intestacy at Juan's death and be distributed to unintended beneficiaries. C) No, because the spousal elective share given to his spouse by state statute will permit her to take all of Juan's estate in any event. D) No, because there will be no estate tax to be paid out-of-pocket by his estate. Explanation Juan needs to consider amending his will because the existing will does not include a residuary clause and thus could be subject to the intestacy statutes. The spouse's portion of the estate would be distributed to Juan's child as a remainder beneficiary of the testamentary trust. The spousal elective share never entitles a surviving spouse to all of a deceased spouse's estate. If his spouse and child do not survive him, there are no named beneficiaries and the estate will be subject to the intestacy statutes.

a

Jud and Harry are equal partners in a small but thriving business. They recently signed a cross-purchase buy-sell agreement. They wish to use life insurance to fund their respective obligations in this agreement but want to do so with the least possible cost. Which one of the following types of insurance products is the most appropriate for Jud and Harry's situation? A) A "first-to-die" joint lives policy B) Key person policies C) A joint and last survivor policy D) Split dollar policies Explanation A first-to-die policy will be less expensive than two individual policies and pays out upon the first death essentially ending the policy when it is no longer needed.

a

Karen is the sole owner of a wildly successful small business. However, her financial adviser told her that her estate would be unable to meet its cash requirements if she were to die today, and the business would have to be sold. Karen, who is a widow with no children, is only 50 and plans to continue running the business for many years. However, she does want to take some action to prevent the possible sale of the business should she die and prevent future appreciation of the business from inclusion in her gross estate. Karen is very close to one of her nieces, who has shown an interest in and aptitude for the business. Which of the following actions would be most appropriate to help meet the liquidity needs of Karen's estate and would be consistent with her objectives and situation? A) Karen should undertake a preferred stock recapitalization of the business in which she retains a majority of the voting stock with a fixed liquidation value and begin a gifting program whereby she gives nonvoting common stock to her niece. B) Karen should enter into a cross-purchase buy-sell agreement with her niece, funded with life insurance on each of their lives. C) Karen should enter into a private annuity agreement with her niece regarding the business. D) Karen should convert the business into an S corporation and gift shares in the corporation to her niece over a period of years. Explanation Karen should undertake a preferred stock recapitalization of the business in which she retains a majority of the voting stock with a fixed liquidation value, and begin a gifting program whereby she gives non-voting common stock to her niece. This will achieve all of her objectives. A buy-sell agreement funded with life insurance would not work because Karen already owns 100% of the business, and the funds would only be available upon Karen's death. If she wants to transfer control while she's still alive, this won't work. An S corporation can only have one share type and would give control to the niece before Karen is ready to do so. LO 6.2.3

a

Last year, Gloria made the following gifts: $10,000 in cash to her husband, Geno Labor and services worth $5,000 to the Preserve Foundation, a qualified public charity $8,000 to a Section 2503(b) trust with her father as the mandatory income beneficiary and her children as the remainder beneficiaries $15,000 worth of non-income-producing real estate to a Uniform Transfers to Minors Act (UTMA) account for her son, Lance Her husband, Geno, refuses to sign the consent section of the Form 709 that Gloria must file. Which of the following reductions can Gloria claim from her total gift calculation? A marital deduction of $10,000 for the gift to her husband, Geno A charitable deduction of $5,000 for the gift to the Preserve Foundation An annual exclusion of $8,000 for the transfer to the Section 2503(b) trust An annual exclusion of the maximum amount for the gift to the UTMA custodial account Gift splitting of all gifts except the gift to Geno A) IV only B) I and IV C) II and V D) I, II, and III Explanation Option I is false because if the gift is a present interest gift to a spouse (or to a qualified charity) the annual exclusion must be taken before the marital deduction (or charitable deduction). Option II is incorrect because only a gift of cash or property qualifies for a charitable deduction. Option III is wrong because the gift to a Section 2503(b) trust must be allocated part to the income interest and part to the remainder interest based on the term of the trust; only the portion of the $8,000 allocated to the mandatory income right (not the full amount gifted) will qualify for the annual exclusion. Finally, option V is false because Geno has refused to consent to gift splitting. Gifts to a custodial account are entitled to the gift tax annual exclusion up to the maximum amount.

a

Lou inherited a parcel of real estate. Five years ago, he changed the title to joint tenancy with right of survivorship (JTWROS) with his wife, Eve. Lou would like to will the property to John, his son from a previous marriage, so John can use the property to start a business. What is one disadvantage of holding the property in its current form? A) If Lou predeceases Eve, the property will pass to Eve as surviving joint tenant without regard to the terms of Lou's will. B) The property will be included in Lou's gross estate based upon his relative contribution. C) Lou's one-half will not qualify for the marital deduction when it passes to Eve. D) The testamentary transfer from Lou to John will occur without Eve's consent. Explanation By owning the property as JTWROS, Lou can only transfer his interest in the property to John in his will if he survives Eve. If Lou dies before Eve, the property will pass to her by right of survivorship. Lou's interest would qualify for the marital deduction when it passes to Eve. The relative contributions toward purchasing the property are irrelevant with spouses. Each spouse is defined by law as having contributed half. Also, even though Lou has made no contribution to acquire this property (it was inherited), his original basis would be the stepped-up basis from the person from whom he inherited the property. If he dies before Eve, Lou's will can have no effect upon JTWROS property. None of the property will be received by John if Lou predeceases Eve unless the current ownership form is changed.

a

Lucinda took all of the following actions within three years of her death: She made a gift to her children by releasing a life interest in a condominium that she had previously placed in an irrevocable trust for her children. She gave $15,000 in cash to her nephew. She assigned all incidents of ownership in a life insurance policy on her life to an irrevocable life insurance trust (ILIT) for the benefit of her children. She released the right to exercise a general power of appointment over assets in a trust established by her father in which she had no other interest. Which of the following assets are included in Lucinda's gross estate? The condominium placed in trust The cash given to her nephew The life insurance policy given to the ILIT The trust assets over which Lucinda held the general power of appointment A) I and III B) III only C) I, III, and IV D) II and IV Explanation The three-year rule requires that something be included in a decedent's gross estate in three instances: (1) gift taxes paid out of pocket on gifts made within three years of death, (2) assignment of ownership rights in a life insurance policy on the assignor's life (option III), and (3) release of a retained interest covered by the transfer sections of the Code (sections 2036, 2037, and 2038). The right to use the condominium (option I) is a retained right mentioned in Code Section 2036. Options II and IV will not be included in Lucinda's gross estate, but the taxable portions of those transfers will be included in her estate tax calculation as adjusted taxable gifts. Option II is not included in her estate, as it is a completed gift under the annual exclusion amount.

a

Nancy is a widow with four children and nine grandchildren. In preparing her will, she wants to leave her entire estate to her children equally. If any of her children are not living when she dies, she wants that child's share of her estate to be split equally among that child's living children. Which of the following will provisions best meets Nancy's needs? A) A provision calling for a per stirpes distribution B) A simultaneous death provision C) A survivorship provision D) A provision calling a per capita distribution Explanation This best meets Nancy's needs because with a per stirpes distribution, members of a designated class inherit property as members of the class. In other words, the children of a deceased child would split their deceased parent's share equally. With a per capita distribution, the children of a deceased child would each receive a share equal to the shares received by the surviving children. A simultaneous death provision is used in the wills of spouses to create a presumption as to the order of death if both spouses die simultaneously. A survivorship clause provides that a beneficiary must survive for a specified period beyond the testator's death to receive a bequest.

a

Odell is the income beneficiary of a trust currently valued at $1 million. He also has a noncumulative general power of appointment with respect to the trust principal of $50,000 (5% of the total value) per year. If Odell dies in the current year without exercising his power over the trust principal and before the power lapses, what amount will be included in his gross estate? A) $50,000 B) $0 C) $1 million D) $500,000 Explanation Because Odell's power over the trust principal is limited to 5% of the total value of the trust, the power qualifies as a 5 or 5 power (i.e., it does not exceed the greater of $5,000 or 5% of the trust principal). When Odell dies, the value of the power that was available but not yet exercised in the current year is included in his gross estate. In this case, $50,000 was the value of the power that was available in the current year but not yet exercised.

a

On January 1, 2020, Paul gifts a piece of land (basis of $100,000, FMV of $300,000) to his father. Which of the following statements is CORRECT? A) Paul's father's basis in the property will be $100,000. B) If Paul's father dies on December 1, 2019, and leaves the land to Paul, Paul will receive a stepped-up basis in the property. C) If Paul had sold the land to his father for $50,000 instead of gifting it, he would have had a deductible loss of $50,000. D) If Paul dies, the property will be included in his gross estate at the date-of-death value. Explanation The gift basis to a donee is the carryover basis of the donor ($100,000). The boomerang rule of Section 1014(e) precludes Paul from receiving a stepped-up basis at his father's death if his father dies within one year of receiving the property from Paul.

a

Prior to his death on January 1, 2020, Garth took the following actions in the years indicated: 2017: He gave a cash gift of $50,000 to his brother, on which he paid $14,400 in gift tax out of pocket 2015: He released a general power of appointment over assets in a trust established by his mother; Garth is not the trustee or a beneficiary of this trust. 2012: He relinquished to his three children a life estate in a condominium that he had retained when he transferred title to them in 2004; the life estate had a value of $56,000 at the time of the gift in 2004 and a value of $52,000 at the time of transfer in 2012. 2010: He established an irrevocable trust for the benefit of his grandchildren, naming himself as trustee and providing that the trustee could distribute income and principal from the trust at his sole discretion; the assets used to fund the trust had a date of gift value of $100,000 and a date of death value of $105,000. Which of the following statements regarding the impact of these lifetime transfers on Garth's estate tax liability are CORRECT? Garth's estate will have to include the $14,400 paid in gift tax in the gross estate and $36,000 in adjusted taxable gifts as a result of the 2017 gift. Release of the power of appointment in 2015 will not be in his gross estate at death. Garth's estate tax liability will not be affected by the transfer in 2012 since it occurred more than three years prior to death. Garth's estate will have to include the date of death value of the assets of the trust established in 2010 in his gross estate, but it will not have to include the amount taxable for gift tax purposes in his adjusted taxable gifts amount. A) I, II, and IV B) I and II C) II and IV D) I, II, and III Explanation For 2017, $14,400 (option I) must be included in the gross estate under the gross-up rule, as gift tax was paid out of pocket on a gift made within three years of Garth's death. The taxable portion of this gift—$36,000 ($50,000 less one maximum annual exclusion of $14,000)—will be an adjusted taxable gift in Garth's estate tax calculation, as the 2017 gift is not brought back into his gross estate. For 2015, release of a general power of appointment (option II) by a holder is deemed to be a gift since the assets that the holder could have taken for himself will now go to someone else. Therefore, the value of these assets will be frozen for estate tax purposes at their date of gift value (as an adjusted taxable gift) rather than their date of death value in the gross estate had the release not been made. In summary, Garth's gross estate will not include the general power of appointment, but the taxable value will be in his tentative base. Garth's estate tax liability (option III) will be affected by the 2012 transfer because he has made a gift of his life estate to his children, and his adjusted taxable gifts will be increased by the taxable portion of this gift. Also, the property in which he held the retained life estate will not have to be included in his gross estate like it would have had he not relinquished the life estate or if he had died within three years of releasing the life estate. The date of death value of the assets included in the trust established in 2010 (option IV) will have to be included in Garth's gross estate under Code Section 2038 of Chapter 14, as Garth retained the right to determine beneficial enjoyment of the trust property.

a

Regina establishes a funded irrevocable life insurance trust (ILIT) as part of her estate plan. Which of the following statements regarding this trust is CORRECT? The trust holds title to a life insurance policy on Regina's life and also income-producing assets that may be used to pay the policy premiums. Income from the ILIT assets is taxed to Regina as grantor of the trust. A) Both I and II B) I only C) II only D) Neither I nor II Explanation Both statements are correct. A funded ILIT is a grantor trust because it uses trust income to pay the premiums on life insurance covering the grantor's life. Because the trust is a grantor trust, trust income is taxed to Regina

a

Robert Williams is thinking of retitling a brokerage account that is currently solely in his name to add a provision that the account be transferred on his death to his daughter. Which of the following statements is CORRECT regarding an advantage or a disadvantage of retitling the account in this manner? A) An advantage is that Robert has reduced his probate estate. B) A disadvantage is that Robert will no longer be able to control the account. C) An advantage is that Robert has reduced his gross estate. D) A disadvantage is that Robert will no longer be able to retitle the account without his daughter's consent. Explanation A brokerage account with a T.O.D. (transfer on death) designation will pass by will substitute, and thus the probate estate has been reduced. The assets, however, will still remain in Robert's gross estate. Because such a retitling is revocable, Robert will still be able to control the account and change the title.

a

Roland was the sole owner of a toy manufacturing company with a fiscal year ending on January 31. Approximately 50% of the company's income is received in the month of January, with expenses incurred evenly over the year. Roland died on January 1, leaving his entire estate, including his interest in the company, to his daughter, Penney. Penney is in the lowest marginal income tax bracket. Roland named his brother, Buck, a contingent beneficiary of the estate. Buck is in the highest marginal income tax bracket. Which of the following postmortem elections would best minimize the income tax liability for Roland's estate and its beneficiaries? A) The executor should use the calendar year as the tax year for the estate. B) The executor should elect a fiscal year for the estate that ends 13 months after Roland's death. C) Penney should make a qualified disclaimer of the interest left to her in the toy company. D) The executor should elect a fiscal year ending January 31 for the estate. Explanation The election of the shorter first fiscal year is incorrect because that will result in fewer deductions available to offset the large amount of income received in January. Also, any income Penney receives from the estate is not reportable by Penney until the estate's taxable year ends, thus allowing her to delay the payment of any tax on such income for an extended period of time after its receipt. The option to elect a fiscal year for the estate that ends 13 months after Roland's death is incorrect, as the estate's taxable year must end on the last day of a month that does not exceed the date of the decedent's death. The action to make a qualified disclaimer would mean that the toy company would pass to Buck and that future income would be taxed at a higher rate than if Penney had retained the interest.

a

Rosy wants to establish a trust for her three grandchildren that will accomplish all of the following objectives: Exclude all assets transferred to the trust from her gross estate Protect the trust assets prior to distribution from the creditors of any beneficiary Keep the trust assets from disqualifying a beneficiary for public assistance benefits such as Medicaid Have all income produced by the trust taxable to someone other than herself Which one of the following trust provisions would NOT help to achieve one or more of these objectives? A) A provision making the trust irrevocable B) A spendthrift provision C) A provision granting each beneficiary a general power of appointment D) A provision making all distributions from the trust-of both principal and income-subject to the absolute discretion of a corporate trustee Explanation A general power of appointment would not accomplish any of the stated objectives and would actually prevent the second and third objectives.

a

Roxanne, a widow, has a gross estate valued at $999,000. One-fourth of her estate is in raw land held for speculation, and another large portion is in a closely held partnership, valued at $380,000. Roxanne's will leaves all property to her son, with all debts, expenses, and taxes to be paid from the residue. She has employer benefits of about $200,000, payable to her son. Five years ago, she created an irrevocable life insurance trust, which is the owner and beneficiary of a $150,000 life insurance policy on her life; her son is the only beneficiary of the trust. She has made prior taxable gifts of $600,000. Which of the following postmortem techniques are available and advisable to increase liquidity in Roxanne's estate if she were to die today? A Section 6166 extension and installment payment of taxes An election of special use valuation for the raw land A request to the trustee of the irrevocable insurance trust to purchase some of the hard-to-sell property from her estate A Section 303 stock redemption A) III only B) I, II, and IV C) I, III, and IV D) II and III Explanation This estate will not qualify for a Section 6166 extension and installment payment of taxes because the raw land is less than 35% of her adjusted gross estate. A special use valuation is not allowed because the land is not a farm. Section 303 does not apply because the business is not incorporated.

a

Sam wants to leave some real estate to his wife, Inez, outside of probate when he dies. He would also like her to receive some interest in the real estate while he is alive. Sam does not want Inez to be able to transfer or encumber her interest in the real estate without his consent. Which one of the following is the most appropriate form of will substitute for Sam to use? A) Place the property in tenancy by the entirety B) Place the property in tenants in common C) Give Inez a remainder interest in the property while retaining a life estate D) Place the property in joint tenancy with right of survivorship (JTWROS) Explanation Tenants in common and JTWROS are partitionable without consent, which is not one of Sam's goals. Giving Inez a remainder interest does not give her the present interest he wants her to have while he is alive.

a

Sharon gives Patrick the absolute right to use her vacation house for life and upon Patrick's death, all rights to the house are assumed by Sharon again. What types of property interests do Sharon and Patrick have, respectively? A) Reversion and life estate B) Life estate and reversion C) Interest for a term of years and life estate D) Fee simple estate and reversion Explanation Sharon has a reversion, because she receives the property back when Patrick dies (i.e., the property reverts to her). Patrick has a life estate, because he is entitled to use and possess the property during his life.

a

Ted and Betty are spouses. In 2020, Betty makes a gift of $25,000 to her mother, and Ted agrees to treat the gift as a split gift. Who must file a gift tax return for 2020 if this is their only gift? A) Betty B) Ted C) Both Betty and Ted D) Neither Betty nor Ted Explanation Only Betty must file a gift tax return. If spouses agree to split gifts for the year and make no gifts in excess of $30,000 (2019), only the donor spouse is required to file a gift tax return. The consenting spouse must sign the donor's gift tax return to consent to the gift splitting. Filing a gift tax return in this situation documents the splitting of the gifts. It also documents the value of the gift.

a

The Chapter 14 valuation rules of the Internal Revenue Code apply to which of the following types of techniques between family members? Corporate recapitalizations Partnership capital freezes Grantor retained trusts Buy-sell agreements A) I, II, III, and IV B) I and II C) III only D) IV only Explanation The Chapter 14 valuation rules apply to all of these techniques among family members. They also apply to buy-sell agreements among non-family members.

a

The XYZ Corporation has purchased a "key person" whole life insurance policy on its founder, major shareholder, and CEO, Albert, age 72. The beneficiary of the policy is the corporation. Which one of the following is a CORRECT statement concerning the income, gift, or estate tax implications of this type of life insurance business planning technique? A) XYZ Corporation will own the cash value of this policy. B) Upon Albert's death, the policy proceeds will be included in his gross estate. C) The premiums paid by XYZ Corporation will be considered gifts to Albert. D) The premiums paid by XYZ Corporation will be taxable income to Albert. Explanation The policy proceeds will benefit the corporation rather than Albert. Albert had no incidents of ownership in the policy and the death benefit is not available to his estate.

a

The following is a complete list of the lifetime gifts made by Julie: $50,000 to her mother in 1972 before Julie was married $600,000 to her brother in 2001. Her husband, Hank, agreed to split this gift; Hank did not make any taxable gifts in this year. $350,000 to an irrevocable trust in 2012. Julie and Hank were income beneficiaries at a corporate trustee's discretion, and their children were equal remainder beneficiaries at the death of the last parent. Hank did not split this gift. $5,000,000 outright to her cousin late in 2016 after Hank had died the year before. Julie paid $264,000 in gift tax when she filed her gift tax return in 2017. Julie died early in 2020. What is the correct amount of adjusted taxable gifts that Julie's personal representative should add to her taxable estate when computing her federal estate tax? A) $5,276,000 B) $4,155,080 C) $3,619,000 D) $2,776,000 Explanation Because the only two gifts that created adjusted taxable gifts for estate tax purposes are the 2001 and 2016 gifts, $5,276,000 is the correct answer. Because of gift splitting, Julie is responsible for only $300,000 of the 2001 gift and is entitled to one 2001 annual exclusion amount of $10,000. Therefore, Julie's taxable gift for this transfer was $290,000. Julie is responsible for $5,000,000 of the 2016 gift since gifts made after a spouse's death are not entitled to gift splitting, and is entitled to one 2016 annual exclusion amount of $14,000. Therefore, Julie's taxable gift for this transfer was $4,986,000. Adjusted taxable gifts only include taxable gifts after 1976, and therefore, the 1972 gift is not included. The 2012 gift is not included because the trust assets must be included in Julie's gross estate at full date of death value because of her retained income interest. Gifted assets included in the gross estate do not require that the taxable portion of the gift be included as an adjusted taxable gift. The gift taxes paid out of pocket on the 2016 gift will have to be included in Julie's gross estate under the gross-up rule and will entitle her estate to a gift taxes payable deduction from the tentative tax, but does not figure in any way the computation of her adjusted taxable gifts.

a

The preliminary computation of assets in the Hugh Campbell Estate indicates the following: Farm land$6,000,000Farm machinery3,000,000Residue of Campbell estate11,000,000Total gross estate$20,000,000Administrative expenses(300,000)Debts of decedent (mortgage on farm land only)>(900,000)Other Deductions(1,200,000)Adjusted gross estate$18,800,000Taxable estate$18,800,000 Hugh, a U.S. citizen, purchased the farm land six years before his death and personally farmed the land throughout this same period. Hugh's will leaves the land and farm machinery to his son, Hobart, who hopes to continue farming the land. Why is the Campbell estate NOT entitled to use special use valuation in the calculation of estate tax due? A) Because the combined value of the farm land and farm machinery, less secured debts, does not exceed 50% of the gross estate as adjusted for secured debts B) Because the farm property has not been in a qualified use for the required number of years C) Because the land was not used by any of Hugh's family members prior to his death D) Because the decedent's son is not a qualified heir for purposes of special use valuation Explanation In order to qualify for the special use valuation, the value of the farm land and farm machinery, less secured debts, must exceed 50% of the gross estate as adjusted for secured debts.

a

The provisions of all of the following types of state statutes can be altered by express provisions in a will except A) elective share statutes. B) abatement statutes. C) after-born child statutes. D) simultaneous death statutes. Explanation The right to or amount of a surviving spouse's elective share cannot be altered because it is based on the public policy of preventing the total disinheritance of the spouse. The statutes listed in the remaining options all set forth provisions that take effect only in the absence of explicit will provisions.

a

Two years before he died, Alberto gave each of his three children a gift of $100,000. Which of the following statements regarding these gifts is CORRECT? The gifts are not included in Alberto's gross estate. The gifts are included in Alberto's estate tax calculation as adjusted taxable gifts (ATGs). A) Both I and II B) I only C) II only D) Neither I nor II Explanation Both of these statements are correct.

a

Warren's will leaves his house (with the home furnishings), car, and bank accounts to his wife, with the residue to his children. Warren also owned a life insurance policy on his life that named his estate as beneficiary. Four years ago, he assigned the ownership of the policy to his wife, Karen, giving her all incidents of ownership; she has not changed the beneficiary of the policy. The premiums on the policy were paid from a joint bank account held in the names of Warren and Karen. Which one of the following is an estate tax implication for Warren's estate? A) His gross estate will include the policy proceeds because they are payable to his estate. B) The value of the policy is excluded from his gross estate because the assignment to Karen eliminated all his incidents of ownership. C) His estate escapes taxation of the insurance death benefit because the assignment occurred more than three years before his death. D) The marital deduction eliminates the value of the policy from his taxable estate. Explanation Regardless who owns the policy, if Warren is the insured and the death benefits are paid to Warren's estate, they will become an asset of his estate and be subject to estate tax. The policy proceeds would go to his children under the will and thus would not receive an estate tax marital deduction. It is always important to follow life insurance benefits until they find their final owner. In this case, it was not enough to know the death benefits would be in his estate. You also have to know who gets the proceeds to know if they are eligible for the marital deduction or not.

a

When Craig's uncle died, Craig inherited the right to live in his uncle's home for as long as Craig lives. The will also provides that when Craig dies, the home will pass to Craig's daughter, Monica. What type of interest does Monica have in the home at the uncle's death? A) Remainder B) Life estate C) Life estate for a term of years D) Fee simple Explanation Monica has a remainder interest in the home because she has the right to possess and enjoy the home after the intervening right of someone else (Craig). Her remainder interest runs for her lifetime and not a fixed number of years. A fee simple interest represents absolute ownership of property. Monica does not have absolute ownership of the home at the uncle's death. Craig has the life estate.

a

Which of the following are CORRECT statements about the nontax characteristics of a gift to a custodial account under the Uniform Transfers to Minors Act (UTMA)? UTMA places no restrictions on the type of property that may be gifted. UTMA allows both lifetime and testamentary gifts. Both income and principal in an UTMA account may be used for the benefit of the minor during his period of minority. A UTMA account is established by irrevocably registering the gifted property in the custodian's name for the benefit of the named minor. A) I, II, III, and IV B) I and III C) I, III, and IV D) I, II, and III Explanation All statements are true. An important restriction for Uniform Gift to Minors Account (UGMA) accounts, on the other hand, is that they are not allowed to invest in real estate.

a

Which of the following are CORRECT statements about the nontax characteristics of a pooled income fund? It involves a trust created by the grantor solely for the benefit of that grantor and the charity. It must pay a fixed dollar amount to the noncharitable beneficiary annually. The principal is distributed to the charitable beneficiary at the end of the noncharitable beneficiary's life. The income that can be paid to the noncharitable beneficiary is limited to 10% of the original principal unless the beneficiary is older than a specified age. A) III only B) I and III C) I and II D) III and IV Explanation Statement I is false because a pooled income fund is not a grantor trust. Statement II is false because income payments are based on returns of the fund in proportion to the donor's contribution relative to the total value of the fund. Statement IV is false because there is no limitation on the income percentage. A pooled income fund is like owning a mutual fund as joints tenants with right of survivorship (JTWROS) with a charity along with many other co-owners who all get a share of the income.

a

Which of the following are CORRECT statements about the responsibility for payment of the federal generation-skipping transfer tax? When a taxable termination occurs, the beneficiaries receiving distributions from the trust are responsible for paying any generation-skipping transfer tax that may be due. The estate of a decedent is responsible for paying any generation-skipping transfer tax due on a direct skip transfer made by the decedent's will. When a taxable distribution occurs, the trustee of the trust making the distribution is responsible for paying any generation-skipping transfer tax that may be due. The donor is responsible for paying any generation-skipping transfer tax due on a direct skip made during life. A) II and IV B) I, II, and III C) III and IV D) I and III Explanation Option I is incorrect because upon a taxable termination, the trustee is responsible for paying any GST tax due; also, there may not be any distributions with a taxable termination. Option III is incorrect because upon a taxable distribution, the beneficiary receiving the distribution is responsible for paying any GST tax due.

a

Which of the following are estate planning issues that face persons in nontraditional family relationships such as cohabitation? the reduction or elimination of transfer taxes without benefit of certain tax deductions that are available in traditional families ensuring that nonrelatives who are significant to them are included in medical decisions in the event of incapacity ensuring that nonrelatives who are significant to them are able to receive a family allowance clarifying the responsibility for debts and the ownership of specific assets for all involved parties and their heirs A) I, II, and IV B) II and IV C) II and III D) I and III Explanation Nonrelatives are not entitled to a family allowance from an estate (option III). Option I is true because of the unavailability of gift splitting and the marital deduction in such situations. Option II is true because relatives are given priority by law to make such decisions unless the incapacitated person has specifically appointed someone else. Option IV is true because most laws dealing with such matters apply only to legally married couples, which in most states do not include cohabiting couples.

a

Which of the following are important characteristics of the gift tax marital deduction? It enables the donor to avoid gift tax liability by transferring the entire liability for gift taxes to the donee spouse. It allows the donor to avoid gift tax liability on up to one-half of the value of the gifted property that is received by the donee spouse. It allows the donor to avoid gift tax liability on the amount of the gift in excess of the annual exclusion amount. It allows the donor to avoid gift tax liability on a gift to a donee spouse. A) III and IV B) I and III C) II and IV D) I and II Explanation Statement I is false because there would be no gift tax liability for the spouse. The marital deduction would transfer estate tax liability, but not gift tax liability. Statement II is false because it allows the donor to avoid gift tax on 100% of the value.

a

Which of the following is NOT a benefit of using a will substitute for liquidity planning? A) A revocable will substitute will remove the asset from the gross estate. B) A will substitute can avoid ancillary probate on assets owned outside of a decedent's state of domicile. C) A will substitute will reduce estate administration expenses. D) A will substitute allows for flexibility if it is revocable. Explanation A revocable will substitute does not remove the asset from the gross estate; however, it does remove the asset from the probate estate.

a

Which of the following meet the basic requirements for a charitable gift to be deductible for income, gift or estate tax purposes? When Grandpa died, his estate tax was enormous. To lower his estate tax, his heirs decided to contribute $1 million in his name to the Red Cross. A qualified appraisal must generally accompany a tax return reflecting donations of property valued in excess of $3,500. Contributions are only deductible if they are made to a charity as defined by the Internal Revenue Code. Contributions of a split interest must be made in the form of a specified trust or remainder interest. A) III and IV B) II and III C) I and II D) I and IV Explanation Statement I is incorrect. Contributions from an estate directed by the heirs are not deductible for estate tax purposes. Only charitable contributions directed by the decedent are deductible for estate tax purposes. Statement II is incorrect. Qualified appraisals are required when a donated property is in excess of $5,000. Statement III is correct. Only contributions to an organization defined by the Internal Revenue Code are deductible. Statement IV is correct. There are rules about split interest gifts which must be followed.

a

Which of the following most likely describes the reason for including Crummey powers in a trust? A) To ensure that gifts to the trust qualify for the gift tax annual exclusion B) To ensure that the trust complies with the rule against perpetuities C) To protect the trust assets against claims by the beneficiaries' creditors D) To relieve the trust of fiduciary obligations regarding the trust assets Explanation The purpose of including Crummey powers in a trust is to ensure that gifts to the trust qualify for the gift tax annual exclusion.

a

Which of the following postmortem estate planning techniques permits certain types of terminable interests left to a surviving spouse to qualify for the estate tax marital deduction? A) Qualified terminable interest property (QTIP) election B) Election against the will C) Qualified disclaimer D) Family settlement agreement Explanation An executor can use a QTIP election to qualify certain types of terminable interests for the estate tax marital deduction.

a

Which of the following should be included in a will? Provision for guardians of minors Funeral instructions Beneficiaries for retirement accounts A) I only B) II and III C) III only D) I and II Explanation Statement I is correct. Statement II is incorrect. Because the will is often read after the funeral, funeral instructions should be included in a side instruction letter. Statement III is incorrect. Retirement plan assets will automatically pass to the beneficiaries designated on each plan account, regardless of the will.

a

Which of the following situations would NOT constitute a transfer that comes within the gift tax statutes? A) Mary Sue creates a joint bank account for herself and her daughter, Rachel. Rachel has made no withdrawals from the account. B) Carla creates an irrevocable trust that provides that her daughter, Amy, is to receive income for life and that Carla's granddaughter, Hayley, is to receive the remainder at Amy's death. C) Kai creates an irrevocable trust giving income for life to his spouse, Mina, and providing that at her death the corpus is to be distributed to their son, Jace. D) Bill purchases real property and has title conveyed to himself and to his brother, Joe, as joint tenants. Explanation Rachel has made no withdrawals from the account. A gift does not occur regarding the joint bank account until Rachel withdraws funds for her own benefit.

a

Which of the following state a premortem purpose of selling an illiquid asset? Illiquid assets may be sold to increase estate liquidity. Illiquid assets may be sold to ensure fair market value is received for the property. Illiquid assets may be sold to reduce estate shrinkage. lliquid assets may be sold to reduce estate administration expenses. A) I, II, III, and IV B) I and IV C) I and III D) II and IV Explanation All of these are purposes of selling an illiquid asset.

a

Which of the following statements about analyzing the liquidity of an estate are correct: Payment of the decedent's debts must be considered. Payment of the estate's cost of administration such as appraisal fees must be considered. The ease of securing a death certificate must be determined to estimate how quickly survivorship benefits can be paid. A) I, II, and III B) I and III C) II and III D) I and II Explanation Statements I, II and III are true.

a

Which of the following statements about how a specific postmortem planning technique affects the liquidity position of an estate is CORRECT? A) The use of a will substitute, such as a joint tenancy with rights of survivorship, has a positive effect on the liquidity position of an estate. B) Selling an illiquid asset has no effect on the liquidity of an estate because it merely exchanges one asset for another. C) Having a life insurance policy on a decedent in which an irrevocable life insurance trust (ILIT) is both the owner and beneficiary of the policy has no effect on the liquidity position of an estate. D) Assuring that a closely held business has a life insurance policy on a person who is key to the success of the business has no effect on the liquidity position of an estate. Explanation This is the correct answer because the use of a will substitute, such as a joint tenancy with rights of survivorship, has a positive effect on the liquidity position of an estate. A will substitute tends to reduce the administrative expenses of the estate since it transfers ownership automatically at death outside of the probate process.

a

Which of the following statements about the federal gift tax are CORRECT? The federal gift tax applies to all gratuitous transfers. Gift splitting means that spouses may elect to file a joint gift tax return. The unlimited gift tax marital deduction has the effect of abolishing the terminable interest rule. Taxable gifts for prior years must be added to taxable gifts for the current year to determine the tax bracket(s) applicable to the current year's taxable gifts. A) IV only B) I only C) I, II, and IV D) I and II Explanation Certain gratuitous transfers, such as political contributions and direct payment of medical and tuition expenses, are exempt from the gift tax. Statement II is incorrect, as there is no such thing as a joint gift tax return. A marital deduction can be taken only when the gift to the surviving spouse is not a gift of a terminable interest. Thus, one does not abolish the other.

a

Which of the following statements about the nontax characteristics of a charitable remainder unitrust (CRUT) are CORRECT? A fixed percentage (but not less than 5%) of the trust assets, revalued annually, is paid to the charity. Assets can be added to a CRUT in subsequent years. If CRUT assets do not earn enough income to pay the specified income stream, the difference must be paid from the trust corpus. At the end of the trust term, the remaining trust assets are paid to the charity. A) II and IV B) I and III C) II and III D) I and IV Explanation Option I is incorrect because the charity receives the remainder interest rather than the income interest in a CRUT. Option III is incorrect because while the trust agreement may provide for invasion of corpus in these circumstances, it is not mandatory that it do so.

a

Which of the following statements best describe the potential for improper planning with life insurance? A) All of these B) Lack of awareness of how the transfer for value rules work. C) Insufficient coverage D) Tax-inefficient ownership of the policy Explanation All of the statements are true. Life insurance works best if there is adequate coverage undiminished by estate and income taxes. LO 1.3.2

a

Which of the following statements concerning titling of assets captures its importance in estate planning? Titling determines how the income of an asset as well as the proceeds from its sale would be split. The distribution of income or sales proceeds not in fact divided as title dictates likely will create unintended gift and income tax consequences. A) Both I and II B) II only C) I only D) Neither I and II Explanation Both statements are true.

a

Which of the following statements is CORRECT about the income, gift, or estate tax implications of making a gift to a charity of a remainder interest in a farm or personal residence? A charitable gift tax deduction is not available because it is a gift of a partial interest. The farm or residence is included in the grantor's gross estate only if the grantor dies before his actuarially determined life expectancy as of the date of gift. The grantor gets a current income tax deduction for the present value of the remainder interest. The grantor's estate will have to pay an estate tax on the value of the farm or personal residence according to the value used for estate tax purposes. A) III only B) II, III, and IV C) I only D) I, II, and IV Explanation Option I is incorrect because a remainder interest in a farm or residence is an exception to the partial interest rule. Option II is incorrect because the asset is included in the grantor's gross estate under all circumstances because of the reserved right of use and enjoyment. The property will be deducted as a charitable deduction, but it starts out in the gross estate. Option IV is incorrect because of the unlimited charitable estate tax deduction.

a

Which of the following statements regarding a family limited partnership (FLP) is CORRECT? The general partnership interests are transferred to the junior family members and will qualify for minority interest discounts. The transfer of the limited partnership interests to the junior family members is considered a future interest gift and is therefore not eligible for the gift tax annual exclusion. The transfer of the limited partnership interests to the junior family members may qualify for both a minority interest discount and a lack of marketability discount for federal gift tax purposes. A) III only B) II only C) II and III D) I and III Explanation Limited partnership interests (not general partnership interests) are transferred to the junior family members and may qualify for valuation discounts. The transfer of such interests is a present interest and is eligible for the gift tax annual exclusion. Another advantage of the FLP technique is that the senior family member retains the ability to control the business and receive income via retention of the general partnership interest.

a

Which of the following statements regarding charitable contributions is CORRECT? Charitable contributions may qualify for a charitable deduction if made during the donor's lifetime or at death. A decedent's estate qualifies for an estate tax charitable deduction if the decedent's beneficiaries make a contribution to a qualified charity. A) I only B) Both I and II C) II only D) Neither I nor II Explanation Statement II is incorrect because a charitable contribution is deductible for estate tax purposes if it is made by the decedent via the will or by contract but not if made by the beneficiaries.

a

Which of the following statements regarding cross-purchase buy-sell agreements funded with life insurance is CORRECT? The death benefits under the life insurance policies are generally received tax free. The increase in the basis of the surviving owner(s) in the purchased interest(s) will equal the purchase price paid. The number of policies required may become cumbersome if there are a large number of businessowners. A) I, II, and III B) I only C) I and III D) III only Explanation All of these statements are correct. With a cross-purchase agreement, the surviving owners receive the life insurance death benefits income tax free. They then personally pay that amount to the former owner's family. This adds to the basis for the surviving owner(s). With an entity plan, the business itself receives the death benefits and then buys out the deceased owner's share of the business. The surviving owner(s)' basis is not changed.

a

Which of the following statements regarding gifts is NOT correct? A) If a gift is made within four years prior to the donor's death, the amount of any gift tax paid on the transfer is included in the donor's gross estate. B) There may be income tax incentives for making an inter vivos gift, which can involve moving taxable income from a high-bracket donor to a lower-bracket donee. C) When a gift is made during the donor's lifetime, any appreciation accruing between the time of the gift and the date of the donor's death escapes the estate tax. D) An individual can give up to $15,000 gift tax free every year (for 2020) to an unlimited number of donees. Explanation Gift taxes paid on gifts made within three years prior to death are included in the donor's gross estate.

a

Which of the following statements regarding qualified personal residence trusts (QPRTs) is CORRECT? A QPRT may have an interest in only one residence. The residence in a QPRT may not be occupied by anyone other than the grantor and members of the grantor's family. A) Both I and II B) I only C) II only D) Neither I nor II Explanation Both of these statements are correct.

a

Which of the following statements regarding supplemental needs trusts is(are) CORRECT? A special needs trust is typically used to help with the needs of a dependent who is developmentally disabled and who is receiving some form of government assistance. Special needs trusts are not permitted in all states. A) Both I and II B) I only C) Neither I nor II D) II only Explanation Both statements are correct.

a

Which of the following statements regarding tax planning techniques for unmarried cohabitants is NOT correct? A) Converting solely owned property to tenancy in common property between unmarried cohabitants will ensure that the surviving cohabitant will eventually own the entire property. B) Outright transfers between unmarried cohabitants entitle the donor to an annual exclusion. C) Converting solely owned property to JTWROS property between unmarried cohabitants will ensure that the surviving cohabitant will eventually own the entire property. D) Having life insurance death benefits available for estate liquidity may be particularly important for an unmarried cohabitant. Explanation Since each tenant in common is entitled to dispose of his interest to anyone, there is no guarantee that the interest of the first cohabitant to die will pass to the surviving cohabitant. It definitely will not pass to the surviving cohabitant unless it is passed in the deceased tenant's will. JTWROS titling would accommodate this. Since the only requirement to be entitled to the annual exclusion is that the donee be given a present interest in the property, the donor will be entitled to an annual exclusion.

a

Which of the following statements regarding the filing of an estate tax return is CORRECT? An estate tax return must be filed if the value of the gross estate plus adjusted taxable gifts on the date of death exceeds $11,580,000 in 2020. The estate tax return is due nine months after the decedent's date of death, although a six-month extension of time to file may be requested. A) Both I and II B) I only C) Neither I nor II D) II only Explanation Both statements are correct.

a

Which of the following statements regarding the goals of estate planning is CORRECT? A) A client may have to choose between two or more estate planning techniques that will achieve his or her objectives. B) An estate planner must choose between two or more estate planning techniques that will achieve his or her client's objectives. C) A client and an estate planner must mutually agree on the client's most important estate planning objective in the event that all such objectives cannot be satisfied. D) An estate planner is in the best position to determine which goals the client should prioritize. Explanation Although there may be two or more techniques that will achieve the client's stated objectives, the client's choice will be based on a preference for one technique's characteristics or tax consequences over those of the other technique(s).

a

Which of the following statements regarding the use of a cross-purchase buy-sell agreement as a premortem liquidity planning device is CORRECT? The purpose is to ensure that the owner of a closely held business interest will be able to sell her interest in specified circumstances. The business entity contracts to pay each owner an agreed-upon amount for his interest in the business under certain circumstances. If a business interest is sold under such an agreement at an owner's death, estate liquidity is enhanced because the estate will be liable for little or no capital gain. If insurance is used to fund such an agreement, the owner of the policy used to accomplish the purchase will be the purchasing owner. A) I, III, and IV B) II and IV C) I and III D) I, II, and IV Explanation Only option II is an incorrect statement. In a cross-purchase buy-sell agreement, the parties obligated to purchase an offered interest are the other individual business owners—the business entity is not a contracting party. If insurance is used, each owner takes out a life insurance policy on every other owner who is a party to the agreement and uses the proceeds of such a policy to fulfill her obligation to purchase that owner's interest under specified conditions. If an estate sells the owner's interest at death, the estate's basis in the business interest will equal its estate tax value (stepped-up), and therefore, the sale will produce little or no capital gain, which allows the estate to keep more of the sale proceeds for liquidity purposes.

a

Which of the following statements regarding the use of the alternate valuation date (AVD) for estate tax purposes is NOT correct? A) The executor is allowed to pick and choose which assets will be valued as of the decedent's date of death value and which will be valued at the AVD. B) The AVD election allows the executor to value estate assets at their fair market value six months after the decedent's date of death. C) Wasting assets must be valued at their date of death fair market value, even if the AVD election is made. D) The AVD election cannot be made unless it results in a reduction of the amount of federal estate tax owed by the decedent's estate. Explanation The AVD election is an all-or-nothing choice, and the executor is not allowed to pick and choose which assets will be valued as of the decedent's date of death and which will be valued at the AVD.

a

Which of the following statements regarding types of gifts is CORRECT? A) A gift of a future interest is not eligible for the gift tax annual exclusion. B) Indirect gifts, such as the payment of another's expenses, are not subject to the gift tax. C) A gift of a future interest is not subject to gift tax. D) The gift tax applies to incomplete gifts. Explanation Indirect gifts, such as the payment of another's expenses, may be subject to the gift tax. The gift tax does not apply to incomplete gifts. The gift tax applies to gifts of future interests as well as to gifts of

a

Which of the following statements regarding unfunded irrevocable life insurance trusts (ILITs) is CORRECT? With an unfunded ILIT, the grantor gifts cash to the trust each year to pay the life insurance premiums. An unfunded ILIT is treated as a grantor trust for income tax purposes. An unfunded ILIT must include Crummey powers if the grantor wants to use the gift tax annual exclusion. A) I and III B) II only C) I, II, and III D) II and III Explanation Statement II is incorrect. A funded ILIT is treated as a grantor trust, but an unfunded ILIT is not.

a

Which of the following terms best describes the trusts created in Michael's will for the benefit of his 4 children? A) Testamentary trusts B) Marital trusts C) Grantor-retained trusts D) Inter vivos trusts Explanation These are testamentary trusts because they are created by Michael's will and made effective at his death. An inter vivos trust is made effective during the grantor's lifetime. A marital trust is established for the benefit of a surviving spouse and is designed to take advantage of the estate tax marital deduction. A grantor-retained trust is established during the grantor's lifetime and is designed to allow the grantor to retain income or the use of transferred property for a limited time.

a

Which of the following transactions are subject to the Chapter 14 valuation rules? Corporate recapitalizations Partnership capital freezes Grantor retained trusts, such as GRITs Buy-sell agreements between non-family members A) I, II, III, and IV B) I and II C) I, II, and III D) IV only Explanation All of these estate freeze transactions are subject to the Chapter 14 valuation rules.

a

Which of the following transfers are gifts for purposes of the gift tax statutes? Kurt creates an irrevocable trust providing that his son is to receive income for life and his grandson the remainder at his son's death. Kurt purchases real property and has the title conveyed to himself and to his brother as joint tenants. Kurt creates an irrevocable trust giving income for life to his spouse and providing that upon her death the corpus is to be distributed to his daughter. Kurt purchases a U.S. savings bond made payable to himself and his spouse. The spouse later surrenders the bond for cash to be used for her benefit. A) I, II, III, and IV B) I, II, and IV C) I and II D) II and III Explanation All of these transfers are gifts for purposes of the gift tax statutes. Statement IV falls under the gift tax statutes, but the unlimited marital deduction may be utilized to offset any possible gift tax due.

a

Which one of the following is a method by which assets can be transferred both during life and at death? A) Irrevocable trust B) Right of survivorship C) General power of attorney D) Testamentary trust Explanation An irrevocable trust can be used to transfer assets both while the grantor is alive (inter vivos) and when the grantor is dead (testamentary).

a

Which one of the following is an incorrect statement concerning tax implications of lifetime gifts? A) If a gift is gift taxable, it is ignored in calculating the donor's estate tax liability. B) Gifting loss property eliminates the possibility of using capital losses to offset capital gains. C) In an outright gift, gifting assets reduces estate tax liability by eliminating all tax on future appreciation that otherwise would be part of the donor's gross estate at death. D) The basis for income tax purposes is not stepped up on assets received as gifts. Explanation The answer is if a gift is gift taxable, it is ignored in calculating the donor's estate tax liability. This is because the question asks for the identification of the incorrect statement about the tax implications of lifetime gifts. It erroneously states that taxable gifts are ignored in calculating the donor's estate tax liability. Remember, the transfer tax system is cumulative. Therefore, the taxable portions of gifts are added to the taxable estate to establish the estate's tax base against which the tax rate is applied, unless the gift is included in the gross estate.

a

Which one of the following statements about the use of a split-dollar life insurance plan in a business setting is false? A) The entire value of the policy and its benefits are subject to claims of the employer's general creditors. B) The insurance premiums are typically split between the employer and the employee/insured. C) The insurance can act as a fringe benefit to a valued employee. D) The benefits from the policy are split between the employer and the employee/insured or the employee's beneficiary. Explanation Only the benefits payable to the employer under the split-dollar agreement are subject to claims by the employer's creditors. All other statements are true.

a

Which one of the following statements is CORRECT about the characteristics of a standby (contingent) trust? A) It is often used to manage the grantor's financial affairs when the grantor becomes incompetent. B) It is a type of testamentary trust. C) It is fully funded at the time it is created. D) The grantor acts as trustee once the trust is funded. Explanation The answer is it is often used to manage the grantor's financial affairs when the grantor becomes incompetent. The trust must be established inter vivos to handle the grantor's financial affairs when he becomes incompetent. The trust is only minimally funded upon creation. Full funding will occur only when the grantor becomes incompetent. If the grantor is incompetent, he cannot act as trustee.

a

Which one of the following statements regarding different forms of property co-ownership is CORRECT? A) Joint tenancy with right of survivorship (JTWROS), tenancy by the entirety (TBE), and community property (CP) are all forms of co-ownership that can be used by a husband and wife. B) Joint tenancy with right of survivorship (JTWROS), tenancy by the entirety (TBE), and tenancy in common are all forms of co-ownership that require the consent of other co-owners before an owner can sell his or her interest in the asset. C) Tenancy by the entirety (TBE) and tenancy in common are the only two forms of co-ownership specifically for spouses. D) Joint tenancy with right of survivorship (JTWROS), tenancy by the entirety (TBE), and community property (CP) are all forms of co-ownership that do not require a probate proceeding when one tenant dies. Explanation JTWROS can be used by anyone, including spouses; only spouses can use TBE and CP.

a

Wiley Willmaker lives in a common law state that has adopted the Uniform Probate Code. Wiley has been married to Jane for 28 years. All family assets are in Wiley's name only. His will, executed in 2001, is structured as follows: It leaves specified tangible personal property and all real property, which amounts to 28% of his estate, to Jane. It leaves specified tangible and intangible personal property, which amounts to 45% of his estate, to "my children, Carrie and Maureen, in equal shares." It leaves specified intangible personal property, which amounts to 25% of his estate, to a qualified charity. It is attested to and witnessed by his daughter, Maureen. No other provisions disposing of property exist. Wiley has two children from his current marriage, Maureen and Chad, and a daughter, Carrie, from his prior marriage. Relations with Chad have become strained, and Wiley feels that Chad has been so disrespectful he should not receive any property from Wiley. Which of the following statements is CORRECT about why Wiley's will needs to be amended? The will needs to be properly witnessed by a disinterested party. Chad may still be able to receive a share of Wiley's property unless the current will is changed. The estate will not be subject to the intestacy statutes. The will needs a residuary clause. A) I, II, and IV B) I and III C) II and IV D) II, III, and IV Explanation Option III is an incorrect statement—here 2% of his estate does not have a named beneficiary. Although it may not be legally necessary to execute a new will without Maureen as a witness (option I), it is advisable to do so to prevent a possible will contest by Chad. While a witness who is a beneficiary of the will increases the chances of a will contest based on undue influence on the testator, it does not make the will invalid per se. However, most states require a will to have two witnesses, and this will has only one. This will may be invalid because of too few witnesses. Chad may still inherit (option II) by arguing that his name was mistakenly omitted from the specific listing of the children—he should be specifically disinherited by name or specifically given a small bequest. Finally, to avoid intestacy (option IV), Wiley's will needs a residuary clause ("rest and remainder to . . .") to indicate how assets not disposed of by other will provisions will be distributed—here 2% of Wiley's assets would pass by intestacy, which is another way in which Chad might inherit a portion of the estate.

a

Your client died recently with a gross estate valued at $425,000. Her estate tax bracket is 34%. Her husband has a gross estate of $155,000. Her will (1) placed $350,000 in a trust that gave her husband and her mother life income interests payable annually, with the trust remainder going to her son from a previous marriage when both have died; and (2) left the residue to her daughter. Her husband was named executor. Most of her estate is in certificates of deposit and securities that recently have shown only minimal growth. During the last six months of her life, she had uninsured medical expenses of $30,000. She and her husband were in the 24% marginal income tax bracket at the time of her death. If her husband, as executor, came to you for advice, you should inform him that the most appropriate postmortem election for his wife's estate is A) the claiming of medical expenses on her final income tax return. B) the waiver of executor fees charged to her estate. C) the use of the alternate valuation date (AVD) for her estate assets. D) the making of a qualified terminable interest property (QTIP) election by her executor. Explanation The estate is way too small to need a QTIP election. Waiving executor fees will help, but not as much as claiming the medical expenses on her final income tax return. With clearly no chance of owing estate taxes, the AVD is not an option.

a

Your client established a funded revocable life insurance trust and transferred a $500,000 face value whole life insurance policy on his life into it. Your client's father was named trustee. The client has made no other lifetime gifts. If the client dies 18 months after establishing the trust, one disadvantage of this life insurance planning technique is that A) the face amount of the transferred policy will be includible in the client's gross estate. B) the client will have a gift tax liability for the value of the policy. C) the amount of gift tax paid will be includible in the client's gross estate under the gross-up rule. D) the premium payments will be considered taxable income to the beneficiary. Explanation For gift tax purposes, the value of a life insurance policy is the replacement cost, but for estate tax purposes, it is the face amount that is included and because death occurred within three years of the transfer, the face amount is brought back into the gross estate.

a

Your client is the sole depositor of $30,000 in a bank account. He is considering naming his girlfriend as the other joint tenant on the account. You should inform him that one consequence of creating the joint tenancy with right of survivorship bank account would be that A) the client's contributions to the account will be excluded from his probate estate. B) the other joint tenant, his girlfriend, may not withdraw the funds from the account without his consent. C) all funds can be withdrawn by either the client or his girlfriend without gift tax consequences. D) the client's contributions to the account will be excluded from his gross estate. Explanation A joint tenancy with right of survivorship (JTWROS) designation is a will substitute that will allow this asset to pass outside of probate. The girlfriend, once added, can make withdrawals without his consent. The account value will be included in his gross estate and, since the girlfriend did not contribute to the account, any withdrawals she makes are gifts.

a

Your client's spouse died this year with a gross estate valued at $5.5 million, which included two solely owned apartment buildings, each valued at $2 million. Debts and expenses payable by her estate are estimated at $250,000. During the past three years, she gave most of the rents from the apartment buildings directly to her son's university to pay for his graduate school tuition. She and your client owned a personal residence worth $1 million as joint tenants with right of survivorship. Her will, which appoints her brother the executor, leaves all her solely owned property to her son, with the remainder to your client. If your client's major objective is to maximize his share of his spouse's estate, which of the following is the most advisable postmortem action for him to take? A) Make an election against his spouse's will as the surviving spouse B) Ask the executor to make a QTIP election C) Consent to split gifts made by his spouse to her son during the past tax year D) Petition the executor to elect installment payment of estate taxes under IRC Section 6166 Explanation Your client will receive much more of his spouse's estate by statute than the will allows. This is reason for the statute—to protect spouses from being disinherited.

a

Your data gathering meeting with Colin Greywhale indicated the following about his property interests: He has the right to decide, without limitation, who will receive the entire corpus of his uncle's trust. Colin's will does not exercise this right. His wife owns a paid-up life insurance policy that insures Colin's life; his wife, Lois, is the primary beneficiary. Five years ago, he created the revocable CG Trust for his children and funded it with $80,000 worth of securities; two years ago, when the trust fund was worth $130,000, he made it irrevocable. Two years ago, he had cumulative taxable transfers that exceeded the applicable exclusion amount and paid $32,000 in gift tax to the federal government. If Colin died in the current year, which one of the following would be excluded from his gross estate? A) The death benefit of the life insurance policy B) The value of the corpus of his uncle's trust C) The $32,000 in gift taxes paid D) The date-of-death value of the CG Trust Explanation Colin's right in his uncle's trust is a general power of appointment, which would cause the maximum value that could be appointed at death to be in the holder's (Colin's) gross estate. He has no incidents of ownership in the life insurance policy on his own life, which would cause inclusion of the death benefit in his gross estate, nor is his estate the beneficiary. There is no indication that Colin assigned ownership of this policy to his wife. If a right to a retained right to alter, amend, or revoke is given up within three years of death, as it was when the CG Trust was made irrevocable, the value of the assets over which the right was retained must be included in the grantor's gross estate. Finally, any gift taxes paid "out-of-pocket" (tax liability beyond the gift tax applicable credit amount) on gifts made within three years of death must be included in the gross estate under the gross-up rule.

a

A CFP® professional is working with a client to develop the client's estate plan. As part of this process, the CFP® professional and client are attempting to project the estate's cash flow needs and develop a plan to ensure the estate's liquidity. The client owns several life insurance policies on his own life and expects his estate will be subject to estate tax. Which of the following statements regarding the development of the cash flow plan to maintain the liquidity of the client's estate is NOT correct? A) The timing of some of the estate's cash outflows will be fairly predictable. B) It is generally not possible to reduce the estate's cash needs for estate tax. C) The cash flow plan should be flexible enough to account for the possibility of unexpected expenses. D) It should be anticipated that there may be a delay in receiving life insurance proceeds on the client's life. Explanation It may be possible to reduce the estate's cash needs by gifting the life insurance policies to get them out of the gross estate. If effective, this will lower the cash needs by the estate tax on the death benefits. To be effective, the donor, who owns the policies and is also the insured, must gift the policies more than three years before death. This is a situation in which an irrevocable life insurance trust (ILIT) would serve well. The death benefits would be out of the gross estate (assuming the transfer was more than three years old). The trustee of the trust might choose to use the life insurance death benefits received by the ILIT to pay the estate taxes. The trustee could also give or lend the money to the deceased's family to provide liquidity for debts, taxes, and administrative expenses. Any remaining assets in the ILIT would be distributed to the trust beneficiaries.

b

A characteristic of the sole ownership form of property ownership is that the property A) owner does not control disposition at death. B) does not avoid probate. C) can benefit multiple parties. D) avoids probate. Explanation Property owned in sole ownership must be probated at death.

b

All of Mark's property has been placed into a revocable trust. Mark is the trustee of this trust until he dies or becomes incompetent. Mark, his spouse, and his children are all income beneficiaries of the trust, with income to be distributed at the trustee's sole discretion. At Mark's death or his incompetency, his spouse is appointed successor trustee. The trust is to continue until his spouse's death, when the trust assets are to be distributed per capita at each generation to Mark's surviving descendants. Which one of the following correctly identifies advantages or disadvantages Mark can achieve by using this method of estate transfer at death? A) Shifting taxation of income produced by the trust assets B) Providing a stepped-up basis in estate assets to the beneficiaries of his estate C) Protecting estate assets from the claims of his creditors D) Freezing the value of his assets for estate tax purposes Explanation Because the trust assets will be in Mark's gross estate (because he has retained the right to revoke), they will be entitled to a step-up in income tax basis. The trust assets will be subject to the claims of Mark's creditors because the trust is revocable. The trust assets will be valued in his gross estate at fair market value as of the date of death or six months later (the alternate valuation date). Mark will still have to report all trust income under the grantor trust rules because the trust is revocable.

b

All of the following intrafamily transfers involve the use of a trust except A) QPRTs. B) FLPs. C) GRATs. D) GRUTs. Explanation Transfers using FLPs (family limited partnerships) involve the use of gifts, not trusts.

b

Arnie attended a recent seminar about the importance of asset protection. He wants to know which of the following would not provide that protection. A) Gifting assets away before incurring any legal judgments. B) Placing assets in a revocable trust. C) Placing assets in an entity like a corporation or LLC. D) Placing assets in trust with a spendthrift clause. Explanation Assets in a revocable trust would be subject to Arnie's ability to demand them. Accordingly, there is no asset protection.

b

Betina is a single taxpayer in the maximum current marginal income tax bracket. She owns stock that has appreciated rapidly in recent years. Five years ago, she paid $18,000 for the stock, which now has a fair market value of $72,000. She would like to gift the stock to her nephew, Jerry, who is in the lowest marginal income tax bracket. Which one of the following is an income tax implication of Betina's proposed transfer? A) Jerry's basis is limited to income tax paid on the stock's net appreciation over the five years Betina owned the stock. B) Jerry's basis is Betina's cost in the stock plus an adjustment for the portion of any gift tax paid out-of-pocket on the transfer that is due to the appreciation of the stock since the original owner purchased it, if gift taxes are actually paid out-of-pocket. C) Jerry receives a stepped-up basis in the stock based upon its fair market value at date of transfer. D) Betina receives an income tax deduction for the excess of the stock's fair market value over her cost basis. Explanation The donee of a gift assumes the donor's basis with an adjustment for the portion of gift tax paid out-of-pocket (if any) that is attributable to the gain element (fair market value of gift minus donor's basis). Betina has not paid any income tax on the gain, and would therefore make Jerry's basis zero; Jerry will assume Betina's basis as adjusted. Income tax deductions are not allowed for gifts to noncharitable beneficiaries.

b

Bill gifts his daughter bonds with a fair market value (FMV) of $1,000. Bill's basis in the bonds is $1,800. Bill's daughter subsequently sells the bonds for $2,000. What is her recognized gain or loss? A) $1,000 gain B) $200 gain C) $2,000 gain D) No gain or loss Explanation On the date of the gift, the FMV of the property was less than the donor's basis. Therefore, the double-basis rule applies. The daughter's gain basis is $1,800. She has a recognized gain of $200, which is calculated as follows: Amount realized$2,000Basis for gains(1,800)Realized gain$200Recognized gain$200

b

Bill's estate paid $8,175 in fees to appraisers and $2,380 in medical bills from his last illnesses that were not covered by his medical insurance. Bill's spouse, who was the personal representative of his estate, elected not to take the $5,000 fee to which she was entitled by statute. Bill's estate did pay his spouse the $705,000 to which she was entitled by virtue of her demand to receive an elective share under the appropriate state statute. Which of the following items may be deducted from Bill's gross estate to calculate his taxable estate? $8,175 in appraisal fees $5,000 personal representative fee $2,380 in medical bills $705,000 elective share A) II and IV B) I, III, and IV C) I and III D) I only Explanation Appraisal fees are a deductible administrative expense if reasonable in amount. The personal representative's fee can be deducted only if actually taken. The medical bills may be deducted the same as any other valid debt because they were not reimbursed by insurance. Although passed by state statute to the spouse rather than by Bill's express direction, the elective share qualifies for the marital deduction and thus may be deducted from the gross estate to calculate the taxable estate.

b

C.J. Yee's taxable income fluctuates from year to year. Her adjusted gross income this year is $60,000, but it will probably be less in the future. Her major objective is to reduce income taxes for the current year. She plans to contribute $35,000 to the American Red Cross. Which one of the following is the most appropriate property for C.J. to gift to the American Red Cross to maximize her current-year charitable income tax deduction? A) A stamp collection she inherited, held long term, valued at $35,000 with a basis of $12,000 B) A life insurance policy with a face value of $35,000; net premiums, paid by C.J., of $24,000; and a replacement cost of $30,000 C) A stock, held short term, valued at $35,000 with a basis of $19,000 D) A tract of farmland, held long term, valued at $35,000 with a basis of $22,000 Explanation A gift of the life insurance policy would allow C.J. to take a current-year income tax deduction of $24,000, which is more than any other listed asset. It is rarely wise to gift short-term capital gain property because only the basis can be deducted as it in this case would allow a current deduction of $19,000. Use-unrelated tangible personal property is not a good choice because what would the Red Cross do with a stamp collection? They would sell it and use the proceeds for their charitable work. Donations that are sold by the charity within two years of the gift are consider use-unrelated. That limits the charitable deduction to the basis, which in this case would only allow a current deduction of $12,000. Farmland is ordinary income property. It you own farmland, you are a farmer by definition. The deduction for ordinary income property is always limited to the basis. In this case, donating the farmland to charity would only allow a maximum current-year deduction of $22,000.

b

Calvin is a married taxpayer whose income is subject to the maximum marginal income tax rate. He would like to transfer some property to his adult daughter, who is just starting out in business and currently is in a significantly lower income tax bracket. Calvin's major objective is to reduce his own income tax liability. The following assets are available for Calvin to give to his daughter: BasisFMVCash$75,000$75,000Corporate bond$85,000$75,000Rental real estate$50,000$75,000Leased equipment$69,000$75,000 The corporate bond is returning income. The rental real estate also is producing significant taxable income after 30 years of sizable depreciation deductions; Calvin's daughter would retain the property for its income and use it as collateral. The equipment was recently purchased and is generating large depreciation deductions. Assuming that each property is producing approximately the same amount of income, which one of the following is the most appropriate property to gift to the daughter? A) The cash B) The rental real estate C) The corporate bond D) The leased equipment Explanation Gifting the rental real estate will eliminate both the rental income and the unrealized capital gain, thereby accomplishing Calvin's objective of reducing his income tax liability. The corporate bond would not be good property to give as it is loss property, which Calvin could use to offset other capital gains. Calvin can also use the depreciation deductions generated by the leased equipment (option b.) against other income. Giving the cash will not likely reduce Calvin's taxable income as much as the rental real estate. Additionally, giving the rental real estate will eliminate its future appreciation potential, while cash does not appreciate.

b

Carrie is preparing her estate plan and wants to include instructions for her funeral. What method should she use? A) She must include these instructions in a formal, notarized list of instructions drawn up by her attorney. B) She should leave written instructions to one or more family members likely to survive her and be involved in funeral arrangements. C) She should include her instructions in her will because it will be the first thing that will be read after her death. D) She must include these instructions in a codicil to her will. Explanation Carrie should leave a letter of personal instruction or side letter. It can be as simple as a handwritten note to her personal representative containing her instructions.

b

Case Study Question If Marie dies today, which of the following statements is(are) CORRECT? Marie has died intestate. The ABC stock will pass through probate. State intestacy law will determine who receives the ABC stock after Marie's death. A) II and III B) I, II and III C) I only D) I and II Explanation All of these statements are correct. Marie has died intestate because she does not have a valid will. The ABC stock is titled in Marie's name alone, so it will pass through probate when she dies. State intestacy law determines who receives the ABC stock because Marie has died without a valid will.

b

Catherine Rich established trusts under Internal Revenue Code Section 2503(c) for each of her eight grandchildren this year and funded each trust with $926,000. Earlier this year, Catherine also paid $25,000 to the local community hospital for medical bills incurred by her niece. Assuming that all of Catherine's children are living at the time of these transfers, which one of the following statements are CORRECT regarding application of the generation-skipping transfer tax (GSTT) to these transfers? A) The transfer to pay medical bills is an indirect skip. B) The transfers to the trusts are direct skips. C) The transfer to pay medical bills is a direct skip. D) The transfers to the trusts are indirect skips. Explanation The transfers to the trusts for the grandchildren are direct skips because each grandchild is a lineal descendant of a grandparent of Catherine and is two generations further removed from this grandparent than is Catherine. The trusts are skip parties because each trust has only one beneficiary who is a skip party in relation to Catherine. The payment of the medical bills is exempt from GSTT because the provider is paid directly.

b

Chip and Pam, a married couple, are both 65 years old and live in a common-law property state. They have a total combined estate of $10 million, of which each owns half as their sole and separate property. They have three grown children, and Chip has one adult child from a prior marriage. Chip and Pam have the following objectives: To allow each estate to use some or all of its estate tax applicable credit amount To allow part of the estate of the first spouse to die to qualify for the marital deduction That, according to Chip's wishes, his children from his current marriage and his prior marriage share equally in his wealth after Pam's death, if he dies first That, according to Pam's wishes, only her children from her current marriage will share equally in her wealth after Chip's death, if she dies first To be assured that the surviving spouse will have a right to an annual income stream from at least part of the decedent spouse's assets Based on Chip and Pam's objectives, which of the following is the most appropriate testamentary transfer technique or combination of techniques for them to use? Assume 2020 estate tax law in making your decision. A) Family bypass (B) trust and power of appointment (A) trust B) Family bypass (B) trust and QTIP (C) trust C) Power of appointment (A) trust D) QTIP (C) trust and power of appointment (A) trust Explanation The A Trust is a marital trust, and as such, it would not allow the first decedent to use any of his estate tax applicable credit amount, or it could not be assured that his wealth would go only to the beneficiaries desired. The A Trust would not allow the first decedent to be assured that his wealth would go only to the beneficiaries desired. The C Trust would meet the income and distribution objectives and, in addition, would meet the use of the applicable credit amount objective if the QTIP election is not made, the marital deduction objective if it is made, or both if a partial election is made. However, since the C Trust is not listed alone in any option, it must be paired only with an A Trust to meet both spouses' distribution objectives of controlling where the assets go at the second death. Assets that are to receive the marital deduction could be placed in the C Trust, with all other assets being placed in the B Trust.

b

Cisero gifted 2,000 shares of his stock in a closely held corporation to his daughter. These shares constitute half of the total number of shares he owned in the corporation prior to the transfer. The value of Cisero's stock in this corporation prior to the transfer was $400,000. Which one of the following statements is CORRECT regarding the value of the stock transferred to his daughter? A) Cisero would have to compute his gift tax on $400,000 because of the Chapter 14 rules. B) Cisero would be able to claim a lack of marketability discount when computing his gift tax for this transfer. C) Cisero would be able to choose special use valuation when computing his gift tax for this transfer. D) Cisero would be able to claim a blockage discount when computing his gift tax for this transfer. Explanation This transfer would not qualify for a blockage discount as it is not publicly traded stock. It would not qualify for special use valuation either, but a reasonable lack of marketability discount is available and would not violate Chapter 14 rules.

b

Debra is the majority stockholder in the Jernigan Family Corporation. Her interest in this corporation is the sole asset in her estate other than her personal use assets. She understands that if she were to die today, an unacceptably large amount of her assets, including the closely held business interest, would have to be sold to pay her estate administrative expenses and death taxes. She would like to do something with her interest in the Jernigan Family Corporation to prevent the need for such a sale, because she would like her children to receive her shares. Which of the following actions have the potential to increase the liquidity of Debra's estate? Have the corporation purchase a key person life insurance policy on Debra Gift shares in the Jernigan Family Corporation to her children Amend her will to place her shares into a testamentary trust for her children Establish an irrevocable life insurance trust (ILIT) and fund it with a policy on her life A) I and II B) I, II, and IV C) II and III D) I, III, and IV Explanation Statement III is false because establishing a testamentary trust will not increase liquidity, but will decrease it since they would go to the children rather than be available for estate cash needs.

b

Dorothy gifts her vacation condo to her grandson but reserves the right to use the condo whenever she chooses for the rest of her life. The market value of the condo at the time of the gift is $300,000. When Dorothy dies, the condo has a market value of $350,000. What amount is included in Dorothy's gross estate for estate tax purposes? A) $300,000 B) $350,000 C) $50,000 D) $0 Explanation Because Dorothy retained the right to use the condo for the rest of her life, the entire fair market value of the condo on Dorothy's date of death is included in her gross estate.

b

During her lifetime, Ella has made several property transfers. As her financial advisor, which of the following transfers would you tell her are completed transfers fully subject to gift tax for the purpose of calculating her federal gift tax liability? The $5,000 in cash for each of her nieces and nephews that she placed in a revocable trust last year for their benefit The $13,000 she paid four years ago to Pembroke Hospital to pay off her mother's medical bill The $8,000 remainder interest given to her son, Zeke, two years ago in the Ella D'Arno Irrevocable Grantor Retained Income Trust The $9,000 in cash she gave six years ago to her then husband, Roberto (this was her sole gift to Roberto that year) A) II and IV B) III only C) I and IV D) I, II, and III Explanation Completed transfers subject to gift tax include completed transfers that are entitled to the gift tax marital and charitable deductions. Therefore, these reductions are irrelevant and gifts that involve marital or charitable deductions are subject to the gift tax rules. On the other hand, completed transfers covered entirely by the gift tax annual exclusion are not subject to gift tax because they are not included in the donor's total calendar year gifts. Said differently, the amounts subject to gift taxes are what is included in the donor total calendar year gifts even if some or all of the gifts eventually wind up being gift tax deductible. This is similar to asking what is in the gross estate even if the asset will later be estate tax free due to the marital or charitable deduction. Option I is not subject to gift tax because gifts to the revocable trust are incomplete gifts. Option II, direct payments to a medical provider on another person's behalf, are not subject to gift tax by statute. Next, the remainder interest in a grantor retained income trust, option III, is included in the donor's total gifts, in spite of the retained income interest. In fact, the retained interest with a GRIT is also included as a taxable gift because of the Chapter 14 rules. The retained interest is a taxable gift with a GRIT in the sense that with a GRIT the retained income interest is valued at zero because the retained interest is not a qualified interest. With the GRIT the transfer is complete—the grantor cannot take the assets back. Option IV, the cash given to her husband six years ago, was not subject to gift tax because of the annual exclusion, although it was a completed transfer.

b

Eduardo dies on March 15 of the current year. His gross estate consists mostly of publicly traded stocks having a market value of $15 million on his date of death. A severe bear market begins within a few weeks after he dies, and the value of his stocks declines to $13 million by September 1. Which of the following postmortem estate planning techniques might be useful to Eduardo's estate? A) Special use valuation B) Alternate valuation date (AVD) election C) Qualified disclaimer D) QTIP election Explanation An election to use the AVD would be useful because it allows the estate to value estate assets for estate tax purposes at their value six months after the date of death rather than on the date of death. Special use valuation allows the estate to value qualifying real property on the basis of its actual use rather than its highest and best use. A QTIP election allows the estate to take the marital deduction for bequests of certain terminable interests, and a qualified disclaimer allows a beneficiary to refuse a bequest or inheritance.

b

Emmet's personal representative is computing his estate tax liability. Among other provisions, Emmet's will provides the following relating to his spouse, Nellie, who is still alive more than seven months after Emmet's death. It grants Nellie a life estate in his residence with the remainder at her death to go to Emmet's only son, John; the executor does not plan to make a qualified terminable interest property (QTIP) election for this property. It establishes Trust 1, on the condition that Nellie survive him by at least three months. This trust grants all income to Nellie for life (to be paid at least annually) and grants her a general power of appointment over the corpus. It establishes Trust 2, naming John and Nellie as income beneficiaries, to receive distributions of income at the trustee's discretion, with the remainder at Nellie's death to go to John. Since Emmet's will did not contain a residuary clause, his remaining assets passed to Nellie pursuant to state intestacy laws. Which of these interests left to Nellie qualify for the marital deduction? A) I and IV B) II and IV C) II and III D) I and III Explanation Option II is a power of appointment, or "A" trust, which gives Nellie the equivalent of outright ownership—she gets all income and can use the general power of appointment to appoint all corpus to herself. The property in option IV, which passes to Nellie by the intestacy laws, qualifies for the marital deduction. Option I is QTIP, which can qualify for the marital deduction only if the proper election is made. Option III is a bypass or family trust, which grants Nellie only a terminable interest (i.e., an income interest that terminates at her death). It does not qualify for the QTIP election because she is not given a qualifying income interest for life.

b

For purposes of the generation-skipping transfer tax, any payment of income or principal from a trust to a person two or more generations junior to the donor's generation is a A) direct skip. B) taxable distribution. C) taxable termination. D) qualified transfer. Explanation Payment of income or principal is a distribution, not a transfer or termination. A distribution might be to a direct skip, or it could be to an indirect skip.

b

Frank is a widower. He has a $17.2 million estate consisting primarily of undeveloped real estate and life insurance. His children are the beneficiaries of his life insurance. His will leaves $900,000 of probate assets to each of his three children, with the residue to his cousin, James. Frank learned that his estate may have liquidity problems when he dies. Which one of the following techniques is the most appropriate to increase liquidity in Frank's estate? A) Change the beneficiary on his life insurance to his estate B) Transfer existing life insurance policies to an irrevocable life insurance trust with his children as beneficiaries, which allows the trustee to purchase some of the hard-to-sell property from the estate and/or to loan funds to the estate C) Place the undeveloped real estate in a qualified terminable interest property (QTIP) trust and make the QTIP election D) Amend his will to place the undeveloped real estate in an estate trust Explanation Neither a QTIP trust nor an estate trust will change the liquidity situation. Naming the estate as beneficiary would cause that asset to go through probate and doesn't change the dollar amount that the children will receive.

b

George and Claire are married and have four children. If they elect gift splitting, what is the maximum amount of total gifts they can make to their children in 2020 and have the gifts be covered by their annual exclusions? A) $60,000 B) $120,000 C) $15,000 D) $30,000 Explanation If George and Claire elect gift splitting, they can give up to $30,000 to each child (a total of $120,000) and have the gifts be covered by the annual exclusion.

b

Gideon owns the following solely owned assets: A savings account at a bank Commercial real estate that is located in a state other than his state of domicile and that has a sizeable mortgage Gideon's will gives all of his property to his daughter. His will makes no mention of his son. Which of the following actions would have the potential to improve the liquidity of Gideon's estate? Executing a codicil to his will stating the reasons why he is leaving no property to his son by will Executing a codicil to his will placing the commercial property into a testamentary trust for the sole benefit of his daughter Executing a codicil to his will denying his daughter a right of exoneration for the mortgage on the commercial property Establishing a transfer on death designation for the savings account naming his daughter as the beneficiary A) I and II B) I, III, and IV C) I and III D) II and IV Explanation Only option II would not have the potential to increase Gideon's estate liquidity because it is a testamentary trust. Testamentary trusts do not avoid probate. Option I could increase liquidity by decreasing the potential for the son contesting the will. Option III could increase liquidity by eliminating the possibility that the estate would have to pay off the mortgage. Option IV could increase liquidity by el

b

Glen, 58, transferred 300 shares of stock to his son, Drew, in exchange for a joint and survivor private annuity. The annuity provides $2,000 per month to Glen during his lifetime. If Glen predeceases his wife, Nelda, she also will receive $2,000 per month for the rest of her life. What is one estate tax implication of this intrafamily transfer? A) If the amount of annuity payments received at death is less than the fair market value of the shares on the date of transfer, the difference, less one annual exclusion amount, will be included in Glen's estate tax calculation as an adjusted taxable gift. B) When Glen dies, the present value of the future payments to Nelda will be included in his gross estate. C) The value of payments made by Drew to Glen within three years of Glen's death will be includible in Glen's gross estate. D) If Glen has received annuity payments at death equal to the fair market value of the transferred property on the transfer date, his gross estate will not include any amount as a result of this transaction. Explanation Because this is a joint and survivor annuity, there is something left at Glen's death to transfer, and thus an asset on which to impose a transfer tax (i.e., estate tax). The correct value of this asset is the present value of the future payments to Glen's wife, Nelda. Thus, the value of the future payments to Nelda will be in his gross estate. However, because she is his wife and the payments meet the LAME restrictions (Lifetime, Annual, Mandatory, and Exclusive), the same value will be eligible for the marital deduction. Notice that the answer said the value would be in his gross estate. It did not say the value would be in his estate tax base/tentative tax base. The only portion of payments made under the annuity prior to Glen's death that will be includible in his gross estate are the unconsumed portion (if any) of such payments. A private annuity is a sale transaction. As long as the present value of the annuity at the time of transfer equals the fair market value of the shares at the time of transfer, there is no gift even if Glen dies prior to receiving the fair market value of the shares.

b

Grant would like to give a gift to his alma mater, Baylor University. However, he wants to be sure that his wife, Rose, will continue to receive an income stream from the gifted assets for the rest of her life and that the income stream has the potential to keep up with inflation. Grant also wants the option of making future gifts to Baylor University under the same conditions. Due to his distrust of lawyers, he does not want to bear the expense and hassle of setting up this gifting arrangement or having anything to do with its future administration. Given Grant's objectives, which one of the following is the most appropriate charitable giving technique for him to use? A) Charitable remainder unitrust (CRUT) B) Pooled income fund (PIF) C) Outright charitable gift D) Charitable remainder annuity trust (CRAT) Explanation Gifting the property will not work because giving property outright means that the grantor retains no interest in the property—such as an income stream. A CRAT will not work because (1) it is created by the donor; (2) it has a fixed dollar income stream that will not increase with inflation; and (3) future additions to principal cannot be made. A CRUT will not work because a CRUT is created (and usually administered) by the donor. A pooled income fund is created and maintained by the charity and allows additional contributions by a donor. The income payments to Rose from a pooled income fund would have the potential to keep up with inflation—the fund's investments would need to produce income that increased in value at a rate greater than inflation.

b

Gustav and Frieda, husband and wife, want to pay the least amount of estate tax possible on their combined gross estates. They own $8 million in assets as joint tenants with right of survivorship. Gustav owns $7 million of assets in his sole name, and Frieda owns $4 million of assets in her sole name. Make the following assumptions: Gustav will be the first to die. Neither estate will have any adjusted taxable gifts. Neither estate will have any debts or administrative expenses. Unless otherwise directed, all of Gustav's probate estate will go to a noncharitable beneficiary other than Frieda. Assume that no deceased spousal unused exclusion (DSUE) amount will be available to Frieda. If the actions described below were taken, which one of the following transactions will accomplish their objective? A) Gustav should transfer $1 million of his sole assets to Frieda immediately. B) Gustav and Frieda should immediately sever $6 million of the $8 million joint tenancy property and place $3 million in Gustav's sole name, and $3 million in Frieda's sole name. C) Gustav should have his will fund a testamentary bypass trust with $5 million in assets. D) Gustav should have his will fund a testamentary marital trust with $2 million, and the rest of his probate estate should go to a bypass trust. Explanation This option comes closest to equalizing Gustav and Frieda's taxable estates. They are both below the applicable credit amount, and it's good to have Frieda with a taxable estate below $11.58 million. Thus, each spouse would be able to use his or her estate tax applicable credit amount and they would avoid paying any estate taxes out-of-pocket. Although part of Frieda and Gustav's credit amount is wasted in all options, enough is used to prevent payment of tax by equalizing their estates.

b

Herbert Lawson, who lives in a common law state, has a will that gives his entire probate estate in equal shares to his three children. All of Herbert's $16.8 million gross estate is owned in his sole name except for his residence, which is owned as joint tenants with right of survivorship with his spouse. Herbert's interest in this residence is valued at $500,000. Despite having been married for 30 years, Herbert's spouse has no substantial estate of her own. Herbert has made $400,000 in adjusted taxable gifts since 1976. Herbert's spouse is named personal representative (PR) of his estate. Assuming Herbert is survived by his spouse and children, which one of the following is a disadvantage of the probate process for Herbert? A) Herbert's spouse will not be allowed to elect against the will unless she disclaims her right to receive Herbert's interest in the house. B) Herbert's spouse will have the right to elect against the will even though she is named as the PR of the estate. C) The probate process will not allow Herbert's children to disclaim any part of his estate so that Herbert's spouse can receive more of his estate. D) Herbert's estate will have to pay estate tax because the marital deduction will be too small to eliminate all tax. Explanation With no substantial estate of her own, Herbert's spouse will be tempted to elect against the will to get part of the probate estate. While the amount she can get by this election is controlled by state law, and this amount may be reduced by the value of his interest in the residence, most states will allow a surviving spouse (especially in a marriage of this duration) to receive a substantial portion of the probate estate. The probate process does not restrict the right to disclaim. A spouse can elect against the will and retain what is given to him or her by will substitute. The amount given by will substitute is simply deducted from the amount given by making the election. And the spouse receives the difference from the probate estate.

b

Hilger Jantzen established and funded a charitable lead trust with a 10-year term; he designated his children as the remainder beneficiaries. What are the tax implications of this inter vivos intrafamily planning technique? Hilger's charitable gift tax deduction is determined by the present value of the charity's right to receive trust assets at the end of the 10-year term. Hilger is liable for gift tax based on the value of the gift to the children as discounted to the date of the gift. The entire value of the assets gifted to the trust will be removed from Hilger's gross estate only if he outlives the 10-year term. Each year, as the trust pays income to the charity, Hilger receives a charitable income tax deduction for that amount. A) I, II, and III B) II only C) I and IV D) I and III Explanation The answer is II only as it is the only correct statement. Option I describes how to value the charitable gift tax deduction for a remainder interest—in a charitable lead trust, the charity receives the income with the remainder to a noncharitable beneficiary. Therefore, the charitable gift tax deduction is the present value of the income interest. Option III is wrong because Hilger has not retained any interest in or control over the trust that would cause the transfer sections of the code to apply. Finally, option IV is incorrect because it indicates successive annual income tax deductions. The amount of the charitable income tax deduction is based on the present value of the income interest given to the charity in the year the gift is made. However, if all of the deduction cannot be taken in the year of the gift because of Hilger's AGI limitation, the deduction can be carried forward for five more years.

b

Holly Miller is in poor health and would like to make a gift to her nephew, Todd. Holly's main goal is to reduce administrative expenses and taxes on her estate. She also would like to keep both her and Todd's income tax liability as low as possible. Her will leaves everything to Todd when she dies. Todd does quite well financially, but Holly would like to give him something while she is alive. Based on her objectives, which one of the following transfers to Todd would be the most appropriate? A) Transferring a stock portfolio worth $45,000 to Todd; her basis in the portfolio is $5,000 B) Assigning to Todd all incidents of ownership in a life insurance policy that Holly currently owns on a friend's life; the policy names Todd as the primary beneficiary of a $100,000 death benefit; replacement cost of the policy is $44,000 C) Making Todd a joint tenant with right of survivorship in a one-acre tract of land she owns in a neighboring state; the land is valued at $35,000 D) Giving up her retained income interest in an irrevocable trust and accelerating Todd's remainder interest; the trust corpus is valued at $39,000 Explanation While making Todd a joint tenant with right of survivorship in a one-acre tract of land she owns in a neighboring state will reduce Holly's transfer and administrative costs by eliminating the need for ancillary probate, her gross estate will not be reduced because of the contribution rule for property owned in joint tenancy by nonspouses. Contrast this with the life insurance policy which if gifted will result in a minimum $44,000 reduction in her gross estate. The three-year rule would not apply to the transfer of this policy because it is not on Holly's life. The stock portfolio is a highly appreciated asset, which usually should be transferred at death to take advantage of the step-up in income tax basis. If she gifted the portfolio to Todd and he sold it, he would have to pay income tax on a $40,000 gain. However, if it was transferred to Todd at her death and he sold it, because of a step-up in basis there would be no gain, and therefore, Todd would not owe income tax on the sale. Finally, Holly's release of her retained income interest would be subject to the three-year inclusionary rule. Holly has been in poor health. If she dies within three years of the transfer, the full date-of-death fair market value will come back into her estate. Therefore, such a transfer would not likely reduce the size of her gross estate.

b

Holly read a newspaper article promoting lifetime gifting. Until reading this article, Holly had planned to leave all of her property through a testamentary transfer. What advice should you give Holly regarding the advantages of a lifetime gift over a testamentary transfer for a person who will owe estate taxes at death? The donee will usually take the donor's carryover basis. The gifted item is removed from the probate estate. A gift during life is less expensive than a testamentary transfer. A) I and III B) II and III C) I, II, and III D) I only Explanation A gift during life is less expensive than a testamentary transfer because of the annual exclusion and because any potential future appreciation is removed from the future transfer tax. Also, the gifted item is removed from the probate estate. Statement I is false because it is not an advantage. The donee does receive the donor's carryover basis, but it is not an advantage when compared to the stepped-up basis received with a testamentary transfer.

b

Humphrey recently gave his nephew several shares of ABC stock, a listed security. On the date of the gift, this stock closed at $12 per share and traded between a high of $14 per share and a low of $8 per share. In valuing ABC stock for gift tax purposes, what is its appropriate per share value? A) $12 B) $11 C) $8 D) $14 Explanation The fair market value of listed securities for gift tax purposes is the mean between the highest and lowest quoted selling price on the date of the gift. This is calculated as $11 per share: ($14 + $8) ÷ 2.

b

If a trust is a grantor trust, the income from the trust is taxed to A) the trust. B) the grantor. C) the trust beneficiaries. D) the trustee. Explanation If a trust is a grantor trust, the income from the trust is taxable to the grantor. Retaining control over an asset in the trust will cause the donor to have to include the trust earnings in gross earnings as they are taxable. The right to pay life insurance premiums, however, is not an incident of ownership and does not subject the donor to income taxes on the trust earnings.

b

In 2018, Roland established an inter vivos irrevocable trust naming his wife as the sole income beneficiary of the trust. All income must be distributed annually. At his wife's death, the balance of the trust will be in her gross estate. The trust was funded with $3 million in cash and assets. Which one of the following most closely approximates Roland's taxable gift for this transfer? A) $1,985,000 B) $0 C) $2,985,000 D) $3,000,000 Explanation The unlimited marital deduction eliminates any gift tax liability on such a transfer.

b

In which of the following situations is a qualified domestic trust (QDOT) necessary for the donor to receive an unlimited gift tax marital deduction? The donor is a resident alien and the donee spouse is also a resident alien. The donor is a resident U.S. citizen and the donee spouse is a resident alien. The donor is a nonresident U.S. citizen and the donee spouse is also a nonresident U.S. citizen. The donor is a nonresident alien and the donee spouse is a resident U.S. citizen. A) I and III B) II and III C) II and IV D) III and IV Explanation Statements I and IV are false because the donor needs to be a U.S. citizen.

b

Janis owns the Pretty Little Celluloid Shop as a sole proprietor. Janis is now 63 years old and is ready to retire. She has a gross estate estimated at $3.9 million, and the value of the business constitutes $2.45 million of that amount. Janis would like to transfer the business to her daughter and remove all future appreciation of it from her estate. In addition, she would like to receive an income stream from the business for the rest of her life. Which one of the following is the most appropriate form of business transfer for Janis to use to best achieve her objectives? A) A family partnership of the business with her daughter as co-partner B) A pure life private annuity sale of the business to her daughter C) An installment sale of the business to her daughter D) A recapitalization of the business with her daughter as co-shareholder Explanation An installment sale will only provide income for a period of time. The remaining two options don't generate income, but simply bring her daughter in as co-owner.

b

Jean died in a common-law state in 2019 and was survived by her spouse, Loren, and three adult children. Jean's gross estate, all of which was owned solely in her name, was composed of the following assets and date-of-death fair market values: AssetsValuesCommon stock$2,900,000Residence750,000Personal property60,000IRAs1,450,000Hummel figurines175,000Total assets$5,335,000 Jean's only liabilities, together with their date-of-death balance, were as follows: LiabilitiesBalanceMortgage on residence$175,000Car loan8,000Total$183,000 The following is a list of all of the gratuitous transfers that Jean made during her lifetime: 2000:Jean placed the common stock listed above in an irrevocable trust in which she retained the right to a 5% distribution of the trust account revalued annually for 25 years, with the remainder to her children at her death. The date-of-gift fair market value of the stock was $190,000; the value of Jean's retained interest on the date the gift became complete was $90,000.2001:She made a cash gift to her niece of $30,000, which Loren agreed to split.2005:She paid the University of Iowa $11,000 for her youngest child's tuition.2019:She gave her brother $70,000 in cash, which Loren agreed to split. Jean's will, executed in 2000, gave the residence and personal property to Loren and the Hummel figurines to a qualified charity. Jean's three children were designated as equal beneficiaries of her IRAs. Each beneficiary of Jean's estate was to pay any transfer tax due on the portions of the estate received. Her will also stipulated that any real property was to be received without a right of exoneration. Further, her will stipulated that funeral and administrative expenses were to be paid equally by her spouse and three children. These funeral and administrative expenses amounted to $60,000 for Jean's estate. Jean's estate paid state death taxes in the amount of $4,184 and will pay off the car loan. Which of the following amounts most closely approximates Jean's gift tax liability, prior to application of the applicable credit amount, for the 2019 gift? A) $0 B) $6,000 C) $25,300 D) $31,300 Explanation When a question is very long, it can help to skip the background information initially and read the actual question first. In this case, none of the information about Jean's estate matters. The taxable amount of the 2000 transfer was $100,000—the value of the remainder interest ($190,000 - $90,000 she retained). The taxable amount of the 2001 gift was $5,000 ($30,000 2 = $15,000 $10,000). The 2005 transfer is exempt from gift tax. The taxable amount of the 2019 gift was $20,000 ($70,000 2 = $35,000; $35,000 $15,000 = $20,000). Therefore, total taxable gifts equal $125,000 ($100,000 + $5,000 + $20,000). The gift tax on $125,000 is $31,300. Prior taxable gifts equal $105,000 ($100,000 + $5,000). Tax on $105,000 is $25,300. Thus, the gift tax due on the 2019 gift (prior to application of the applicable credit amount) is $6,000 ($31,300 - $25,300).

b

Jeanine and Scott are married. Jeanine gifted $100,000 to their son and $100,000 to their daughter in 2020. Scott also gifted $50,000 to their son in the same year. They made no other gifts during the year. Jeanine and Scott elected to split the gifts on their gift tax returns. What is the total amount of taxable gifts made by Jeanine and Scott, respectively, for 2020? A) Jeanine: $110,000; Scott: $110,000. B) Jeanine: $95,000; Scott: $95,000. C) Jeanine: $170,000; Scott: $20,000. D) Jeanine: $185,000; Scott: $35,000. Explanation Taxable gifts = $95,000 each ($250,000 − $60,000 annual exclusions) ÷ 2.

b

John transferred a $2 million life insurance policy on his own life to an irrevocable trust nine years ago. The policy was paid up and had a gift tax value of $500,000. He paid gift tax of $198,000 out of pocket in the year of the gift. If John dies, how much is included in his gross estate? A) $500,000 B) $0 C) $198,000 D) $2,000,000 Explanation Nothing is included in John's gross estate because the policy on his own life was gifted more than three years prior to death.

b

Juniper and her spouse purchased a home for $1M when they were first married. Juniper contributed 75% of the purchase price. They titled the property as joint title with right of survivorship (JTWROS). If Juniper died today, how much of the residence would be included in the gross estate? A) $1,000,000 B) $500,000 C) $0 D) $750,000 Explanation Although the residence is valued at $1,000,000 at Juniper's death, and she contributed 75% of the purchase price, 50% of the value of the residence is included in Juniper's gross estate because the residence is owned jointly by spouses.

b

Kent, a widower, has made lifetime gifts to his two children in an effort to reduce the size of his gross estate. In 2014, Kent made a $300,000 taxable gift, and in 2016, he made another taxable gift of $100,000. He used his gift tax applicable credit amount to offset any gift tax liability for all gifts. What amount of the gift tax applicable credit, if any, remains available to Kent for gifts he may desire to make in 2020? A) $4,125,800 B) $4,456,000 C) $4,577,800 D) $4,025,800 Explanation To calculate how much of the applicable credit amount remains in 2020, the amount used in prior years must be computed. Because the gifts for 2014 and 2016 are stated to be taxable gifts, nothing further needs to be deducted before computing the tax. Therefore, the tax on $400,000 ($300,000 + $100,000) is $121,800 ($70,800 + 34% of $150,000). Because the maximum gift tax applicable credit amount available in 2019 is $4,577,800 and prior gifts have used $121,800 of this amount, $4,456,000 remains.

b

Linda Plantier wants to establish a trust for her three grandchildren that will accomplish all of the following objectives: Exclude all assets transferred to the trust from her gross estate Protect the trust assets before distribution from the creditors of any beneficiary Keep the trust assets from disqualifying a beneficiary for public assistance benefits such as Medicaid Which of the following trust provisions would NOT help to achieve one or more of these objectives? A) A provision that once the trust is established, Linda cannot change any of the provisions B) A provision granting each beneficiary a Crummey power C) A provision making all distributions from the trust—of both principal and income—subject to the absolute discretion of a corporate trustee D) A provision making the trust irrevocable Explanation A Crummey power, which makes any gift to the trust a gift of a present interest and thus eligible for the annual gift tax exclusion, would not accomplish any of the stated objectives. A creditor would be able to exercise the Crummey power to take possession of the property affected by the Crummey power. The remaining options would help at least Linda achieve at least one of her objectives.

b

Marin, age 43, has a gross estate valued at $6.1 million. She married Steve, age 38, 10 years ago. They have four children, ages 4 through 9. Steve has problems holding on to money. Marin wants to ensure that Steve receives adequate income after her death, if she should die first, but she does not want him to be entitled to principal at any time. She would like all her property to pass to their children in equal shares after Steve's death. Which one of the following transfers is most appropriate for Marin? A) Placing the entire estate in a 2503(c) trust B) Creating a family bypass (B) trust equal to the estate tax applicable exclusion amount, with the remainder in a qualified terminable interest property (QTIP) trust C) Placing the entire estate in a power of appointment (A) trust D) Equalizing the taxable estates using an (A) trust and a QTIP trust Explanation Marin does not want Steve to be entitled to principal at any time, so a power of appointment trust will not work since it is entitled to the marital deduction and gives the spouse a general power of appointment. A 2503(c) trust is incorrect because such a trust is designed for minors, not an adult. While a QTIP trust can be a marital trust if the proper election is made, the corpus of this trust never comes under the unlimited control of the surviving spouse. The corpus of a bypass trust by definition bypasses the surviving spouse's control.

b

Matteo, an unmarried man, owns the following assets: (1) investment accounts worth $6 million; (2) a life insurance policy worth $5 million; (3) a house worth $2.5 million; (4) retirement accounts worth $2 million; and (5) tangible personal property worth $1 million. Matteo created a revocable living trust that names his three children as the beneficiaries, and he funded the trust with all of his assets except the retirement accounts. Which of the following statements regarding the effect of this trust on the potential liquidity of Matteo's estate at his death is CORRECT? A) The trust represents neither a potential cash requirement nor a potential source of liquidity. B) The trust assets will be included in Matteo's gross estate and will be available to meet estate liquidity needs. C) The trust will shield Matteo's estate from an estate tax liability, and therefore, it is sufficient to meet estate liquidity needs. D) The trust does not represent a potential cash requirement, but it does represent a potential source of liquidity. Explanation A funded revocable living trust will allow the assets to transfer outside of probate and provide ready cash for estate needs. Revocable living trusts do not shield assets from estate taxes, as the assets are included in the gross estate.

b

Max McFly owns one-third of the shares of Future Past, Inc., a closely held corporation. Max, together with all remaining shareholders of the corporation, has executed a stock redemption agreement obligating the corporation to purchase all the shares of a deceased shareholder, or of a shareholder who withdraws—voluntarily or involuntarily—from the corporation for any reason. The agreement is funded by a cash value life insurance policy on each shareholder with all premiums paid by the corporation, which is the named beneficiary of each policy. The agreement states that the purchase price—under all circumstances—is to be the fair market value of the shares as established by competent appraisal. Which of the following statements is CORRECT concerning the tax implications of this business transfer technique? A deceased shareholder's estate would not receive a stepped-up basis on the decedent's shares because a value has already been agreed upon in the redemption agreement. The life insurance proceeds would be included in a deceased shareholder's gross estate. Premium payments made by the corporation are not taxable income to any shareholder. Policy proceeds would be received income tax free by the corporation. A) I and IV B) III and IV C) II only D) I, II, and III Explanation Option I is incorrect because a step-up in basis to the value used for estate tax purposes is available for every asset in a decedent's gross estate (that is not IRD) and is not affected by any outside agreement. Option II is incorrect because the assets includible in a decedent's gross estate are fixed as of the date of death, and the insurance proceeds are received after the date of death as the result of a transaction between the deceased shareholder's estate and the corporation. Also, the deceased shareholder has no incidents of ownership in the policy.

b

Rick placed his portfolio of income-producing stock valued at $200,000 into an irrevocable trust. The trust provides that the trustee is to pay to Rick 7% of the initial value of the trust annually for a period of 12 years. After the 12-year term, the trustee is directed to pay the remaining assets in the trust to Rick's daughter. Which of the following are CORRECT statements regarding the income, gift, or estate tax implications of this trust arrangement? Rick will owe a gift tax based on the present value of the remainder interest given to his daughter. Taxation of the income earned by this trust will be affected by the grantor trust rules. If Rick dies during the 12-year term of the trust, the trust assets will be included in his gross estate. The right that Rick has retained in this trust is considered to be "qualified" for purposes of the Chapter 14 rules. A) II and IV B) I, II, III, and IV C) I and IV D) I and III Explanation This is an example of a grantor-retained annuity trust (GRAT). The interest retained by Rick is deemed to be qualified for Chapter 14 purposes because it is an annuity amount. Payment of the 7% is not dependent upon the trust assets earning any income. The 7% must be paid regardless of trust earnings. The retained interest is qualified, so the normal valuation rules apply, and Rick will have to pay gift tax only on the present value of the remainder interest given to his daughter and not the entire fair market value of the trust assets. Because trust income may be given to the grantor (Rick), the grantor trust rules apply. If Rick dies during the trust term, the assets will be included in his gross estate by virtue of Code Section 2036 (one of the transfer sections) because he retained the right to receive trust income for a period that did not in fact end before his death.

b

Robert gifted a duplex to his brother Harold, who is terminally ill. Harold is not married and has no children. Robert purchased the duplex 20 years ago for $175,000. Today, the property is worth $450,000. Harold's will leaves his entire estate to Robert upon his death. This means that the duplex may ultimately be passed back to Robert. This type of transfer is called A) a split gift. B) a reverse gift. C) a net gift. D) a bargain sale. Explanation This type of transfer is called a reverse gift. A net gift is a situation in which the donee pays the gift taxes rather than the donor. A bargain sale involves selling appreciated property to a donor at below the market value. A split gift is one that is split between donor spouses, effectively doubling the annual exclusion for lifetime gifts. For a reverse gift to be effective, the donee must have the property for more than a year. If the donee passes away within a year of the gift, the property reverts back to the donor, but it does not get a stepped-up basis.

b

Robert is the sole income beneficiary of a charitable remainder unitrust (CRUT) established by his recently deceased wife in her will. A qualified public charity will receive the trust remainder at Robert's death. Which one of the following is a CORRECT statement regarding the effect of this trust on the potential liquidity of Robert's estate at his death? A) The trust does not represent a potential cash requirement, but does represent a potential source of liquidity because of the charitable deduction. B) The trust represents neither a potential cash requirement nor a potential source of liquidity for Robert's estate. C) The trust represents both a potential cash requirement and a potential source of liquidity for Robert's estate. D) The trust represents a potential cash requirement since the estate of Robert's spouse did not pay estate tax at the time of funding.

b

Roger has made the following gratuitous lifetime transfers: $30,000 in 2000 to his mother and father; $30,000 to each of his three children in 2001 (these amounts were placed in three separate Section 2503(c) minor's trusts); and $90,000 in 2011 to a revocable trust that gives the corporate trustee the discretion to make disbursements of income or principal for the health, education, maintenance, and support of Roger's brother. Through the end of 2011, the trustee made disbursements of $14,000 from the trust to Roger's brother. Roger's spouse passed away in 1999. What amount of gift tax applicable credit does Roger have available in 2020 for subsequent lifetime transfers after reporting these transfers for gift tax purposes? A) $330,440 B) $4,561,940 C) $984,140 D) $4,283,800 Explanation The taxable gifts were as follows: 2000: $30,000 - [2 × $10,000 for annual exclusions (AE)] = $10,000 2001: $30,000 × 3 - [3 × $10,000 (AE)] = $60,000 2011: $14,000 - $13,000 (AE) = $1,000 (only the amount actually disbursed is a completed gift) Therefore, Roger has made total cumulative taxable gifts of $71,000, which has incurred cumulative gift taxes in the amount of $15,860, and therefore he has used $15,860 of his gift tax applicable credit amount, leaving $4,561,940 ($4,577,800 - $15,860) of gift tax applicable credit amount available for lifetime gifts in 2020.

b

Sam Cahill died this year. His revocable living trust provided that after his death, the trustee was given discretion to distribute trust income among his spouse, children, and grandchildren for their health, education, maintenance, and support. The corpus and any undistributed income are to be distributed in equal shares to his grandchildren when Sam's youngest child reaches age 45. Sam was survived by his spouse, three children, and eight grandchildren. Which of the following are CORRECT statements regarding the application of the generation-skipping transfer tax (GSTT) to this trust? Sam's trust is an example of an indirect skip. If the trustee distributes any income to Sam's spouse, for GSTT purposes a taxable distribution will have occurred. No generation-skipping transfer from this trust can take place until the youngest of Sam's children reaches age 45. When Sam's grandchildren receive the corpus and undistributed income, a taxable termination will occur.

b

Sandra dies owning a family farm. The value of the acreage as farmland is $2,500 per acre. Many adjoining farms have been converted to tract housing and have sold for as much as $10,000 an acre. Sandra's will leaves the farm to her daughter, who intends to use it as farmland for the rest of her life and then pass it to her children. Which of the following postmortem estate planning techniques will be most useful to Sandra's estate? A) Election against the will B) Special use valuation (Section 2032A) C) Alternate valuation date (AVD) D) QTIP election Explanation Special use valuation under Section 2032A will be most useful to Sandra's estate because it permits the farm to be valued at its current use as farmland ($2,500 per acre) rather than at its highest and best use as tract housing ($10,000 per acre). The maximum reduction under Section 2032A is a little over $1 million (indexed). In 2020, the maximum reduction is $1.18 million which is the latest data as of the time of this writing.

b

Shawn has died, and his will directs that all of his probate estate be paid to the trustee of a testamentary trust established in his will. The trust contains the following provisions: The sole income beneficiary of the trust is Shawn's surviving spouse The remainder beneficiaries are Shawn's children by a prior marriage Income is to be paid on an annual and mandatory basis A bank trust department is appointed trustee The trust term ends 10 years from Shawn's death Is Shawn's estate eligible to make a qualified terminable interest property (QTIP) election? A) No, because Shawn's spouse is not given a general power of appointment over the trust B) No, because Shawn's spouse does not have a qualifying income interest C) No, because the trust assets will not be paid to the estate of Shawn's spouse at her death D) Yes, because all necessary prerequisites have been met Explanation The income interest of Shawn's spouse lasts only for 10 years, and not for her lifetime, which is necessary for a qualifying income interest, which is required for a QTIP trust. Payment of trust assets to the spouse's estate and giving the spouse a general power of appointment would require that the trust assets be included in the spouse's gross estate, but still would not entitle Shawn's estate to make the QTIP election.

b

Tasha and her mother, Marleen, meet with a financial advisor to get a better understanding of Marleen's estate and financial planning. Marleen has recently updated her will but Tasha fears there may be gaps in her mother's planning. If Marleen is in poor health, which of the following estate planning devices should Marleen include in her overall estate planning? Springing durable power of attorney for health care Advance medical directive Special needs trust Qualified domestic trust (QDOT) A) I, III, and IV B) I and II C) I only D) II, III, and IV Explanation Because Marleen is in poor health, there may soon be a need for someone else to step in and manage both her health care and her assets, making springing powers of attorney for health care and property desirable. She should also execute an advance medical directive (living will) so her physicians are informed of her wishes regarding end-of-life care. A qualified domestic trust (QDOT) is not appropriate as Marleen does not have a nonresident alien spouse.

b

Tess Thomas lives in a common law state. She "wrote out" a will in her own handwriting and signed it. In the will, she left everything to her sister, Kate, because she and her spouse separated last month. Since executing the will, Tess has become concerned about its adequacy. You should refer her to an attorney after informing her that A) this is a nuncupative will, which is invalid and allows all her property to avoid probate. B) if she has not met the state's will requirements, her will is unenforceable, and probate property interests will pass according to the intestate succession statute. C) her will is valid and will result in all her solely owned property going to her sister. D) the fact that she drafted her own will creates a presumption that she was not of sound mind when it was drafted, which causes the will to be invalid. Explanation If Tess has not met the state's will requirements, her will is unenforceable, and probate property interests will pass according to the intestate succession statute. A handwritten and signed will described is a holographic will (a nuncupative will is an oral will). If the will is invalid, property not held in will substitute form at death would still be subject to probate under state intestacy laws. Even if the will were valid, the property would still be subject to probate; only property governed by a will substitute (such as joint tenancy with right of survivorship, beneficiary designation, etc.) is not subject to probate. Several states recognize holographic wills. Generally, in a common law state, one spouse cannot legally disinherit the other spouse, because the surviving spouse has a right to elect against the will to receive his spousal share.

b

Walter died with a gross estate of $18 million, which included an interest in a closely held business. His total estate tax is $2 million, and the estate tax attributable to the closely held business interest is $800,000. Assuming Walter's estate qualifies for the installment payment of estate taxes, what is the maximum amount of federal estate tax that may be deferred? A) $1,200,000 B) $800,000 C) $400,000 D) $2,000,000 Explanation The amount of federal estate tax that may be deferred is limited to the tax attributable to the value of the closely held business or businesses.

b

Which of the following are characteristics of a revocable living trust that transfers all of a person's assets at death? It allows trust assets to receive a stepped-up basis at the grantor's death Trust assets will be included in the grantor's gross estate It ensures protection of trust assets from the claims of the grantor's creditors during life Trust assets will be included in the grantor's probate estate A) II and III B) I and II C) I and IV D) III and IV Explanation Assets in the revocable trust will receive a stepped-up basis at death (option I) as long as they are included in the decedent's gross estate (option II), as they will be under the transfer sections. The trust assets will not be included in the grantor's probate estate (option IV) as the trust acts as a will substitute. Because the trust is revocable, the grantor's creditors can still get at trust assets during life (option III).

b

Which of the following are characteristics of the probate process? It provides for the orderly distribution of property that passes by will or intestate succession to the ultimate beneficiary. It usually provides a longer time period for the filing of claims than if assets were to pass outside of probate. It provides for systematic administration of the decedent's estate. It provides for administration of all of the decedent's gross estate.

b

Which of the following are effective method(s) of limiting or avoiding federal estate taxes? Use the annual gift tax exclusion Create an irrevocable life insurance trust Use qualified transfers to pay tuition and medical expenses directly to the provider Use the unlimited marital deduction A) II and III B) I, II, III, and IV C) I, III, and IV D) I, II, and IV Explanation All of these items will reduce the gross estate or the taxable estate.

b

Which of the following are essential in establishing and defining the client & planner relationship? A) Determine how the planner is compensated. B) All of the above. C) Understanding the personal and financial circumstances of the client. D) Determine what the planner is to do or not do. Explanation All would be examples of activity useful in establishing the client and planner relationship.

b

Which of the following are fiduciary relationships created by law to enable one person to manage the property of another? Conservatorships Guardianships A) Both I and II B) I only C) II only D) Neither I nor II Explanation Statement II is incorrect because a guardianship is a fiduciary relationship that enables one person to manage the personal care and well-being (but not the property) of another. Think of prison guards. You guard a person. You conserve money or property.

b

Which of the following are prerequisites for application of the generation-skipping transfer tax (GSTT)? A gratuitous completed transfer Transferee deemed to be two or more generations younger than the transferor Transfer qualifies as a direct skip transfer No exceptions or exemptions from the normal rules apply A) I and II B) I, II, and IV C) II and III D) III and IV Explanation These are the three prerequisites for application of the GSTT. Transfers can qualify as either a direct or indirect skip and be liable for the GSTT. The times at which the GSTT must be reported and will be due depends in the first instance on whether the GST is a direct or indirect skip.

b

Which of the following are property interests that must go through the probate process? A car that passes to the decedent's spouse by the state's intestate succession statute. A government savings bond that is titled in the name of the decedent payable on death to the decedent's daughter. Life insurance proceeds payable to the decedent's estate. Assets in a revocable trust that pass to the decedent's children at the decedent's death. A) II and IV B) I and III C) II, III, and IV D) I, III, and IV Explanation Assets that pass by the laws of intestacy (option I) are subject to probate. Although life insurance proceeds that are payable to a named beneficiary pass by will substitute, when they are payable to the decedent's estate (option III) they become subject to the probate process. The government savings bond (option II) passes by will substitute because of the beneficiary designation. Assets in a revocable trust (option IV) also pass by will substitute, because the trust names remainder beneficiaries.

b

Which of the following are requirements that must generally be met for property to qualify for the estate tax marital deduction? The property must be included in the decedent's gross estate. The property must pass to the decedent's surviving spouse. The value of the property must not exceed $1 million. The property must not be a terminable interest or must qualify under one of the exceptions to the terminable interest rule. A) II and IV B) I, II, and IV C) I, II, III, and IV D) I and III Explanation Statement III is incorrect because the estate tax marital deduction is unlimited in amount; if the other requirements are met, there is no limit on the amount of the marital deduction.

b

Which of the following characteristics is common to both the federal gift tax and the federal estate tax? A) The property receives a stepped-up income tax basis in the hands of the recipient of the transfer. B) The ability to shield qualifying transfers to a qualified charity from the tax C) The ability of one spouse to assume half of the transfers made by the other spouse D) The tax-exclusive nature of the tax Explanation Both the gift tax and estate tax allow a charitable deduction for qualifying transfers. The gift tax is tax exclusive, while the estate tax is tax inclusive. Only lifetime transfers (gifts) can be split by spouses. A stepped-up income tax basis is awarded only for property subjected to the estate tax.

b

Which of the following charitable remainder trusts can be revoked by the grantor after they are established? Charitable remainder annuity trusts (CRATs) Charitable remainder unitrusts (CRUTs) A) Both I and II B) Neither I nor II C) II only D) I only Explanation Neither is correct. For a trust to qualify as a CRAT or a CRUT, the trust must be irrevocable upon inception.

b

Which of the following charitable trusts may NOT invest in tax-exempt securities? Pooled income fund (PIF) Charitable remainder annuity trust (CRAT) Charitable remainder unitrust (CRUT) A) I, II, and III B) I only C) II and III D) III only Explanation The pooled income fund is the only option that cannot invest in tax-exempt securities. CRATs and CRUTs are allowed to invest in tax-exempt securities.

b

Which of the following correctly explains why a beneficiary who received nonprobate assets by will substitute may agree to the use of all or part of the assets to meet estate liquidity needs? A) To avoid an IRS lien against the nonprobate property that the beneficiary received from the decedent B) All of these C) To avoid the liquidation of probate assets to meet cash needs D) To avoid a lawsuit by the personal representative for a prorated portion of the taxes Explanation All three answers correctly explain why a beneficiary who received nonprobate assets by will substitute may agree to the use of all or part of the assets to meet estate liquidity needs. First, without the aid of such nonprobate beneficiaries, the personal representative (PR) may be forced to liquidate probate assets to meet cash needs. If the nonprobate beneficiary also is to receive a bequest, it may be considerably reduced or eliminated because in a forced liquidation of estate assets, the estate is likely to have to sell assets far below their fair market value, and administrative expenses will be greatly increased. Second, if taxes are equitably apportioned, the PR usually has authority to withhold the probate share of a beneficiary who has received a nonprobate asset that generates estate tax or to sue such beneficiary for her share of the taxes. Finally, if property received by a nonprobate beneficiary was included in the decedent's gross estate, and estate taxes are not paid, the IRS has the right to pursue nonprobate beneficiaries to collect such tax. In some cases, this might involve the filing of a lien against the property that the beneficiary received from the decedent.

b

Which of the following correctly identify a premortem technique that can be used to reduce the cash needs of an estate? Retitling property as joint tenants with right of survivorship (JTWROS). Executing a will that includes a self-proving clause and meets all legal formalities required by state law. Investing in real estate, such as rental properties, and retaining closely held business interests. Purchasing real property in multiple states for investment purposes. A) II and IV B) I and II C) I, II, III, and IV D) I, III, and IV Explanation Statement III is false because real estate and closely held businesses can only help increase cash available, but won't reduce cash needs. These will increase cash needs if liquid assets are used to acquire these assets. Statement IV is false because this will require ancillary probate which will increase cash needs, not decrease cash needs.

b

Which of the following describe(s) a reverse QTIP election? A special election made by an executor to treat qualified terminable interest property as if the QTIP election had not been made for generation-skipping transfer tax purposes. An election that allows better utilization of a decedent's GSTT exemption. A) I only B) Both I and II C) II only D) Neither I nor II Explanation Both of these statements describe reverse QTIP elections.

b

Which of the following estate planning techniques is useful for nonspouses in nontraditional relationships? Convert individually owned property into joint tenancy with right of survivorship (JTWROS) Establish a revocable living trust naming the other partner as beneficiary at death Rely on intestate succession laws to pass property to the other partner Name the other partner as beneficiary of qualified retirement plans and IRAs A) III only B) I, II, and IV C) II and IV D) I, II, III, and IV Explanation Statement III is incorrect because intestate succession laws generally do not recognize the rights of unmarried domestic partners to inherit property from one another.

b

Which of the following estates could use the alternate valuation date to value estate assets for federal estate tax purposes? An estate with an estate tax base of $1 million An estate valued at $20 million in which all assets pass to the surviving spouse and qualify for the marital deduction A) II only B) Neither I nor II C) I only D) Both I and II Explanation An estate cannot use the alternative valuation date when the estate tax base is less than the estate tax exemption equivalent ($11,580,000 in 2020) or when all assets pass to the surviving spouse and qualify for the marital deduction.

b

Which of the following is an effective way for a client to remove the value of an asset from her estate? A) Placing the asset in her irrevocable trust in which she retains an income interest B) Gifting the asset to another individual C) Placing the asset in her revocable trust D) Placing the asset in a grantor trust Explanation The asset is no longer in the possession, and therefore, no longer in the gross estate, of the donor.

b

Which of the following marital trusts restrict who may serve as trustee? Qualified terminable interest property trusts (QTIPs) Qualified domestic trusts (QDOTs) A) Both I and II B) II only C) I only D) Neither I nor II Explanation Statement I is incorrect. QDOTs and QTIPs include similar provisions, but only a QDOT restricts who may serve as the trustee.

b

Which of the following statements concerning the calculation of the generation-skipping transfer tax (GSTT) is CORRECT? The GSTT is determined by multiplying the taxable amount by the applicable GSTT rate. The applicable GSTT rate is the highest federal estate and gift tax rate multiplied by an inclusion ratio. A) II only B) Both I and II C) I only D) Neither I nor II Explanation Both of these statements are correct.

b

Which of the following statements correctly describes the gross-up rule? A) The value of any gifts made within three years of the date of death must be included in the gross estate. B) Gift taxes paid on gifts made within three years of the date of death must be included in the gross estate. C) Gift taxes paid on any gifts made during the decedent's lifetime must be included in the gross estate. D) Gift taxes paid on gifts made within five years of the decedent's death must be included in the gross estate. Explanation The gross-up rule states that gift taxes paid on gifts made within three years of the decedent's death must be included in the gross estate. This is an example of being taxed on a tax. The deceased is estate taxed on the gift taxes paid out of pocket within three years of death (not the value of the gifts on which the gift taxes were paid).

b

Which of the following statements correctly identify advantages or disadvantages of a living will? The provisions of a living will may apply only when the maker is in certain medical conditions. A living will must be re-executed periodically so that the medical care provider knows that it represents the maker's most current desires. A living will cannot be used in conjunction with a medical durable power of attorney for health care (DPOAHC) or a do not resuscitate (DNR) order. A living will becomes invalid when the maker becomes incompetent unless it has certain language. A) III only B) I only C) II, III, and IV D) I and IV Explanation Many states restrict the effectiveness of living wills to when the maker is in a terminal or incurable condition. Although periodic execution of a new living will (option II) might have some benefit, it is generally not required. A living will can and often is used in conjunction with DPOAHC and DNR orders (option III). The main purpose of a living will is to let it speak for a patient when the patient is incompetent (option IV).

b

Which of the following statements correctly identify an advantage or a disadvantage of a durable power of attorney for health care (DPOAHC)? A disadvantage is that the agent's authority can be exercised only when the principal is in a terminal or chronic condition. An advantage is that the authority of the agent can be revoked as long as the principal is competent. A disadvantage is that the agent's authority can be exercised even when the principal is competent and makes a contrary decision. An advantage is that the agent can be given authority to do more than simply make decisions regarding medical treatment. A) I, II, and III B) II and IV C) III and IV D) I and II Explanation The agent under a DPOAHC can be authorized to do such things as file lawsuits or interpret the principal's living will. Option I is incorrect, as the advantage of a DPOAHC over a living will is that the agent's authority can be used in any situation in which the principal is incompetent to make medical decisions. Option III is incorrect because the principal's contrary decision in this situation would act as an implied revocation of the agent's authority.

b

Which of the following statements is CORRECT regarding a CFP® certificant's role in defining a client's financial goals, needs, and priorities? The role of the planner is to facilitate the goal-setting process. IThe role of the planner is to assist clients in recognizing the implications of unrealistic goals and objectives. The role of the planner is to make sure a client's goals and objectives are consistent with the client's values and attitudes. The role of the planner in this process will involve exploring a client's expectations and time horizons. A) III and IV B) I, II, III, and IV C) I and II D) II, III, and IV Explanation All of these statements are correct.

b

Which of the following statements regarding a QTIP election is CORRECT? If the decedent's executor makes a QTIP election, the percentage of the trust property that is the subject of the election is included in the surviving spouse's gross estate when the surviving spouse dies. In determining whether to make a QTIP election, the executor should determine the best overall estate tax result for both the decedent's and the surviving spouse's gross estate. A) Neither I nor II B) Both I and II C) II only D) I only Explanation Both statements are correct.

b

Which of the following statements regarding a payable-on-death (POD) account is(are) CORRECT? A payable on death account is a bank or savings account controlled by the depositor so long as living, but with a provision that the account is payable to another if still open when the depositor dies. Payable-on-death accounts are included in the depositor's probate estate. Payable-on-death accounts are considered a will substitute. Payable-on-death accounts could create guardian problems if paid to a minor beneficiary. A) I and III B) I, III, and IV C) III only D) I, II, and IV Explanation Payable-on-death (POD) accounts are not included in the depositor's probate estate because they pass to the beneficiary by contract. They are considered a will substitute and can create guardian problems if paid to a minor beneficiary.

b

Which of the following statements regarding a taxable termination is CORRECT? A taxable termination cannot occur as long as at least one nonskip beneficiary has an interest in the trust property. If a taxable termination occurs, the skip person is responsible for paying any generation-skipping transfer tax (GSTT) that may be due. A taxable termination occurs when an interest in a trust is terminated because of death or lapse of time, resulting in a skip beneficiary holding interests in the trust. A) II and III B) I and III C) I only D) I, II, and III Explanation Statement II is incorrect; if a taxable termination occurs, the trustee is responsible for paying any GSTT that may be due.

b

Which of the following statements regarding adjusted taxable gifts (ATGs) is CORRECT? ATGs are taxable gifts made by the decedent after December 31, 1976. ATGs are included in the estate tax calculation at their date of death value. ATGs are included in the decedent's gross estate. A) II only B) I only C) I and II D) I, II, and III Explanation Statements II and III are incorrect because ATGs are added to the taxable estate (not the gross estate) at their date of gift value. All appreciation after the gift is made escapes transfer taxation for the original giver.

b

Which of the following statements regarding charitable gift annuities (CGAs) is CORRECT? A) A CGA is usually secured and presents no risk to the donor-annuitant. B) Either cash or appreciated property may be donated to charity in a CGA. C) A CGA is a contract between a donor and a qualified charity in which the donor pays an annuity to the charity in exchange for cash. D) The donor of a CGA is eligible for an income tax charitable deduction equal to the fair market value (FMV) of the donated property. Explanation A charitable gift annuity (CGA) is a contract between a donor and a qualified charity in which the donor transfers cash or appreciated property to the charity in exchange for an annuity. The donor is eligible for an income tax charitable deduction equal to the FMV of the donated property minus the present value of the annuity to be received. A CGA is unsecured and presents some risk to the donor.

b

Which of the following statements regarding private foundations are CORRECT? The private foundation must distribute at least 5% of its assets annually for charitable purposes. Private foundations may be administratively expensive to operate because they are subject to strict IRS reporting requirements. Nonoperating private foundations engage in charitable activity directly. A) I, II, and III B) I and II C) III only D) II only Explanation Nonoperating private foundations do not engage in any charitable activity directly but generally distribute funds for charitable purposes.

b

Which of the following statements regarding the annual exclusion for purposes of generation-skipping transfer tax (GSTT) is(are) CORRECT? The annual exclusion amount is $15,000 for 2020. The annual exclusion is allowed for lifetime direct skips. The annual exclusion is allowed for testamentary direct skips. A) I, II, and III B) I and II C) I only D) II and III Explanation Statement III is incorrect because the annual exclusion is not allowed for testamentary direct skips.

b

Which of the following statements regarding the basis of inherited assets is CORRECT? When spouses own property as community property, both halves of the community property receive a stepped-up basis when the first spouse dies. When spouses own property as joint tenants with right of survivorship (JTWROS), 50% of the property receives a stepped-up basis when the first spouse dies. Inherited property that is income in respect of a decedent (IRD) receives a stepped-up basis when the decedent dies. A) II only B) I and II C) I only D) I, II, and III Explanation Statement III is incorrect. Assets that are considered IRD do not receive a stepped-up basis at the decedent's death.

b

Which of the following statements regarding the consequences of holding property jointly is CORRECT? A) A tenancy in common is treated the same as a joint tenancy with the right of survivorship when one owner dies. B) When spouses are joint tenants with a right of survivorship, 50% of the value of the property will be included in the gross estate of the first spouse to die. C) The federal estate tax treatment of jointly held property is the same for spouses and nonspouses. D) Joint tenancy with right of survivorship can exist between spouses only. Explanation A tenancy in common is not treated the same way as a joint tenancy with right of survivorship because a tenancy in common does not provide a right of survivorship. Joint tenancy is not limited to spouses, and the treatment of joint tenancy for estate tax purposes is different for spouses and nonspouses. Spouses are always defined as having each contributed half towards the purchase of the property. For estate taxes, nonspouse decedents are initially assumed to have contributed 100%, and thus will be estate taxed on 100% of the property unless the other joint tenancy with right of survivorship (JTWROS) can be shown as having made an actual contribution to the purchase of the JTWROS property.

b

Which of the following statements regarding the group term life insurance provided by an employer are CORRECT? The employer can deduct the premiums paid as a necessary and reasonable business expense. No part of the premiums that the employer pays will be considered income to any of the employees. The death benefit of the policy will be included in the employee's gross estate. The beneficiary of the death benefit will have taxable income to the extent that the death benefit exceeds the value of premiums paid by the employer. A) I and IV B) I and III C) II and III D) III and IV Explanation Statement II is false because group life premiums for amounts over $50,000 are considered income to the employee. Statement IV is false because death benefits would be received income-tax-free.

b

Which one of the following is a CORRECT statement regarding the characteristics of a salary increase or selective pension plan using life insurance under IRC Section 162? A) The employee pays part of the policy premium. B) The premium payments are taxable income to the employee. C) The insured employee has no incidents of ownership in the policy. D) Part of the death benefit will be paid to the employer. Explanation The premium payments are taxable because this insurance is considered to be additional compensation. The employee holds all incidents of ownership in the policy, but the employer pays the entire premium. All of the death benefit will go to the policy beneficiary designated by the employee.

b

Which one of the following is the least important consideration in selecting a competent personal representative (PR)? A) Having knowledge and competency in dealing with financial matters B) Being a blood relative of the testator C) Being willing to accept the fiduciary responsibility D) Recognizing when to retain competent professional assistance Explanation Picking a relative as an executor is the least important consideration. Choosing an executor requires a careful examination of a person's traits. Honesty, financial competency, common sense, and a willingness to accept the appointment are far more important considerations than whether the person is related to the testator, is a beneficiary, or will serve without compensation. The goal is thorough, effective, and efficient administration of the deceased's estate.

b

Which one of the following statements is CORRECT about the nontax characteristics of a reverse gift? A) The donee does not have the right to sell the gifted property. B) The purpose of the gift is that the donor will get the gift back with a stepped-up basis. C) A gift given anytime within three years of the donee's death will achieve the objective of the donor. D) Property with an income tax basis close to its fair market value is often the subject of the gift. Explanation The gift is given to a donee who will most likely predecease the donor, with the expectation that the donee will bequeath it to the donor and that the property will have received a stepped-up basis at the donee's death, thereby wiping out any gain. Property with an income tax basis close to its fair market value is incorrect because property that has a large amount of gain is selected for this technique. More than one year must pass between the time of the gift and the death of the donee to achieve a step-up in basis. To get the desired step-up in basis, the donee must have sufficient dominion and control so that the donee could sell the property if she chose, rather than bequeath it back to the donor.

b

Which one of the following statements regarding Henry, who recently married for the first time, is CORRECT? A) In a community property state, any property Henry owns at death will go to his spouse by right of survivorship. B) In a community property state, Henry's earnings from his job subsequent to the date of his marriage will be considered community property. C) In a community property state, Henry's spouse is deemed to have a vested 50% interest in all of the property Henry owned at the time of the marriage. D) In a community property state, Henry's earnings from his job both before and after the date of his marriage will be considered community property. Explanation Income earned after marriage is considered community property.

b

Within three years prior to her death, Himari established an irrevocable trust with her children as trust beneficiaries. She transferred ownership of a vacation cabin into it, and in the deed conveying the property to the trust, Himari reserved the right to use the vacation cabin when it is not occupied. Which of the following statements regarding whether the trust property would be included in Himari's gross estate is CORRECT? A) It is included because the transfer was made within three years prior to death. B) It is included because she retained the right to use the property. C) It is excluded because her use is dependent upon the implied consent of the children. D) It is excluded because she retained no administrative control over the trust. Explanation Retention of the right to use property makes the asset includible in Himari's gross estate no matter how long ago she established the arrangement. The limitation "when it is not occupied" is irrelevant. The three-year rule has no application to this situation.

b

Your client has an estate valued at $3 million. Two months ago, his wife died. He and his deceased wife did not have any children together, but she had two children from a prior marriage. His will, drafted in 2007, leaves everything to his wife. Nocontingent beneficiary is named in the will, and it does not contain a residuary clause. Included in the client's estate are real estate holdings in three other states. He wants to retain lifetime ownership of these properties because of the income they provide him. He would like the real estate holdings to pass to his wife's children in equal shares upon his death. He would like the remainder of his estate to go to his brother. Which of the following are serious estate planning pitfalls that can be avoided if your client amends his will to carry out his objectives? Having the estate pass under the laws of intestacy Having the estate assets distributed through probate Having the estate pay any estate tax Having part of the estate pass to unintended beneficiaries A) III and IV B) I and IV C) I and II D) II and III Explanation Statement II is false because amending a will has no effect on whether probate can be avoided. Wills are probated. Statement III is false because amending a will has no effect on the estate tax calculation.

b

Which of the following statements regarding the federal gift tax return IRS Form 709 is CORRECT? For a calendar-year taxpayer, an extension of time for filing IRS Form 1040 also extends the time for filing IRS Form 709. George gives $5,000 of separate property to his son. If Mary, George's wife, elects to split the gift with George, they must file a gift tax return. George and Mary give $20,000 of community property to their son. No gift tax return need be filed. An extension of time for filing the gift tax return does not extend the time for payment of the gift tax. A) III and IV B) I, II, III, and IV C) I and II D) I, II, and III Explanation All of the statements are correct.

b (all)

A decedent is a U.S. citizen who owned a farm immediately prior to the date of his death. Which of the following conditions must be met before special use valuation is allowed for estate tax purposes? The value of the farm must constitute at least 75% of the decedent's gross estate. The farm must pass to a qualified heir. The decedent or a member of his family must have materially participated in the operation of the farm for at least five out of the eight years prior to his death. The farm must continue to be used as a qualified use for at least 10 years after the decedent's death. A) I and II B) I and III C) II, III, and IV D) III and IV Explanation Statement I is incorrect. The value of the farm must constitute at least 50% of the decedent's gross estate after certain adjustments for mortgages and liens.

c

Alan created and funded an irrevocable trust with $150,000 for the benefit of his two minor children with income to be accumulated for five years, at which time the trust will terminate and all the income and corpus are to be distributed equally between the two children. Which of the following is a CORRECT statement about the impact of this lifetime transfer on any subsequent lifetime transfers Alan might make? A) Any annual exclusions that Alan applies to the transfer creating the trust will decrease the annual exclusions available for transfers to the same children in the future. B) Alan could not take any annual exclusions in the year he created the trust, but he will be able to do so at the time the trust is distributed to the children. C) Any subsequent taxable lifetime transfers will be taxed at a higher rate. D) This transfer will have no impact on any subsequent transfers. Explanation The taxable amount of this transfer is $150,000 because no annual exclusions can be taken (there are no present interest gifts because the income will be accumulated for five years); $150,000 is at the top of a bracket on the unified rate chart. Therefore, the next taxable transfer will be taxed at a higher rate. There is no gift at the time of distribution to which to apply an annual exclusion. No annual exclusions can be taken at the creation of the trust. Annual exclusions are available each year.

c

Alex and Megan need to have their wills updated; however, they would like to avoid the expense of writing entirely new wills. Which one of the following techniques would best meet their needs? A) File qualified disclaimers B) Elect against their wills C) Execute codicils D) File will contests Explanation A codicil is an instrument used to modify or amend an existing will. Executing a codicil is less expensive and cumbersome than writing an entire new will.

c

All of the following statements regarding charitable gift annuities (CGAs) are correct except A) the donor of a CGA is eligible for an immediate income tax deduction for the full amount transferred. B) the annuity payable under a CGA may be for one or two lives using a joint and survivor payment. C) a CGA presents no risk to the donor-annuitant because the agreement is secured. D) with a CGA, the donor transfers cash or appreciated property to a charity in return for the charity's promise to pay an annuity to the donor or other designated annuitant. Explanation A charitable gift annuity presents some risk to the donor-annuitant because it is unsecured. The donor of a gift tax annuity receives an income tax charitable deduction for the difference between the FMV of the assets transferred and the present value of the annuity.

c

An older couple, who both have children from prior marriages, is contemplating marriage. They disagree about how to handle their finances after their marriage, and their children are concerned about receiving their inheritances. Which of the following strategies would you recommend to address these issues? A) Have the couple title all of their assets as JTWROS after they get married B) Have each spouse draft a will disinheriting the other C) Have the couple execute a marital (prenuptial) property agreement D) Have the families enter into a family settlement agreement Explanation Drafting a marital property (prenuptial) agreement is the best solution here because it will allow the couple to agree on how their property will be divided while they are living and also agree on the relinquishment of marital property rights when one of them dies. Titling their property as JTWROS will not assure that their children receive their inheritances because the surviving spouse will gain full ownership of all assets. Drafting wills which disinherit each other is ineffective because the surviving spouse might file a will contest or, in many states, make an election against the will. Finally, a family property agreement may be ineffective because it is not entered into until after death, and the heirs may not be able to reach an agreement.

c

Archer's current estate plan disposes of his entire estate as follows: $1,500,000 outright to his wife, Doris $11,800,000 in a family bypass trust for his children Archer has not made any lifetime taxable gifts. He estimated the estate tax on the current plan to be $160,000 if he were to die in 2020. He is considering a suggestion to restructure his estate plan as follows: $90,000 outright to his wife, Doris $11,580,000 in a family bypass trust for his children $1,630,000 in a qualified terminable interest property (QTIP) trust payable to his children at his wife's death What amount of tax, if any, will the proposed plan save Archer's estate assuming death in 2020 after the restructured plan is implemented, and the QTIP election is made for the entire amount placed in the QTIP trust? A) $31,000 B) $59,000 C) $160,000 D) $0 Explanation Answering this question requires calculating the estate tax liability under the proposed plan and comparing such liability with that of the existing plan. Under the proposed plan, the bequests to Archer's wife and the QTIP trust will receive marital deductions in a corresponding amount and thus will incur no estate tax. (We are told to assume that the QTIP election is made.) The bequest to the family bypass trust will incur an estate tax of $4,577,800, which will be negated by Archer's estate tax applicable credit amount. Because the proposed plan will generate no estate tax due, it will save $160,000—the estimated tax on the existing plan.

c

Ashley has a gross estate valued at $25 million when she dies. She is survived by her spouse, Roland, and their five children. Ashley had previously executed a will that incorporated bypass planning. Which of the following statements regarding Ashley's estate is CORRECT? Ashley's estate will owe significant estate tax. None of Ashley's estate qualifies for the marital deduction. A) I only B) II only C) Neither I nor II D) Both I and II Explanation Statement I is incorrect because with bypass planning (the lifetime exemption amount to a B Trust and the remainder to the marital deduction), no estate tax is due because the entire estate is sheltered by either the estate tax exemption amount (the B Trust) or the marital deduction (the rest of the estate). The portion of the estate that exceeds the estate tax exemption amount qualifies for the marital deduction

c

Assume that in 2020, Rose makes a gift of $25,000 in cash to Samantha. For purposes of the generation- skipping transfer tax (GSTT), which type of transfer is this? A) Qualified transfer B) Taxable distribution C) Direct skip D) Taxable termination Explanation This is a direct skip because it is an outright gift to a skip person. A taxable distribution is a distribution made from a trust to a related person 2 or more generations below the transferor's generation. A taxable termination occurs when an interest in a trust is terminated, resulting in a skip person holding interests in the trust. A qualified transfer is a transfer that is not subject to the generation-skipping transfer tax, such as a payment made directly to an educational institution for tuition or fees.

c

Assuming that a decedent left no valid last will and testament, which of the following assets will pass by the laws of intestate succession? A) Property held by the decedent and his spouse as joint tenants with right of survivorship B) An asset gifted by the decedent during his lifetime C) A life insurance policy owned by the decedent with his wife as the insured D) Assets placed in the decedent's revocable living trust Explanation This asset remains after his death and does not pass by will substitute as the remaining answer choices do.

c

Assuming that a decedent left no valid last will and testament, which one of the following assets will pass by the laws of intestate succession? A) Assets placed in an inter vivos irrevocable trust in which the decedent/grantor was the sole income beneficiary and the decedent's children were the remainder beneficiaries B) A money market account at his bank that was held in the decedent's name that was payable on death (P.O.D.) in favor of his spouse C) Assets held by the decedent and his spouse as community property in a community property state that were designated community property by a nuptial agreement D) A life insurance policy on the decedent/grantor's life placed into an irrevocable life insurance trust (ILIT) for the benefit of the decedent's children two years before the decedent's death Explanation Community property does not have a right of survivorship feature, and thus it must be transferred at death by either will or the laws of intestate succession. Assets included in the trust in each case acts as a will substitute as does a bank account held in P.O.D. form.

c

Bob Daniels and his brother, Jack, own a parcel of rental real estate as tenants in common. They inherited the property from their grandmother, who specified in her will that Bob was to have a 60% ownership interest and Jack the remaining 40%. The property generated an income of $10,000 last year. Since Jack was unemployed, Bob let Jack keep the entire $10,000. The property was worth $100,000 when Bob and Jack inherited it and is now worth $120,000. Their grandmother's basis in the property was $30,000. Bob recently died. Which of the following are CORRECT statements concerning the income tax implications of this form of property ownership and these transactions? Jack's basis in the property after Bob's death is $100,000. Bob's fiduciary must report $6,000 of income from this property for last year. Jack must report $10,000 of income from this property for last year. Jack's basis in the property after Bob's death is $40,000. A) I only B) III and IV C) II and IV D) II only Explanation Option I is incorrect because option IV is correct. Jack does not automatically get Bob's interest in the property since tenancy in common does not have a right of survivorship feature. Option III is incorrect because income for tenants in common is divided according to each tenant's proportional interest. Therefore, Bob was entitled to $6,000 of the income last year. Bob's income tax return for last year must show the income to which he was entitled. Since he allowed Jack to take that income, he will be deemed to have made a gift to Jack of $6,000. However, this gift will not be taxable because of the annual exclusion. Option II is correct for the same reasons option III was incorrect. Option IV is correct because Jack's basis is 40% of the estate tax value in his grandmother's estate (the fair market value of the property at the date of her death). Whoever inherits Bob's 60% of the property will have a basis of $72,000 ($120,000 x .60). If the question would have said that Jack inherited the property from Bob, Jack's new basis would have been $112,000 (his original $40,000 + $72,000 of stepped-up basis from that was in Bob's gross estate).

c

Bob, Frank, Hector, and Fermin are equal partners in a closely held business. Although the four partners work well together, their spouses and children do not. No partner is ready to quit the business and retire, but they are each worried about how the business would operate if this were to happen. Each partner is financially overextended and thus not able to pay a gift tax or capital gains tax, as they started the business from scratch and it has become very profitable. Which one of the following is the most appropriate business transfer technique for the partners to use considering these circumstances? A) A preferred stock recapitalization of the business B) A cross-purchase buy-sell agreement among the partners C) An entity buy-sell agreement between the business and each partner D) A private annuity agreement among the partners Explanation This is the best choice because only four policies are required with an entity purchase plan as compared to the 12 policies a cross-purchase plan would require. Neither a private annuity nor a preferred stock recapitalization make sense for this scenario as the partners are not family members.

c

C.J.'s taxable income fluctuates from year to year. Her adjusted gross income this year is $60,000, but it will probably be less in the future. Her major objective is to reduce income taxes for the current year. She plans to contribute $35,000 to the American Red Cross. Which one of the following is the most appropriate property for C.J. to gift to the American Red Cross to maximize her current-year charitable income tax deduction? A) A tract of farmland, held long term, valued at $35,000 with a basis of $22,000 B) A stamp collection she inherited, held long term, valued at $35,000 with a basis of $12,000 C) A life insurance policy with a face value of $35,000; net premiums, paid by C.J., of $24,000; and a replacement cost of $30,000 D) A stock, held short term, valued at $35,000 with a basis of $19,000 Explanation A gift of the life insurance policy will allow C.J. to take a current-year income tax deduction of $24,000, which is more than any other listed asset. Gifting the stock would allow a current deduction of $19,000. It is rarely wise to gift short-term capital gain property because only the basis can be deducted. The stamp collection is use-unrelated tangible personal property. What will the Red Cross do with a stamp collection? They will sell it and use the proceeds for their charitable work. Donations that are sold by the charity within two years of the gift are consider use-unrelated. That limits the charitable deduction to the basis. Thus, that gift would only allow a current deduction of $12,000. The farmland is ordinary income property. If you own farmland, you are a farmer by definition. The deduction for ordinary income property is always limited to the basis. In this case, donating the farmland to charity would only allow a maximum current-year deduction of $22,000.

c

Charles purchased a parcel of commercial real estate. He wants the bulk of his estate to pass to his wife. However, he plans to pass the commercial real estate to his adult children. Charles does not have a will and does not intend to make one. Which one of the following states the most appropriate form of titling for the commercial real estate to accomplish his objectives? A) Titling it in tenancy in common with his children is preferable because it will pass to them automatically by operation of law at his death. B) Holding it in sole ownership will allow him to pass the property to his children without the surviving spouse taking a share of the property. C) Holding it in joint tenancy with right of survivorship with his children will allow the property to pass directly to them without the consent of the surviving spouse. D) Titling it in tenancy by the entirety will accomplish his testamentary goals and reduce the amount of federal estate tax payable. Explanation Because Charles refuses to execute a will, a will substitute must be used to convey the property without it going through probate. Sole ownership would require probate by means of the intestacy statutes that always give something to a surviving spouse. Tenancy by the entirety can be used only between husband and wife. Tenancy in common does not have an automatic survivorship feature, and therefore, Charles's share of the property would be distributed in probate by the intestacy statutes.

c

Conrad owns a closely held partnership interest that currently represents 60% of the value of his adjusted gross estate (one-half of the closely held business value is real estate). Conrad is concerned about paying sizable estate taxes at his death, so he is considering the inter vivos transfer of part of the partnership interest to his son. If Conrad decides to make the transfer, his partnership interest will be reduced to 30% of his adjusted gross estate. His will bequeaths $30,000 cash to his favorite qualified charity, the business interest to his son, and leaves the rest of the estate to his wife. A disadvantage for Conrad of transferring the business interest, while he is alive, to his son is that the estate will no longer qualify for which of the following? A) A Section 303 stock redemption B) The marital deduction for property given to his spouse C) Section 6166 installment payment of estate taxes D) The alternate valuation date Explanation Transferring the business property will prevent more than 35% of Conrad's adjusted gross estate from being attributable to the value of a closely held business (ownership of real estate is irrelevant). The alternate valuation date is incorrect, as there is no indication that the value of any estate assets was declining, and if they were, the estate could still qualify for this benefit after the transfer. The marital deduction would still be available regardless of the transfer of the business property. A Section 303 stock redemption cannot be used because Conrad owned an interest in a partnership, not a corporation.

c

Corrine wants to donate money to an arrangement that will make income payments to a charity for a specified period of years, with the remainder passing to her grandchildren when that period has expired. Which of the following charitable transfers will meet Corrine's needs? A) Pooled income fund B) Charitable remainder unitrust (CRUT) C) Charitable lead trust (CLT) D) Charitable remainder annuity trust (CRAT) Explanation With a charitable lead trust (CLT), income payments go to a charity and the remainder interest either reverts to the donor or passes to noncharitable beneficiaries. Because the present value of the charity's interest is a large portion of the gift to the trust; and because appreciation above the distribution rate accrues for the remainder beneficiaries, a family that has the ability to go without the asset(s) for the term of the trust can pass the trust assets to family members at the end of the trust term with extremely low transfer taxation (gift or estate taxes). With CRATs, CRUTs, and pooled income funds, the remainder interest passes to a charity.

c

Debra is the majority stockholder in the Jernigan Family Corporation. Her interest in this corporation is the sole asset in her estate other than her personal-use assets. She understands that if she were to die today, an unacceptably large amount of her assets, including the closely held business interest, would have to be sold to pay her estate administrative expenses and death taxes. She would like to do something with her interest in the Jernigan Family Corporation to prevent the need for such a sale because she would like her children to receive her shares. Which of the following actions have the potential to increase the liquidity of Debra's estate? Have the corporation purchase a key person life insurance policy on Debra Gift shares in the Jernigan Family Corporation to her children Amend her will to place her shares into a testamentary trust for her children Establish an irrevocable life insurance trust (ILIT), and fund it with a policy on her life A) II and III B) I, III, and IV C) I, II, and IV D) I and II Explanation Statement III is false because leaving her assets to heirs in a testamentary trust will not increase liquidity or decrease cash needs. The remaining answer choices will all help her achieve her goals.

c

Duran had a gross estate of $11.5 million when he died. He and his wife, Florence, died from injuries sustained in an auto accident. She died two weeks before he did. Part of his gross estate was $6.5 million in stock in the closely held Ortiz Family Corporation, which purchases cargo containers for lease to large shippers. Due to a great increase in demand for the containers, the stock recently has experienced rapid appreciation. His estate had unsecured debts of $300,000 and administrative expenses of $50,000. Duran made $1 million in adjusted taxable gifts. His will leaves his property to his children in equal shares. Which of the following postmortem techniques are available to provide liquidity to Duran's estate? Special use valuation Section 303 stock redemption Alternate valuation date Installment payment of federal estate taxes A) I only B) II and III C) II and IV D) I, III, and IV Explanation Duran's estate meets the 35% adjusted gross estate requirement for a Section 303 stock redemption and installment payment of taxes (Section 6166). The estate will owe estate tax because of the adjusted taxable gifts. The estate cannot qualify for special use valuation because there is no evidence of real estate ownership. Finally, since Duran was a widower at death, the estate cannot qualify for a QTIP election.

c

Estate planners are often asked to assess whether a client's estate plan has adequate provisions to accomplish client objectives. Which of the following would be appropriate questions to ask and answer? A) How titles to property are held. B) Marital and family status of client. C) All of the above. D) Competency of intended beneficiaries. Explanation All would be appropriate questions and there would also be many other appropriate questions to raise to analyze the client's current position and potential courses of action.

c

Estate planning data for Harriet indicates the following: She transferred income-producing real estate to a QTIP trust for the benefit of her spouse, George, four years ago, and she elected the marital deduction for all trust assets. She made a series of present-interest gifts of securities for the maximum annual exclusion to each of her three children over the past five years. Six years ago, she made gifts of twice the amount of the maximum annual exclusion to UTMA accounts for each of her four nieces and nephews, naming her brother, Fred, as custodian; George agreed to treat the gifts as split gifts. When she became ill last year, she gave her interest in the family farm to Fred; she withdrew money from a bank account to pay the gift tax due that was in excess of the applicable credit amount. Which of the following statements about the impact of the gifts on her estate tax liability is NOT correct? A) By making the series of gifts to her children, Harriet has transferred assets without making them subject to any transfer tax. B) The gift to her husband, George, will reduce her estate tax liability by deferring taxation until George dies and the value of the trust becomes part of his gross estate. C) The gift tax paid by Harriet last year from her bank account will reduce the size of her gross estate, and thus, her estate tax liability, but only if she dies in the next two years. D) Harriet's gifts to her nieces and nephews reduced her estate tax liability by reducing her tax base. Explanation This is the correct answer because the statement regarding the effect of gifting and the gross-up rule is incorrect. Under the gross-up rule, gift taxes paid out of pocket on gifts made within three years of a donor's death must be added back to her gross estate. Harriet's withdrawal from her bank account does not reduce the size of her gross estate, and therefore, her tax liability until at least three years have passed. When gifts are not taxable, they were not subject to gift tax, nor will any part of these gifts be included in her estate tax calculation as adjusted taxable gifts. Also, they were not subject to generation-skipping transfer tax since the donees were not skip parties in relation to Harriet. The property placed in the QTIP trust for George will be removed from Harriet's gross estate as a completed gift, and it will not be included as an adjusted taxable gift because the marital deduction keeps the transfer from being taxable. The transfers to the UTMA accounts reduce her gross estate by the total amount transferred, and no part of this amount is added back as adjusted taxable gifts because of gift splitting and the annual exclusion. (Transfers to UTMA accounts are eligible for the annual exclusion.)

c

Fran and her husband, Dan, own their residence in joint tenancy with right of survivorship. Until recently, the property was held solely in Fran's name. Then, at the suggestion of her advisors, Fran changed the title. Dan furnished no consideration at the time of transfer. Which one of the following statements concerning the gift tax treatment of the transfer from Fran to Dan is CORRECT? A) There is a taxable gift of one-half of the value of the residence, less the amount of the annual exclusion. B) Dan has made a taxable gift of one-half of the value of the property. C) Fran has no gift tax liability because the annual exclusion and the gift tax marital deduction reduce the taxable gift to zero. D) Fran has made a taxable gift of one-half of the value of the residence because Dan paid no consideration. Explanation Although Fran has made a gift to Dan, it is not a taxable gift because of the annual exclusion and the marital deduction. The word taxable means that after all applicable deductions (gift splitting, annual exclusion, marital, and charitable deductions) a value still remains to be gift taxed.

c

Gary and Georgeann Sutter have the following objectives: For Gary to provide Georgeann exclusively with a mandatory stream of income from the assets included in his gross estate if he predeceases her To ensure that Gary's children from his prior marriage will ultimately receive the income-producing assets upon Georgeann's death To prevent assets used to provide income to Georgeann from being included in her gross estate Which of the following estate planning techniques would accomplish the Sutters' first objective of providing a mandatory stream of income? a power of appointment trust a QTIP trust, with an election a QTIP trust, without an election A) II only B) II and III C) I, II, and III D) I only Explanation The key portion of the Sutters' first objective is that Gary wants Georgeann to have a mandatory income stream. All of these trusts must grant the surviving spouse a mandatory income stream to qualify for the marital deduction. In a QTIP trust, there is a mandatory income stream to the surviving spouse whether or not the election is made. The election affects only tax goals, not distribution goals.

c

Gary and Georgeann have the following objectives: If Gary predeceases Georgeann, to provide her exclusively with a mandatory stream of income from the assets included in his gross estate To ensure that Gary's children from his prior marriage will ultimately receive the income-producing assets upon Georgeann's death To prevent assets used to provide income to Georgeann from being included in her gross estate Which of the following estate planning techniques would accomplish Gary and Georgeann's first objective of providing a mandatory stream of income? A power of appointment (A) trust A qualified terminable interest property (QTIP) (C) trust, with an election A QTIP (C) trust, without an election A) I only B) II and III C) I, II, and III D) II only Explanation The key portion of their first objective is that Gary wants Georgeann to have a mandatory income stream. All of these trusts must grant the surviving spouse a mandatory income stream to qualify for the marital deduction. In a QTIP trust, there is a mandatory income stream to the surviving spouse whether or not the election is made. The election affects only tax goals, not distribution goals.

c

Georgia established a trust over which she retained the power to revoke. She transferred $150,000 of income-producing securities to the trust. The trust sold some of the securities and purchased life insurance on Georgia's life, using the income from the remaining securities to pay the premiums. Excess income is to be accumulated in the trust. Georgia died two years and 364 days after the creation of the trust. Which one of the following statements correctly describes a disadvantage of this trust arrangement for Georgia? A) She gives up the power to control the disposition of the insurance proceeds through transfer to the trust. B) She must include the policy proceeds in her estate as a transfer within three years of death but could have excluded them if she had lived only a few days longer. C) She cannot take advantage of income shifting because the trust income is taxable to her under the grantor trust rules. D) She cannot take advantage of the gift tax annual exclusion because payment of the premiums constitutes a future interest. Explanation Because Georgia retained the power to revoke, the grantor trust rules apply, and Georgia will be taxed on all trust income whether disbursed to her or not. The other choices are incorrect. The power to revoke obviously includes the power to determine the beneficiaries of trust assets. The three-year or inclusionary rule does not apply because Georgia never gave up any incidents of ownership in this policy. At all times, Georgia was the owner of this policy because of her right to revoke the trust. Payment of the premiums (with trust income) is not a gift because of her power to revoke the trust.

c

Harold established and funded a generation-skipping trust for the benefit of his lineal descendants by a provision in his will. The trust had no termination date as the trust was settled in a state with no rule against perpetuities. He funded the trust with assets in the amount of the estate tax exclusion amount in the year of his death, and assigned all of his generation-skipping transfer tax (GSTT) exemption to this trust as he had not used any part of this exemption previously. Distributions to beneficiaries are at the discretion of the trustee. Harold was survived by his spouse, five children, and 12 grandchildren. All other estate assets were given to Harold's spouse. Which of the following statements regarding this trust are CORRECT? This is an example of an indirect generation-skipping transfer. Harold's spouse can become the transferor of this transfer if a reverse QTIP election is made. No GSTT will ever be due on this transfer if it is reported on Harold's estate tax return, and the deemed allocation rules are allowed to allocate his GSTT exemption. The trust is considered to be a skip party for GSTT purposes. A) I, II, III, and IV B) I, II, and III C) I and III D) III and IV Explanation Because Harold's children are beneficiaries of the trust, this trust is an example of an indirect (option I) skip. The transferred property may go to both skip parties (Harold's grandchildren, etc.) and nonskip parties (Harold's children). Option III is correct because the maximum GSTT exemption is the same amount as the estate tax exclusion amount for the year of transfer; when this exemption is assigned to a taxable amount of transferred property of the same amount on the first gift or estate tax return on which the transfer is first reported, no amount of future distributions will be taxable regardless of the growth in the corpus. The deemed allocation rules would assign the full amount of Harold's GSTT exemption in these circumstances. Harold's spouse cannot become the transferor of this property. A reverse QTIP election (option II) can be made only when the regular QTIP election has been made. This trust is not a QTIP trust as Harold's spouse is not a beneficiary of the trust. This trust is not considered a skip party (option IV) because future distributions can be made to nonskip parties.

c

Hong's gross estate is $15.5 million. His estate taxes are $1.4 million, and his estate administration expenses are $500,000. Assuming his estate owns closely held stock and qualifies for a Section 303 stock redemption, what is the maximum dollar amount of stock that may be redeemed under Section 303? A) $1,000,000 B) $1,400,000 C) $1,900,000 D) $500,000 Explanation Under Section 303, only an amount of stock equal to the total of the decedent's estate taxes plus administration expenses is eligible for the favorable tax treatment. In Hong's case, the maximum amount of stock that may be redeemed under Section 303 is $1,900,000 ($1,400,000 + $500,000).

c

If Arthur Greene dies today, which one of the following assets will be included in his probate estate? A) His profit-sharing plan B) His IRA C) The family limited partnership (FLP) he has set up. D) His personal residence owned jointly with his spouse. Explanation Arthur's FLP interest will be included in the probate estate. The other assets will pass via will substitute.

c

If Marleen Harris died and her daughter Tasha Harris decided to disclaim part or all of her inheritance, which of the following steps should Tasha take to make sure that any such disclaimer is a qualified disclaimer? The disclaimer must be irrevocable. The disclaimer must be in writing. The disclaimer must be filed within six months of Marleen's death. Tasha cannot have benefited from any disclaimed assets before execution of the disclaimer. A) II, III, and IV B) II and IV C) I, II, and IV D) I and II Explanation Tasha should make sure the disclaimer is writing and is irrevocable. Additionally, she should make sure that it is filed with the executor within nine months, not six months. Finally, Tasha cannot have benefited from disclaimed assets and cannot try to direct Marleen's disclaimed assets to particular heirs.

c

If a decedent's property does not pass to someone by will substitute or by will, and there are no legal heirs under the applicable state intestate succession statute, the property will A) be held by the court until a distant relative can be located and petitions the court for distribution. B) be claimed by the IRS as a death tax. C) escheat to the state. D) be donated to a charitable organization. Explanation The property will be held by the state in trust for a stated number of years, and if no legal heirs under the intestacy statutes come forward, the property escheats to the state.

c

In 2020, Michael incurs substantial medical bills at the local clinic and Esteban pays $50,000 directly to the hospital in payment of Michael's medical expenses. What is the amount of Esteban's taxable gift as a result of this transaction? A) $85,000 B) $100,000 C) $0 D) $30,000 Explanation The payment of another person's medical expenses directly to the medical provider is a qualified transfer and is not considered a gift for gift tax purposes.

c

In which of the following situations is a holder's power to appoint property to himself considered to be a general power of appointment over the entire property for gift tax purposes? The holder's power is limited by an ascertainable standard so that the power is exercisable only for the holder's health, education, maintenance, or support (HEMS). The holder may exercise the power only with the consent of the grantor or a third party who has an interest that is adverse to the holder's. The right to exercise the power each year is limited to the greater of $5,000 or 5% of the total value of the property subject to the power. A) I, II, and III B) II and III C) None of these D) I only Explanation None of the statements describe situations in which a holder's power to appoint property to himself is considered a general power of appointment over the entire property. For gift tax purposes, the power of the holder to appoint property to himself is not a general power of appointment in any of these situations.

c

In which type of charitable trust does the income interest pass to a qualified charity and the remainder interest pass to one or more noncharitable beneficiaries? A) Charitable remainder annuity trust (CRAT) B) Charitable remainder unitrust (CRUT) C) Charitable lead trust (CLT) D) Pooled income fund (PIF) Explanation With a charitable lead trust, income payments go to a charity and the remainder interest passes to noncharitable beneficiaries. With CRATs, CRUTs, and PIFs, the remainder interest passes to a charity.

c

Jack created an irrevocable trust in 2014 for the benefit of his son, Bill, and his granddaughter, Karen. This year, the trustee distributes $40,000 in trust income to Karen. For purposes of the generation-skipping transfer tax (GSTT) the $40,000 distribution to Karen is a A) qualified transfer. B) taxable termination. C) taxable distribution. D) direct skip. Explanation This is a taxable distribution because it is a distribution made from a trust to a related beneficiary two or more generations below the transferor.

c

Janelle wants to draft a power of attorney that will allow her daughter to manage Janelle's property if Janelle ever becomes incapacitated. Which of the following powers of attorney will meet Janelle's needs? Nondurable power of attorney Durable power of attorney A) Neither I nor II B) I only C) II only D) Both I and II Explanation Only statement I is incorrect. Statement II is correct. Only a durable power of attorney survives the principal's incapacity. A nondurable power of attorney becomes invalid if the principal becomes incapacitated. Both powers of attorney become null and void when the principal passes away.

c

Jocelyn dies owning a large block of stock in a closely held corporation. Her estate is short of liquid assets, and the executor needs to sell the stock to pay estate expenses. The only potential buyer for the stock is the corporation itself, but Jocelyn's executor is afraid that if she sells the stock back to the corporation, the money received will be taxable as a dividend. Which of the following postmortem estate planning techniques would be most useful to Jocelyn's estate? A) Reverse QTIP election B) QTIP election C) Section 303 stock redemption D) Special use valuation (Section 2032A) Explanation A Section 303 stock redemption would be useful to Jocelyn's estate because it would permit the estate to sell stock back to the corporation and have the transaction treated as a sale rather than a redemption. If the transaction is treated as a sale, any gain would be treated as a capital gain (relative to the stepped-up basis from the date of death value). The proceeds from a non-Section 303 redemption would be treated as an ordinary dividend.

c

Joellen dies on March 13. Her gross estate includes publicly traded stock that has a market value of $6 million on Joellen's date of death. Joellen's estate sells the stock for $5.7 million on May 2. The same stock has a market value of $5.5 million on September 13. If Joellen's estate elects the alternate valuation date, the stock will be valued at what amount in Joellen's gross estate? A) $5.6 million B) $5.5 million C) $5.7 million D) $6 million Explanation If an estate elects the alternate valuation date and sells an asset before the six-month valuation date, the asset must be valued at its sale price, regardless of whether that price is more or less than the AVD value.

c

John Cie, before he was sworn in as governor of his state, established a trust to manage his property through a corporate trustee, without his direction or input, for an irrevocable period of six years—the length of his term of office. As he intends to live solely from his salary as governor, none of the trust income is to be distributed to him. At the end of the six-year term, the principal of the trust will revert to John, but all accrued income is to be distributed equally to John's spouse and to his children, who are currently five and nine years of age. Which one of the following is a correct statement regarding this trust? A) The income will be taxed to the trust until distributed, and to the recipients upon distribution. B) The income of this trust will be taxable to John because he has retained the power to revoke the trust. C) At least one-third of the income of this trust will be taxable to John even though he will never receive it. D) The income of this trust will be taxable to John, as his spouse has retained a reversionary interest that exceeds 5% of the value of the trust at its creation. Explanation This trust is subject to one of the grantor trust rules since the trust income is accumulated and one-third of the income will be distributed to the grantor's spouse (and possibly all of the income will be taxed to John because of his reversionary interest). The grantor trust rules apply to at least part of the income. John's spouse has retained nothing, and John has not retained such powers.

c

Jon and Bob own a house as tenants in common. Jon owns 30% and Bob owns 70%. They purchased the house 10 years ago for $125,000 and today the house is valued at $200,000. If Jon dies today, what amount is included in Joe's gross estate? A) $37,500 B) $140,000 C) $60,000 D) $100,000 Explanation Because the property is owned as tenants in common, the amount included in the gross estate is based on Jon's share of the fair market value on the date of death. Therefore, the value included in Jon's gross estate is $60,000 (30% of $200,000).

c

Jose and Maria have been married for 50 years. They have three children and seven grandchildren. Their estate is $30 million. They do not want to rely on the deceased spouse's unused exemption (DSUE) because they have seen a lot of tax law changes through the years. They would like to arrange their assets so there are no estate taxes when the first spouse dies. Maria wants to be sure their son, Fred, receives their vacation home when the second of them passes away. Jose wants to ensure their daughter Amelia eventually takes possession of their family home upon the second of the couple to die. Other than these assets, they are fine with allowing the survivor to control the remaining property. Which of the following will best allow them to accomplish all three goals? A) A Type A power of appointment trust B) A Type B bypass trust C) A combination of an A trust and a B trust D) A Type C qualified terminable interest property (QTIP) trust Explanation The combination of the two is the only way to achieve all of their goals as the remaining choices except for the QTIP will allow for some of the goals to be achieved, but not all of them. The QTIP trust doesn't align with their goals at all.

c

Kris dies, leaving an adjusted gross estate of $20 million, which includes several hundred shares of closely held stock. For Kris's estate to qualify for a Section 303 stock redemption, the value of the closely held stock must exceed what amount? A) $5 million B) $2 million C) $7 million D) $10 million Explanation To qualify for a stock redemption under Section 303, the value of the closely held stock must exceed 35% of the decedent's adjusted gross estate.

c

Last year, Julie Poppins sold to her daughter, Mary, a daycare business for $180,000, which was its fair market value. Julie's basis in the business was $90,000. Mary gave Julie an unsecured promissory note in which she promised to pay the purchase price in 15 annual installments composed of only interest at the prevailing rate for the first five years, with each of the remaining 10 annual payments to be composed of $18,000 principal, plus interest at the same rate. At Mary's request, the note also contained a provision that if Julie died while any part of the note was not yet due, the payments not yet due would be canceled. Which of the following statements correctly describe the tax implications of the intrafamily sale that Julie has made to Mary? If Julie dies while any part of the note remains outstanding, her gross estate must include the fair market value of the daycare business. If any annual installments under the terms of the note are canceled, the present value of the canceled installments will be included in Julie's gross estate. Cancelation of any annual installments by Julie's estate under the terms of the note will cause her estate to realize a taxable gain on the forgiven installments. If Mary closes the daycare business before Julie dies, and Julie cancels the entire note, Julie will be subject to both income taxes and gift taxes. A) II and III B) I, II, III, and IV C) II, III, and IV D) I and IV Explanation The answer is II, III, and IV only. One key to this question is that this is only an installment sale and not a self-canceling installment sale (SCIN). We know this because a SCIN must pay a premium over the normal installment sale. The premium, which would be calculated by a CPA is either a higher interest rate or a larger initial principal amount (the fair market value (FMV) + the SCIN premium amount). In this case, the principal was the FMV and the interest rate is the prevailing interest rate, not a higher than prevailing interest rate as the premium for a SCIN. Only option I is an incorrect statement because Julie's estate would include the present value of the forgiven payments, but would not include the value of the daycare business as of the date of her death. Option II is correct because in such circumstances, Mary did not pay the required premium for the cancelation provision. If this premium is not paid, the transaction is an installment sale rather than a valid SCIN. When payments pursuant to an installment sale are canceled or forgiven, the seller must recognize any gain in the forgiven payments, and pay transfer tax on them. Option III is correct because in such circumstances, Julie will have to recognize for income tax purposes the proportionate gain (FMV less basis) that she would have otherwise recognized had the payment not been canceled. Option IV is correct for the same reasons as stated for options II and III.

c

Last year, your client and his wife gave their adult son a one-third interest in a commercial office building. Each has a one-third interest as tenants in common. If your client dies while still owning the property as a tenant in common, an estate tax implication of this form of property ownership is that A) the entire value of the property will be included in your client's gross estate because his estate cannot prove contribution by the other tenants in common. B) your client's estate will be entitled automatically to a marital deduction of one-half of the date-of-death value. C) one-third of the value of the property will be included in your client's gross estate. D) one-half of the value of the property will be included in your client's gross estate. Explanation Each tenant in common owns their share of the property and as owner, that share is included in the gross estate of each.

c

Leilani made lifetime taxable gifts of $2.5 million. At her death in 2020, she was survived by her spouse and daughter and owned the following property interests: Sole ownership: Life insurance on her spouse's life with a death benefit of $5 million and a replacement cost of $100,000, with her children as the beneficiaries An investment portfolio worth $7 million A personal residence with a fair market value of $1 million Co-ownership: Property held with her spouse as joint tenants with right of survivorship (JTWROS) worth $4 million total Property held with her daughter as tenants in common, including a vacation condo valued at $750,000 (Leilani owns 80%) In her will, Leilani bequeaths the personal residence outright to her spouse and leaves the remainder of her probate estate to her daughter. Leilani had $200,000 of combined debts, funeral, and administrative expenses. Her estate paid $50,000 in state death taxes. Many years ago, she paid $210,000 in gift tax out of pocket. Which of the following amounts most closely approximates the net federal estate tax liability for Leilani's estate? Use the Unified Federal Estate and Gift Tax Rates table. A) $4,300,000 B) $3,945,800 C) $0 D) $414,900 Explanation Leilani's gross estate is $13,200,000 ($2.5 million in taxable gifts + $100,000 replacement cost of the life insurance on her spouse's life + $7 million in investments + $1 million personal residence + $2 million as her half of the JTWROS property + $600,000 as her share of the vacation condo). But, the residence and JTWROS property are eligible for the marital deduction, removing $3 million from her estate and thereby reducing it to a level where her applicable credit would eliminate taxes. The debts and expenses, death taxes, and gift taxes previously paid would further reduce the taxable estate.

c

Lela was a widow at her death in 2020. She died with a gross estate of $11,705,000, consisting entirely of publicly traded income-producing stock. Her debts were $75,000, and her estate administrative expenses were $50,000. Lela made no lifetime taxable gifts. She left her entire estate to her daughter. Which one of the following postmortem planning techniques will help meet the liquidity needs of Lela's estate? A) Application to pay estate taxes under Section 6166 B) Use of the alternate valuation date C) Use of the election to take the estate administrative expenses as a deduction on the estate's fiduciary income tax return D) Special use valuation Explanation The value of income-producing stock is unlikely to have declined over time, so the alternate valuation date would be of no benefit. In addition, the alternate valuation date is not available when no estate tax will be due after use of the applicable credit amount. The estate does not qualify for special use valuation even if tax were due since it has no real estate used in a closely held trade or business or farming operation. After deducting the estate administrative expenses of $50,000 and debts of $75,000, there would be no estate taxes to be paid (gross estate of $11,705,000, minus debts of $75,000 and debts of $50,000, leaves a taxable estate and tax base—no adjusted taxable gifts—of $11,580,000, which is covered by Lela's estate tax applicable credit amount), so there would be no need to apply to pay estate taxes in installments under Section 6166. In addition, Lela's estate does not qualify under this section since her estate does not contain a closely held business interest exceeding 35% of her adjusted gross estate.

c

Leon Brown has consulted a CFP® professional for estate planning advice. One of the CFP® professional's recommendations is that Leon take steps to avoid probate when he dies. In communicating this recommendation to Leon, which of the following statements made by the CFP® professional would be CORRECT? Probate will not be necessary if Leon dies testate. Avoiding probate will help keep Leon's affairs private after he dies. Probate might delay the distribution of Leon's assets to his heirs. A) II only B) I and III C) II and III D) I, II, and III Explanation Statement I is incorrect because probate is necessary when a decedent dies testate (with a valid will). Probate is also necessary for those who die intestate. Statements II and III are correct.

c

Lonnie wants to execute a power of attorney naming his son, Brad, as attorney-in-fact. Lonnie wants Brad's authority to terminate if Lonnie ever becomes legally incapacitated. Which of the following types of power of attorney will meet Lonnie's wishes? Durable power of attorney Nondurable power of attorney A) I only B) Both I and II C) II only D) Neither I nor II Explanation Only statement II is correct. A durable power of attorney survives the principal's legal incapacity (but not the principal's death). A nondurable power of attorney becomes legally invalid at the onset of the principal's incapacity. That means a nondurable power of attorney becomes ineffective when the principal needs it the most. However, this is what the holder is asking for in this situation.

c

Luka is the personal representative (PR) of an estate, and he is concerned that there may not be sufficient liquidity within the decedent's probate estate to pay all of the creditor's claims. Which of the following statements regarding estate liquidity is CORRECT? A) The PR may always use probate assets, but nonprobate assets may only be used with permission from the court to provide estate liquidity. B) The PR may always use nonprobate assets but may not be able to use probate assets. C) The PR may utilize probate and nonprobate assets to provide estate liquidity. D) The PR may always use probate assets, but nonprobate assets may only be used with consent from the beneficiaries to provide estate liquidity. Explanation In addition to meeting liquidity requirements from assets that are part of the probate estate controlled by the estate's PR, they may also be met from nonprobate assets that are passed to estate beneficiaries by will substitute. This may either be voluntary (e.g., a loan) or involuntary by a court order enforcing a claim or lien (e.g., for the beneficiary's share of transfer taxes or administrative expenses) against the nonprobate property.

c

Margie is the sole owner of a successful business that constitutes the vast majority of her wealth. Margie never married, nor does she have any children or close relatives. Because of her health, she has been advised to retire, but cannot afford to do so unless she can get some value for the business. Burt, one of her employees, has offered to purchase the business, but has little money of his own, and it is unlikely he could obtain a loan for even a down payment, let alone the full value of the business. Burt has been a good "second in command," but Margie has doubts about his ability to successfully lead the business. Which one of the following is the most appropriate form of business transfer technique for Margie to use to achieve her objectives if she decides to accept Burt's offer? A) A private annuity sale transaction so that she can be assured of receiving payments until she dies B) A cross-purchase buy-sell agreement C) An installment sale transaction with the purchase price being the fair market value of the business D) A gift leaseback transaction so that she can control the business until she dies Explanation An installment sale would allow Margie to get full value for the business, report her capital gain on an installment basis, and secure the promised payments by a lien on the assets and accounts receivable of the business. A gift leaseback transaction would not allow Margie to successfully transfer her business, but perhaps some business equipment. In addition, Margie would incur a gift tax liability for any assets transferred and would still need a plan for her business interest. With a private annuity, the seller must report all gain in the year of sale. Because Burt cannot get a loan for the down payment, Margie would have to pay the capital gains tax out of her own funds rather than in installments as she receives the payments under the installment sale. A cross-purchase buy-sell agreement would not allow Margie to retire immediately, and it is appropriate only when the contracting parties are already owners of the business.

c

Mario has accumulated significant wealth over his lifetime, and he is currently implementing gifting techniques. He would like to take advantage of the annual exclusion. Transfers to which of the following trusts/accounts permit Mario to utilize the gift tax annual exclusion? Uniform Gift to Minors Account (UGMA) Grantor retained annuity trust (GRAT) Qualified tuition plan Section 2503(c) trust A) I, II, and IV B) II and III C) I, III, and IV D) I and IV Explanation A transfer to a GRAT is a gift of a future interest that is not eligible for the annual exclusion. Transfers to an UGMA, a qualified tuition plan, a Section 529 plan, and a Section 2503(c) trust are all eligible for the gift tax annual exclusion.

c

Michael and Marie are currently saving for Max's and Sam's high school educations by investing in CDs that are in the children's names. They are concerned that Max and Sam might cash in the CDs and use the funds for noneducational purposes once they reach the age of majority, which is age 18 in the state where the Andersons live. Which of the following planning alternatives might alleviate the Andersons' concerns? Establishing custodial accounts (i.e., UGMA or UTMA) for the children's benefit Establishing trusts for the children's benefit A) Both I and II B) Neither I nor II C) II only D) I only Explanation Statement I is incorrect because with a custodial account, the account legally belongs to the child once the account is established. The children could still access the accounts when they turn 18. Statement II is correct. Establishing trusts would allow the Andersons to specify a date when the income and principal will be distributed to the children.

c

Mohammed and his spouse, Fatema, own $13 million worth of property as equal joint tenants with right of survivorship. Neither has made any lifetime taxable gifts. Assuming death in 2019, Mohammed's estate tax liability would be zero, but Fatema's would be $640,000, assuming Mohammed's DSUE amount is not available. Mohammed and Fatema may revise their estate plan in the following ways: Transferring $6.5 million into Mohammed's name as sole owner and $6.5 million into Fatema's name as sole owner Amending their wills to provide that their solely owned property shall be placed in a trust that gives the trustee the right to pay as much income to the surviving spouse as the institutional trustee, in its sole discretion, determines appropriate, with the the remainder going to their children. The surviving spouse has a right of invasion of trust corpus based on receiving the children's consent. If the plan is implemented, and both spouses died in 2020, which of the following amounts best describes the family's estate tax savings? A) $320,000 B) $5,145,800, only if Fatema dies first C) $640,000, regardless of who dies first D) $0 Explanation The trust described does not qualify for the marital deduction—it gives the surviving spouse a terminable interest without an exclusive lifetime mandatory right to income. The taxable amount in both estates would be $6.5 million, which can easily be covered by the estate tax credit amount of $4,577,800—the tax on $11.58 million. Thus, neither Mohammed's nor Fatema's estate would owe estate taxes, regardless of which one dies first. Savings would be $640,000 ($640,000 tax under the old plan minus zero tax under the new plan).

c

One common estate planning concern of unmarried persons who live together in a close relationship is avoiding exposure to a lawsuit regarding the ownership of property and rights to income earned by the other person when the relationship ends. Which one of the following techniques can be used to address this type of problem? A) Use of a QDRO (qualified domestic relations order) B) Use of a funded ILIT C) Entering into a property ownership and settlement agreement prior to, or early in, the cohabitation relationship D) Accumulating a large estate and focusing on liquidity planning Explanation The other techniques are more appropriate to solve other problems. For example, another problem that cohabiting couples have is not being able to defer the estate tax at the death of the first cohabitant because they are not entitled to a marital deduction. Therefore, general liquidity planning to pay estate taxes that will be due at the death of the first cohabitant becomes a major focus of estate planning. A funded ILIT might be used to provide the needed liquidity without increasing the gross estate so that other assets will be available to pass to the surviving cohabitant under the deceased's will. A QDRO is used in the case of the separation or divorce of a married couple to protect one spouse's interest in the other spouse's employee benefit plans—it is available only to couples who are married.

c

Owning property in joint tenancy with right of survivorship (JTWROS) is a will substitute because A) the property is subject to probate. B) this form of ownership eliminates the need for a will. C) the property passes outside of probate. D) the property may be owned by more than two owners. Explanation It is precisely because JTWROS passes property outside of probate that it is a will substitute.

c

Paul and Cheryl are husband and wife who initially lived in a community property state. Soon after their marriage they began establishing an emergency fund using money that each earned from their respective jobs. This fund was used to meet unexpected expenses as they arose. Three years ago, Cheryl liquidated stock that she had purchased prior to her marriage, and placed the proceeds in the emergency fund. There have been many deposits and withdrawals from the fund since that time. Last year, Paul filed for divorce. Cheryl is seeking to recover the full value of the stock proceeds that she placed in the emergency fund as her sole and separate property, and half of the remaining emergency fund. Paul claims he is entitled to half of the entire emergency fund. Which one of the following statements is CORRECT regarding Paul's and Cheryl's rights in the emergency fund? A) The entire emergency fund is community property, and Cheryl and Paul are each entitled to a percentage of the total emergency fund equal to their respective contributions. B) The entire emergency fund is separate property, and Cheryl and Paul are each entitled to a percentage of the total emergency fund equal to their respective contributions. C) The stock proceeds are community property, and Cheryl and Paul are each entitled to one-half of the total emergency fund. D) The stock proceeds are Cheryl's separate property, and she should be entitled to recover these funds in full as well as one-half of the remaining emergency fund. Explanation Because Cheryl commingled separate property with community property, the stock proceeds lost their separate property status and because community property.

c

Rhonda owns the following assets: A solely owned closely held business that comprises one-half of the value of her estate A collection of antique figurines that forms a substantial part of her remaining estate A residence owned with her husband as tenants by the entirety Rhonda's will bequests $10,000 to her only niece and leaves the balance of her estate to her husband if he survives her. Rhonda is looking for methods to provide the liquidity needed for her estate. Which one of the following actions would have the potential to improve the liquidity of Rhonda's estate? A) Retitling the residence she owns with her husband as joint tenants with right of survivorship B) Entering into a cross-purchase buy-sell agreement with her husband regarding the business C) Eliminating the bequest in her will to her niece D) Amending her will to place the antique figurines in a testamentary estate trust Explanation None of the other actions will have any effect, positive or negative, on liquidity.

c

Ricky and Lucy, a married couple with two young children and minimal assets, are considering creating an estate plan. Which one of the following statements is CORRECT? A) Ricky and Lucy need an estate plan to reduce their estate tax exposure. B) Ricky and Lucy do not need an estate plan since they do not have any financial concerns. C) Ricky and Lucy should create an estate plan even though their assets do not exceed the applicable exclusion amount. D) Ricky and Lucy do not need an estate plan since they own minimal assets that do not exceed the applicable exclusion amount. Explanation There are many non-tax reasons for developing an estate plan, such as planning to meet the needs of their dependent children. With minimal assets, they will not be faced with having to address estate taxes.

c

Rita created an irrevocable trust and funded it with securities valued at $200,000. The trustee has discretion to distribute income to Rita's son, Wally, for his health, education, maintenance, or support until he reaches age 25, at which time the trustee is to distribute all assets in the trust to him. Rita named herself trustee. Which one of the following statements concerning the gift tax treatment of the trust is CORRECT? A) This is an incomplete transfer because of Rita's power to distribute income. B) This is an incomplete transfer because Wally's enjoyment of the trust assets is postponed until a future date. C) This is a completed transfer to Wally of the present value of the remainder interest. D) This is an incomplete transfer because the gift to Wally is a partial interest. Explanation Because the trust is irrevocable, the remainder interest is irretrievably given to Wally. Even though Wally may never receive anything until age 25, the transfer is complete because neither Rita nor anyone else can change his interest. Finally, Rita's retention of a fiduciary power limited to an ascertainable standard will not cause a transfer that is otherwise complete to become an incomplete transfer. Rita's power is subject to an ascertainable standard because it is limited to a demonstrated need of funds for "health, education, maintenance, or support." Because Wally will get all income from the trust (either before or after he reaches age 25), the entire value of the trust assets is a completed gift. Thus, Rita must declare the present value of both the remainder and the income interest as a gift.

c

Rollie plans on purchasing some U.S. savings bonds with his son, Steven. He has been told that he can title the bonds either as "Rollie or Steven" or "Rollie payable on death to Steven." Which of the following statements are CORRECT regarding advantages and disadvantages of these two methods of titling? "Rollie or Steven" would not avoid probate of the bonds. "Rollie payable on death to Steven" would give Rollie sole control of the bonds during his life. "Rollie payable on death to Steven" would allow Rollie to remove Steven as beneficiary. "Rollie or Steven" would allow the survivor to become the sole owner of the bonds without the bonds going through probate. A) I and III B) I, III, and IV C) II, III, and IV D) II and IV Explanation Rollie would avoid probate using a joint tenancy with right of survivorship (JTWROS) or payable on death (P.O.D.) designation because these are will substitutes.

c

Saquon transferred the ownership of a whole life insurance policy he owned to his daughter last year. This policy had a face value of $1,500,000 and a cash value of $125,000. He also owns a universal life insurance policy with a death benefit of $250,000 and a cash value of $50,000. If Saquon died today, what amount of life insurance proceeds would be included in his gross estate for estate tax purposes? A) $1,500,000 B) $0 C) $1,750,000 D) $250,000 Explanation The death benefits of the whole life policy ($1,500,000) and the variable universal life policy ($250,000) would both be included in Saquon's gross estate. The death benefit from the whole life policy is included because Saquon transferred ownership of the policy within three years of his death. The death benefit from the variable universal life policy is included because Saquon owned the policy on his life at his date of death.

c

Sarah and Jenny, a married couple, own real estate as JTWROS, and Sarah is contemplating a transfer of her interest to Jenny. Assuming the transfer occurs, which of the following statements are CORRECT? A taxable gift occurs. The transfer is eligible for the marital deduction. A) I only B) Neither I nor II C) II only D) Both I and II Explanation Only Statement II is correct. A taxable gift has not occurred because of the marital deduction.

c

Since undergoing a quadruple bypass operation, Robert has become concerned that future heart problems may leave him incapacitated. He wants to grant his brother the right to make crucial medical decisions for him in the event he ever becomes unable to make decisions for himself. Which of the following documents best meets Robert's needs? A) Living will B) Springing power of attorney for property C) Durable power of attorney for health care (DPOAHC) D) Nondurable power of attorney Explanation A durable power of attorney for health care (DPOAHC) will become effective if Robert becomes incapacitated and will grant the attorney-in-fact (his brother) the power to make health decisions for him. A living will does not create a power of attorney but only states the drafter's wishes should he ever become terminally ill. A nondurable power of attorney would cease to be effective if Robert ever became incapacitated.

c

Takisha owns an undeveloped tract of land located in a state other than the state of her domicile. She would like to develop this tract so that it produces income for her retirement years. Although Takisha's current will gives everything to her spouse, she now wants this tract to go to her niece at her death. She has also been informed by her CFP® professional that she should begin taking steps to make her estate more liquid at her death. Which of the following actions regarding this tract can Takisha take to accomplish her objectives and improve her estate liquidity? Gift a remainder interest in the tract to her niece and retain a life estate in the property Amend her will to give the tract to her niece Place title to the tract in the name of the trustee of a revocable trust that Takisha establishes, naming herself as income beneficiary and the niece as remainder beneficiary. Establish a testamentary trust in her will that leaves the tract to her niece. A) II, III, and IV B) II and IV C) I and III D) I, III, and IV Explanation Statement II is false because amending her will will have no effect on estate liquidity. Statement IV is false because a testamentary trust will do nothing to increase cash or reduce cash needs at Takisha's death.

c

The Bells have decided to establish a charitable trust. Their goal is to receive an annual income from the trust during their lives, with the trust assets passing to charity when they die. Because they both expect to live a long time, they want the income from the trust to provide them with a potential hedge against inflation. Which of the following charitable trusts will best meet the Bells' objectives? A) Charitable lead unitrust (CLUT) B) Charitable lead annuity trust (CLAT) C) Charitable remainder unitrust (CRUT) D) Charitable remainder annuity trust (CRAT) Explanation A charitable remainder unitrust (CRUT) will best meet the Bells' needs because it provides an annual income payment based on the market value of the trust assets as revalued each year and will provide a potential hedge against inflation. A CRAT provides a fixed annual income payment. A CLAT or CLUT will not meet the Bells' needs because charitable lead trusts provide an income interest to a charity and a remainder interest to noncharitable beneficiaries. This is the opposite of what the Bells want.

c

The CFP Board Rules of Conduct prohibits a CFP® certificant from doing which of the following activities? Borrowing money from a client who is not in the business of lending money Loaning money to a client Establishing relationships that might compromise the planner's objectivity (i.e., create a conflict of interest) Omitting facts where that disclosure would mislead clients A) I and IV only B) I and II only C) I, II, III, and IV D) I, II, and IV only Explanation A CFP® certificant may not loan money to clients or borrow money from clients (unless the client is in the lending business) and may not mislead a client. The CFP Board Code of Ethics does not prohibit establishing relationships that might compromise the planner's objectivity; however, it does require disclosure of these relationships. CFP® certificants obviously may not make misleading statements, but also may not fail to disclose information that would prevent misleading a client.

c

The Chapter 14 zero valuation rules focus on proper valuation of assets at the time of transfer for purposes of determining gift tax. Which of the following statements regarding the Chapter 14 valuation rules is CORRECT? An estate freeze involving the intrafamily transfer of corporate stock or partnership interests generally results in an immediate gift tax based on the entire value of the business held by the senior family member. In the case of buy-sell agreements, the Chapter 14 rules do not apply to transfers between nonfamily members. A) Neither I nor II B) II only C) I only D) Both I and II Explanation Statement II is incorrect because in the case of buy-sell agreements, the Chapter 14 valuation rules apply to transfers between nonfamily members as well as transfers between family members.

c

The estate tax applicable credit amount in 2020 effectively shelters what amount of taxable property transfers from the estate tax? A) $4,577,800 B) $15,000 C) $11,580,000 D) $1,000,000 Explanation The answer is $11,580,000. The 2020 applicable credit amount of $4,577,800 shelters up to $11,580,000 in taxable transfers at death (assuming no adjusted taxable gifts were made during the person's lifetime).

c

The preliminary computation of Dorothy Rohler's estate tax is as follows: Dorothy Rohler Estate Tax3,000 Shares of Dart, Inc. (1/3 of total outstanding shares)$9,200,000Real estate3,000,000Remainder of Rohler Estate1,785,000Gross Estate$13,985,000Administrative expenses(75,000)Debts of the decedent(75,000)Adjusted gross estate$13,835,000Charitable deduction$220,000Marital deduction0Taxable Estate$13,615,000 Dorothy bequeathed her shares of Dart Inc. to her two nephews. They currently participate in the business and plan to hold the shares indefinitely. She made no lifetime taxable gifts. Assuming death in 2020, does the Rohler estate qualify to use the Section 6166 installment method of paying the estate tax generated by the Dart Inc. shares? A) Yes, because the value of the business exceeds 35% of the taxable estate. B) No, because the stock included in the estate fails to exceed 50% of the taxable estate. C) Yes, because the value of the business exceeds 35% of the adjusted gross estate. D) Yes, because the stock included in the estate exceeds 20% of the company's stock. Explanation The percentage test for Section 6166 is whether the value of the stock held by the estate exceeds 35% of the adjusted gross estate, not the taxable estate. Therefore, 35% of $13,835,000 = $4,842,250. The percentage of the company stock owned by the estate is relevant only when aggregating stock ownership in two or more companies to meet the 35% (not 50%) requirement.

c

This year, Rhonda makes present interest gifts to five different donees and makes future interest gifts to three other donees. Rhonda is entitled to how many gift tax annual exclusions this year? A) One B) Three C) Five D) Eight Explanation A separate annual exclusion applies to each donee who receives a gift of a present interest, so Rhonda is entitled to five annual exclusions. Each annual exclusion amount covers up to $15,000 of present interest gifts per recipient in 2019.

c

Tommy wishes to transfer his house valued at $60,000 to his son, Bob, in trust for Bob's lifetime, with the remainder to Bob's children. Tommy plans to occupy the house until his death. Which of the following statements are CORRECT? The gift to Bob is eligible for the annual exclusion. The gift of the remainder interest to Bob's children is eligible for the annual exclusion. Tommy has not made a gift. The transfer of the house is a future interest gift. A) III and IV B) I, II, III, and IV C) IV only D) II only Explanation Neither the gift to Bob nor the gift of the remainder interest gift to Bob's children is eligible for the annual exclusion. A present interest gift that qualifies for the annual exclusion generally transfers an immediate right to possession or enjoyment of the property or property interest to the donee. In this case, the gift is one of a future interest, both to Bob and his children.

c

Trusts are one method often used to leave property to care for a minor child from a prior marriage. Which of the following are correct statements about the advantages and disadvantages of such a technique? Such trusts can be either inter vivos or testamentary trusts. A conservator must be appointed for the minor child. The trust will be subject to the continuing supervision of a local court. Such trusts can continue for the minor child's benefit after he or she reaches the age of majority. A) II, III, and IV B) II and III C) I and IV D) I, II, and IV Explanation It is not necessary for a conservator to be appointed, as the trustee is under a fiduciary duty to comply with the trust terms and to manage and spend the trust funds in the best interests of the minor child. One of the biggest advantages of using such a trust is that the hassle and expense of court proceedings can be avoided.

c

Trusts created under a will are examples of which type of trust? A) Inter vivos trust B) Totten trust C) Testamentary trust D) Revocable living trust (RLT) Explanation A trust that is created under a will and that does not take effect until the testator's death is a testamentary trust.

c

Two sisters, Donna and Mary, own a home as joint tenants with right of survivorship (JTWROS). They purchased the home 10 years ago for $100,000. Donna contributed $40,000 and Mary contributed $60,000. Today, the home is valued at $180,000. If Donna died today, what amount would be included in her gross estate? A) $40,000 B) $90,000 C) $72,000 D) $80,000 Explanation For JTWROS property owned by non-spouses, the amount included in the gross estate is based on the decedent's percentage of contribution to the original cost of the property. Donna contributed 40% of the purchase price of the home, so 40% of its value is included in her gross estate ($180,000 × 40% = $72,000).

c

Wanda Skaggs would like to avoid the time, expense, and inconvenience of probate. She also would like to reduce the size of her gross estate. Which one of the following strategies would be most likely to meet all of Wanda's goals? A) Changing the deed to her personal residence so that she has a retained life estate, with the remainder going to her children B) Converting her bank accounts to payable on death (P.O.D.) accounts C) Transferring property to her son, Chauncy, in return for his promise to pay her a fixed annuity for the rest of her life based on the value of the property D) Retitling all of her solely owned property with her children and herself as tenants in common Explanation A single life private annuity will remove the asset involved from the seller's gross estate. Both the sold asset and the annuity will avoid probate because the asset is no longer owned by the seller, and there is no interest in the annuity to transfer after Wanda's death. Retitling property as tenancy in common would reduce her gross estate, but her remaining fractional interest would still have to go through probate. The remaining options would allow her estate to avoid probate, but they would not reduce the size of Wanda's gross estate, as she will have retained a right over the property that invokes the transfer sections.

c

When Alex dies, his will leaves his entire estate to his widow, Grace. The will provides that if Grace does not survive him by 60 days, the estate will pass to their children. Grace has substantial assets of her own and would prefer that Alex's property pass directly to their children now instead of to her. Which of the following postmortem estate planning techniques would enable Grace to achieve her wishes? A) Reverse QTIP election B) Partial QTIP election C) Qualified disclaimer D) Election against the will Explanation A qualified disclaimer will achieve Grace's objective because it constitutes a refusal by Grace to accept the property and allows the property to pass directly to her children. A partial QTIP election is used to qualify a portion of a qualified terminable interest for the marital deduction, and a reverse QTIP election is used to permit the decedent's estate to use the generation-skipping transfer tax exemption for QTIP property. An election against the will allows a surviving spouse who has been disinherited under the deceased spouse's will to claim a share of the deceased spouse's estate.

c

When a married couple divorces, one estate planning issue that may arise is allowing one spouse access to retirement benefits earned by the other spouse. This goal can be achieved by use of a qualified domestic relations order (QDRO). Which of the following statements correctly identifies an advantage or a disadvantage of a QDRO? An advantage of using a QDRO is that it can be used to satisfy the plan participant's obligation to pay child support, alimony, or marital property rights. An advantage of using a QDRO is that it can force the plan administrator to accelerate the distribution of benefits to the alternate payee. A disadvantage of using a QDRO is that distributions to an alternate payee who is the participant's spouse are taxable to him or her to the same extent they would have been to the participant. An advantage of using a QDRO is that if the participant's spouse is the alternate payee, his or her interest may be distributed to an IRA owned by the spouse. A) I and III only B) II, III, and IV only C) I, III, and IV only D) II and IV only Explanation A QDRO cannot be used to accelerate distribution of plan benefits (option II). All other statements are true.

c

Which of the following actions should an executor take when developing a cash flow plan for an estate? Determine which assets are the best candidates for sale if it appears necessary to sell estate property Begin planning a strategy to make up any projected cash deficit Account for the possibility of unexpected expenses Explore ways of reducing the estate's cash needs by making special elections under the estate tax laws A) I, III, and IV B) II only C) I, II, III, and IV D) I and III Explanation An executor should take all of these actions when developing a cash flow plan for an estate.

c

Which of the following are CORRECT statements about the filing requirements and/or the responsibility for payment as they relate to federal transfer taxes? A donee can be held responsible for paying the gift tax on a transfer that she has received if the IRS cannot collect from the donor. A federal estate tax return need not be filed unless an estate owes estate taxes in excess of the unified credit. The beneficiary is responsible for paying the generation-skipping transfer tax on a distribution from a trust and must file a tax form. A federal gift tax return need not be filed for a gift that is split with the donor's spouse. A) I, II, and IV B) II and III C) I and III D) III only Explanation Statement II is false as an estate tax return is always required. Statement IV is false because the exact opposite is true: A gift tax return, while not always required, is required when gift splitting is used.

c

Which of the following are CORRECT statements concerning a buy-sell (business continuation) agreement funded with life insurance? The business is a party to the contract if a stock (entity) redemption plan is used. With a cross-purchase plan, the surviving shareholder's new cost basis is equivalent to his or her old cost basis plus the life insurance proceeds used to purchase the deceased shareholder's interest at the price established by the agreement. A cross-purchase plan is preferable to a stock (entity) redemption plan when all shareholders are in a higher income tax bracket than the corporation. With a stock (entity) redemption plan, premiums paid by the corporation on life insurance to fund the purchase are taxable income to the shareholders because they will eventually benefit. Under a stock (entity) redemption plan, the value of the deceased's business interest is included in his or her gross estate, while the life insurance proceeds used to purchase his or her business interest are excluded. A) II and IV B) II, III, and IV C) I, II, and V D) I and III Explanation When shareholders are in a higher income tax bracket than the corporation, a stock (entity) redemption, not a cross-purchase plan, is preferable because payment of premiums by the corporation provides a greater economic benefit to the shareholders than payment of salaries and/or dividends to them which will be taxed at a higher rate before they are used to pay premiums. Under a stock redemption plan, the premiums are paid by the corporation with after-tax dollars (i.e., premiums are not deductible to the corporation) and therefore are not taxable income to the shareholders.

c

Which of the following are CORRECT statements regarding the characteristics and purpose of a "no contest" clause? This type of clause is used when a close family member is disinherited in a will. This type of clause imposes a penalty for contesting the validity of provisions in a will. This type of clause prohibits contesting the validity of provisions in a will. This type of clause prevents a person from contesting a will if he or she has feloniously caused the death of the testator. A) II and IV B) III and IV C) I and II D) I, III, and IV Explanation Statement III is false because a no contest clause is designed to discourage a will contest, but cannot prevent it. A person who has feloniously caused the death of the testator is prohibited from collecting from the will by statute, which has nothing to do with a no contest clause.

c

Which of the following are advantages of using an individual fiduciary rather than an institutional fiduciary? An individual fiduciary is less likely to charge a fee for his or her services. An individual fiduciary usually will have more knowledge of, and sensitivity to, the beneficiaries. An individual fiduciary is likely to be more impartial and have fewer conflicts of interest with the beneficiaries. An individual fiduciary is more likely to be able to serve for an extended period of time. A) I, II, and IV only B) III and IV only C) I and II only D) II and III only Explanation An institutional fiduciary would be more likely to charge a fee since acting as a fiduciary is part of its business. An institutional fiduciary is also less likely to be familiar with the needs of the beneficiaries than would an individual family member. An individual fiduciary may know one beneficiary better than another and may be less impartial as a result. If the individual fiduciary is also a beneficiary (which is not uncommon), he or she is more likely to have a conflict of interest. An individual fiduciary may be unable or unwilling to serve because of changed circumstances in his or her life. This is less likely to be a problem with an institutional fiduciary.

c

Which of the following are characteristics of a gift-leaseback? The property involved in the transaction usually is a business-related asset. The property involved in the transaction usually is gifted to a donee in a lower marginal income tax bracket. The donor retains security in the gifted property. The lease payments made by the donor to the donee are considered additional gifts. A) III and IV B) II and III C) I and II D) I and IV Explanation Statement II is false because in a gift-leaseback the donor relinquishes security and control of the gifted property. Statement IV is false because lease payments are income to the donee, not additional gifts.

c

Which of the following are characteristics or consequences of establishing a T.O.D. (transfer on death) account? If the named beneficiary is the owner's spouse, the transfer will qualify for the gift tax marital deduction. Assets in an account titled in this manner will avoid probate. Assets in an account titled in this manner will be included in the owner's gross estate. The owner can change the named beneficiary at any time prior to death if still competent. A) III and IV B) I and II C) II, III, and IV D) I and III Explanation Only option I is an incorrect statement. Because titling an account in this manner is not a completed gift, the transfer is not subject to gift tax and does not qualify for the gift tax marital deduction. The beneficiary designation is revocable until death, so the assets in the account are included in the account owner's gross estate and would receive an estate tax marital deduction if the named beneficiary is the owner's spouse at death.

c

Which of the following are factors that a financial planner should monitor for every client? Changes in the client's objectives Changes in the client's marital status Changes in property laws Changes in the amount of lifetime gifts made by the client A) I and II only B) II, III, and IV only C) I, II, III, and IV D) I, III, and IV only Explanation All of these are factors that a financial planner should monitor for every client.

c

Which of the following are reasons business succession planning is complex and challenging? Determining the value of the business may be difficult. It may be difficult to find a buyer who has the resources necessary to purchase the business. Family issues may be involved if the business is closely-held or family owned. A new owner may have difficulty adapting to the idiosyncrasies of the owner's business. A) I and II B) III and IV C) I, II, III, and IV D) II and IV Explanation Business succession planning is complex and challenging for all of these reasons.

c

Which of the following characteristics of the unified transfer tax system are common to both testamentary transfers and lifetime gifts? A single applicable credit amount that offsets taxes due A step-up in income tax cost basis when the transfer is made An annual exclusion per donee, per year from taxation A cumulative calculation using the same progressive tax rate schedule A) I, II, III, and IV B) I and III C) I and IV D) II and IV Explanation Statement II is false because only testamentary transfers receive a step-up in basis. Statement III is false because the annual exclusion applies only to lifetime gifts.

c

Which of the following concepts associated with liquidity planning, matched with its description, is NOT correct? A) Forced liquidation—estate shrinkage that occurs when the decedent's personal representative must sell estate assets, usually at less than market value, to make up for a cash deficiency. B) Estate administrative expenses—the money that the decedent's personal representative must spend to collect the decedent's assets; pay claims of the estate; and distribute the remaining assets to the decedent's devisees, legatees, or heirs. C) Estate liquidity—the decrease in the value of a decedent's estate from the time of the decedent's death until the time of the ultimate distribution to the decedent's devisees, legatees, and heirs D) Estate debts—the money that the decedent's personal representative must spend to pay the decedent's lifetime obligations that had not been paid at the time of the decedent's death. Explanation Estate liquidity is a term used to describe the current or future potential of an estate to meet its cash requirements. The decrease in the value of a decedent's estate from the time of the decedent's death until the time of the ultimate distribution to the decedent's devisees, legatees, and heirs describes the concept of estate shrinkage.

c

Which of the following correctly state characteristics of the laws of intestate succession (intestacy)? If a decedent has a surviving spouse, the spouse is certain to get some part of the intestate estate, assuming there are remaining assets after payment of debts and taxes, if the felonious homicide statutes do not apply. If a decedent has a surviving spouse and children, the children are more likely to receive some part of the intestate estate if they are children from a prior marriage rather than from the decedent's final marriage. If a decedent has any heirs, no matter how remote, the intestate property will not escheat to the state. Heirs who have identical degrees of kinship to the decedent will receive equal shares. A) III and IV B) I, II, and III C) I and II D) II and III Explanation Intestacy statutes tend to give intestate property to children from a prior marriage since it is uncertain that the surviving spouse would leave any part of the decedent's property to such children. Some states prohibit remote heirs from inheriting by way of the intestacy statutes. Whether heirs who have identical degrees of kinship to the decedent inherit equally will depend upon whether the intestacy statutes distribute by representation (or per stirpes) or per capita at each generation.

c

Which of the following estate planning objectives can be accomplished through a will? Creating a presumption of survivorship Establishing a priority for eliminating or reducing bequests if the estate has insufficient assets Avoiding probate of estate assets Naming a residuary beneficiary to take all assets that remain after specific bequests are allocated A) II and IV B) I and III C) I, II, and IV D) II and III Explanation Only option III cannot be accomplished through a will because the will itself must be probated, as well as any estate assets that are distributed by the will.

c

Which of the following estate planning techniques can be used by unmarried cohabitants to reduce estate tax due at the death of the first cohabitant to die? The gift tax annual exclusion The estate tax charitable deduction A qualified domestic trust (QDOT) A qualified personal residence trust (QPRT) A) II and IV B) I and III C) I, II, and IV D) II, III, and IV Explanation Statement III is false because a QDOT is used only when a U.S. citizen spouse wishes to leave assets to a nonresident alien spouse.

c

Which of the following is NOT an advantage of making a lifetime gift? A) It shifts income from the gifted asset to the donee. B) It diminishes the size of the gross estate. C) It excludes gift tax paid on the gift from the donor's gross estate regardless of how soon the donor dies after making the gift. D) It provides personal satisfaction. Explanation Gift tax taxes paid out of pocket are excluded from the donor's gross estate only if the donor's date of death is not within three years of the gift. In order for a donee to pay gift taxes out of pocket (instead of using the applicable gift tax credit), the donor must have already made adjusted taxable gifts of more than the exemption equivalent.

c

Which of the following is a characteristic of gifts to a noncitizen spouse? A) Must always be made in the form of a qualified domestic trust B) Qualify for the unlimited marital deduction C) Gifts exceeding super annual exclusion amount will reduce the donor spouse's lifetime applicable credit amount (unified credit) or result in a gift tax if this credit has already been used D) Can only be made when the noncitizen spouse becomes a US citizen before the death of the US citizen spouse Explanation Gifts exceeding super annual exclusion amount will reduce the donor spouse's lifetime applicable credit amount (unified credit), or result in a gift tax if this credit has already been used.

c

Which of the following is a court-supervised arrangement in which a person is appointed to manage the personal care and well-being of another? A) Trust B) Power of attorney C) Guardianship D) Conservatorship Explanation A guardianship is a fiduciary relationship created by law for the purpose of enabling a person (known as the guardian) to manage the personal care and well-being of another (known as the ward). A conservatorship is a fiduciary relationship concerned with managing the ward's property. In other words, you guard people and you conserve money/assets. Trusts and powers of attorney are private documents that do not involve court supervision.

c

Which of the following is a requirement of a qualified disclaimer? A) It is made either in writing or orally B) It directs that the interest pass to someone else C) It is an irrevocable and unqualified written refusal to accept the interest D) It can be made after the disclaimant has already accepted any interest in the benefits from the property Explanation For a disclaimer to be qualified for tax purposes, it must be irrevocable and also in writing. If the disclaimer is qualified, the bequest or disposition will be treated as though a gift was never made. The disclaimant must not have previously accepted any interest in the benefits from the property.

c

Which of the following is an advantage of spouses holding property jointly with right of survivorship? Total administration expenses and attorney fees may be reduced because the property avoids probate at the death of the first spouse. The surviving spouse now has a basis equal to the fair market value of the property at death of first spouse. A) Neither I nor II. B) Both I and II. C) I only. D) II only. Explanation However the basis in the hands of the surviving spouse is the sum of her basis plus 1/2 the fair market value at date of the decedent's death.

c

Which of the following laws were designed to provide for the needs of a surviving spouse or dependent children? Homestead statute Family allowance statute Family benefits statute A) I, II, and III B) II only C) I and II D) II and III Explanation Statement III is false because there is no such thing as a family benefits statute.

c

Which of the following postmortem planning techniques require the decedent to have executed, prior to death, a document that is enforceable after death? An election that property in a QTIP trust qualify for the marital deduction A Section 303 stock redemption election A disclaimer trust that will allow a surviving spouse to disclaim property and give her an interest in the property as a beneficiary of the trust An election to waive personal representative (executor) fees A) I only B) I, II, and III C) I and III D) III and IV Explanation This recognizes that for option I, a QTIP trust must have been established, and for option III, the decedent must have established a disclaimer trust. Although an agreement for the redemption of stock under Section 303 (option II) may be entered into prior to death, such an agreement is not required. Section 303 requires only that the stock meet certain percentage requirements and that the business be willing to redeem the stock. Option IV requires only a signed waiver that is executed after the date of death.

c

Which of the following questions would be appropriate in planning with life insurance? A) How would a soon-to-be-issued policy be owned? B) How is the policy currently owned? C) All of these D) Is the amount of insurance coverage adequate? Explanation All of the these would be appropriate as among the most important questions to be initially raised.

c

Which of the following results from Arthur creating a charitable remainder annuity trust (CRAT) or charitable remainder unitrust (CRUT) and donating his ownership of Bell's Animal Care Center to such a trust, as opposed to a direct sale of the business? He avoids capital gains tax upon the transfer of the business to the trust. He gets an immediate charitable income tax deduction. He may select a joint life annuity. He can receive annuity income. A) II and IV B) I, II, and IV C) I, II, III, and IV D) II only Explanation He avoids capital gains tax upon the transfer of the business to the trust. He gets an immediate charitable income tax deduction for transferring assets to the CRAT or CRUT (subject to the rules on the type of charity and AGI limits). He is able to get a retirement annuity from the CRAT or CRUT. He can choose a joint life expectancy for either the CRAT or CRUT.

c

Which of the following statements about the gift tax charitable deduction is NOT correct? A) A gift of a partial interest will qualify for a charitable deduction only if it meets the requirements of the Internal Revenue Code and IRS regulations. B) A charitable gift tax deduction is given only for the portion of the contribution in excess of any value the donor receives from the charity. C) The charitable gift tax deduction is limited by the type of property gifted, the type of charitable donee, and the donor's adjusted gross income (AGI). D) To qualify for a charitable deduction, a gift must be of cash or property. Explanation The charitable gift tax deduction is unlimited for qualifying transfers. For income tax purposes, the charitable deduction has limits based on the type of charitable donee, the type of property gifted, and the donor's AGI (adjusted gross income).

c

Which of the following statements concerning gifts of appreciated property are CORRECT? The donor's holding period carries over to the donee. Generally, the donor's basis carries over to the donee. If the donor paid gift tax on the gift, the donee's basis is increased by a portion of the gift tax paid. A) I and III B) II and III C) I, II, and III D) II only Explanation All of these statements are correct.

c

Which of the following statements is CORRECT concerning income earned by spouses in a community property state? A) Income earned by each spouse prior to marriage is considered community property. B) Income earned by each spouse prior to and after marriage is considered community property. C) Income earned by each spouse after marriage is considered community property. D) Income earned by each spouse after marriage is considered community property only if it is commingled. Explanation Even though earned by only one spouse, such earnings are considered community property.

c

Which of the following statements is CORRECT regarding a payable on death (P.O.D.) account used as a will substitute? Use of a P.O.D. designation is a completed gift, but is entitled to an annual exclusion for each named beneficiary. The named beneficiary can transfer up to half of the assets in the account. The account assets will be included in the account owner's gross estate. The account assets will be transferred outside of probate. A) I, III, and IV B) I and III C) III and IV D) II only Explanation A P.O.D. designation is revocable. Therefore, no completed gift is made, and the named beneficiary has no use of the account assets until the account owner dies. The account assets are included in the owner's gross estate, but not the owner's probate estate.

c

Which of the following statements is CORRECT regarding the purpose and characteristics of a nuncupative will? A nuncupative will is one whose material provisions are written entirely in the testator's handwriting. A nuncupative will requires at least one witness. State law may limit the types and amounts of property that may be distributed by this form of will. This form of will is recognized in most states. A) III and IV B) I, II, and IV C) II and III D) I and II Explanation A nuncupative will is an oral will. This form of will is recognized only in a few states, which often place limits on the types and amounts of property that can be transferred using this method. At least one witness is necessary to hear the oral statements.

c

Which of the following statements regarding Medicaid planning and eligibility is(are) CORRECT? There are 2 types of assets for determining Medicaid eligibility—countable assets and exempt assets. Exempt assets are not counted in determining a person's eligibility for Medicaid. A married couple's primary residence is considered to be an exempt asset. A) II and III B) I and II C) I, II, and III D) I only Explanation All of these statements are correct.

c

Which of the following statements regarding QTIP trusts are CORRECT? QTIP trusts allow a terminable interest to be left to the surviving spouse and still qualify for the estate tax marital deduction. Assets in a QTIP trust are usually included in the gross estate of the second spouse to die. QTIP trusts are useful when the first spouse to die has children from a prior marriage. A) I and III B) II and III C) I, II, and III D) II only Explanation All of these statements are correct. The QTIP is included in the estate of the second spouse to die at the same percentage as the first spouse took as a marital deduction. If 100% of the QTIP was taken as a marital deduction when the first spouse dies, then 100% of the QTIP is included in the second spouse to die's gross estate. If the first spouse's estate only took a martial deduction for 70% of the QTIP, then only 70% of the QTIP's value when the second spouse to die passes away is included in the estate of the second spouse to die. LO 2.2.1

c

Which of the following statements regarding a QTIP election is CORRECT? The QTIP election by a decedent's executor is voluntary. If the executor makes a QTIP election, the estate tax is increased in the decedent's estate. A) II only B) Both I and II C) I only D) Neither I nor II Explanation Statement II is incorrect. If the executor makes the QTIP election, the estate tax is decreased in the decedent's estate because more property qualifies for the marital deduction.

c

Which of the following statements regarding a springing power of attorney is CORRECT? With a springing power of attorney, the attorney-in-fact's authority to act is delayed until the principal actually becomes incapacitated or incompetent. A springing power of attorney can be used in planning for the principal's possible incapacity. A) Neither I nor II B) II only C) Both I and II D) I only Explanation Both of these statements correctly define a springing power of attorney.

c

Which of the following statements regarding gift splitting are CORRECT? The annual gift tax exclusion allows spouses who consent to split their gifts to transfer up to $30,000 (for 2020) to any one person during any calendar year without gift tax liability, if the gifts are of a present interest. To qualify for gift splitting, a couple must be married at the time the gift is made. For gift tax purposes, spouses must file a joint income tax return to qualify for the gift splitting benefits. Both spouses must consent to the use of gift splitting and at least one gift tax return must be filed. A) II, III, and IV B) I only C) I, II, and IV D) I and II Explanation Spouses do not have to file a joint income tax return to elect gift splitting. The other statements are correct.

c

Which of the following statements regarding guardianships and conservatorships is(are) CORRECT? A guardianship is the same as a conservatorship. Guardianships and conservatorships are both fiduciary relationships. The same person may be both the guardian and conservator of a ward. A) I, II, and III B) II only C) II and III D) I only Explanation Statement I is incorrect because a conservatorship enables a person to manage the ward's property, while a guardianship enables a person to manage the ward's health and personal care.

c

Which of the following statements regarding lifetime gifts are CORRECT? Annual exclusion gifts will escape gift taxation and will not be included in the donor's gross estate. Future appreciation in the value of gifted property will escape estate taxation in the donor's estate. Income from gift property will generally be taxed to the donee for income tax purposes. Generation-skipping transfer taxes do not apply to lifetime gifts. A) II and III B) II, III, and IV C) I, II, and III D) I and III Explanation Only Statement IV is incorrect; the generation-skipping transfer tax (GSTT) does apply to lifetime gifts. Statements I, II, and III are correct.

c

Which of the following statements regarding property owned as joint tenants with right of survivorship (JTWROS) between spouses is CORRECT? The entire value of the property is included in the gross estate of the decedent spouse and the entire value of the property receives a stepped-up basis. One-half of the property is included in the gross estate of the decedent spouse and one-half of the property receives a stepped-up basis. A) I only B) Neither I nor II C) II only D) Both I and II Explanation Statement I is incorrect because only one-half of the property is included in the gross estate of the decedent spouse and only one-half of the property receives a stepped-up basis. The stepped-up basis from the deceased spouse's half is added to the surviving spouse's original basis to determine the surviving spouse's new basis. This is different from community property in which both the deceased spouse and the surviving spouse receive a stepped-up basis.

c

Which of the following statements regarding qualified personal residence trusts (QPRTs) is CORRECT? A QPRT may have an interest in more than one residence. There are no restrictions on who may occupy a residence that is owned by a QPRT. A) II only B) I only C) Neither I nor II D) Both I and II Explanation Neither statement is correct. Statement I is incorrect because a QPRT may have an interest in only one residence. Statement II is incorrect because a residence owned by a QPRT cannot be occupied by anyone other than the grantor and members of the grantor's family.

c

Which of the following statements regarding the annual exclusion for purposes of generation-skipping transfer tax (GSTT) is CORRECT? The annual exclusion amount is $15,000 for 2020. The annual exclusion is allowed for lifetime direct skips. The annual exclusion is allowed for testamentary direct skips. A) II and III B) I, II, and III C) I and II D) I only Explanation Statements I and II are correct. Statement III is incorrect because the annual exclusion is not allowed for testamentary direct skips.

c

Which of the following statements regarding the gift tax lifetime exemption amount (applicable exclusion amount) and applicable credit amount are CORRECT? The gift tax lifetime exemption amount in 2020 is $11,580,000. The gift tax applicable credit amount in 2020 is $4,577,800. A) I only B) II only C) Both I and II D) Neither I nor II Explanation Both of these statements are correct.

c

Which of the following statements regarding the liquidity of an estate is CORRECT? The types of assets that constitute an estate are not an important factor in assessing the estate's liquidity. Lack of liquidity may force an estate to sell assets under disadvantageous conditions. Liquidity is important because cash may be needed to satisfy many obligations. A) III only B) I, II, and III C) II and III D) I and II Explanation Liquidity is defined as the ease with which an asset can be converted to cash, without loss in value; therefore, the type of asset can be very important. An estate with many real estate holdings is less liquid than one heavily invested in government bonds or publicly traded stocks.

c

Which of the following statements regarding the role of guardians in estate planning is(are) CORRECT? A guardian manages the personal care and well-being of the ward. A guardian manages the property of the ward. A) Both I and II B) Neither I nor II C) I only D) II only Explanation Statement II is incorrect because a guardian does not manage the ward's property; this function is performed by a conservator. One way to remember this is that you guard people, but you conserve assets. For example, you conserve water. You guard prisoners.

c

Which of the following statements regarding the use of a buy-sell agreement for liquidity planning purposes for an estate is CORRECT? A) It increases the debts of the decedent. B) It can turn an illiquid estate asset into a liquid asset. C) It removes the closely held business interest from an owner's gross estate. D) It reduces administrative expenses because it keeps an owner's interest from going through probate. Explanation By providing the means for the co-owner(s) to buy the decedent's share of the business, a buy-sell agreement properly funded (usually with life insurance) removes an illiquid asset from the estate.

c

Which of the following statements would avoid probate at the owner's death? Payable on death (P.O.D.) account Tenancy by entirety Funded inter vivos trust Joint tenancy with right of survivorship (JTWROS) A) I and II B) II, III, and IV C) I, II, III, and IV D) I and III Explanation All of these are will substitutes that can be used to avoid probate.

c

Which of the following transfers is a taxable gift? A) John donates $10,000 to his church. B) John pays $25,000 of medical expenses to a hospital for his invalid niece. C) John gives a $20,000 necklace to his fiancée 9 months before they are married. D) John pays the University of South Texas $12,000 in college tuition for his 22-year-old niece. Explanation The necklace is a taxable gift. Because John was not married at the time of the gift, he cannot take advantage of the marital deduction on gifts to his fiancée. Payments of tuition or medical expenses directly to a provider are NOT considered taxable gifts for gift tax purposes. The donation to the church is eligible for the gift tax charitable deduction.

c

Which of the following types of property will be treated as separate property assuming the couple always has lived in a community property state? A) A closely held business interest purchased by one spouse during the marriage B) A coin collection purchased by one spouse during the marriage C) Real estate acquired by one spouse before the marriage D) A tax-exempt bond purchased by one spouse with the after-tax proceeds of her salary Explanation The real estate acquired before the marriage is separate property.

c

Which of the statements about maintaining liquidity of an estate are correct? Premortem liquidity needs are not considered. Postmortem liquidity needs are met primarily with life insurance. A) Neither I nor II B) I only C) II only D) Both I and II Explanation The need for premortem liquidity should be considered to meet the retirement needs of a client. Life insurance is the most likely source of liquidity at the death of a decedent.

c

Which one of the following is a characteristic of the unified tax system that is NOT common to both federal gift taxation and federal estate taxation? A) Availability of the charitable deduction B) Availability of the marital deduction C) Availability of the annual exclusion amount D) Availability of an applicable credit amount Explanation This is the correct answer because the annual exclusion only applies to qualifying lifetime transfers of wealth (i.e., for federal gift taxation).There are no similar exclusions for wealth transfers at death, only deductions for certain transfers and credits that apply against any tax that is due.

c

Which one of the following is a document that designates a trust as the recipient of all property that has not been otherwise disposed of upon the death of the decedent? A) Codicil B) Power of appointment C) Pourover will D) Testamentary trust Explanation The answer is pourover will.

c

Which one of the following statements about the use of a springing durable power of attorney is correct? A) The attorney in fact receives title to the principal's property. B) The attorney in fact has immediate authority to act for the principal. C) The attorney in fact generally must obtain a letter from the principal's physician prior to acting. D) The attorney in fact's authority will survive the principal's death. Explanation The answer is the authority of the attorney in fact under a durable power of attorney will survive the principal's incompetency, but not his or her death. The authority of the attorney in fact is not immediate in a springing durable power of attorney, as it does not "spring into action" until the occurrence of a stated event—usually the principal's incompetency. The attorney in fact does not take title to the principal's property but merely has authority as the principal's agent to deal with the principal's property according to the terms of the instrument. A springing power of attorney becomes effective upon incapacity, which generally must be confirmed by two physicians.

c

Which one of the following statements correctly describes the characteristics of an entity purchase buy-sell agreement? A) The replacement cost of life insurance on the lives of the other owners used to fund the agreement will be included in the gross estate of the first owner to die. B) Each business interest owner (and/or her estate) has an obligation to sell or offer to sell her interest to the other business interest owners when specified events occur. C) The purchase price established for an owner's interest in the agreement will be accepted for transfer tax purposes by the IRS if more than 50% of the value of the property subject to the agreement is owned directly or indirectly by individuals who are not members of the transferor's family. D) Each business interest owner purchases a life insurance policy on the life of every other business interest owner to be able to meet her obligation to purchase her share of the interest of the other owner or owners. Explanation The purchase price established for an owner's interest in the agreement will be accepted for transfer tax purposes by the IRS if more than 50% of the value of the property subject to the agreement is owned directly or indirectly by individuals who are not members of the transferor's family. The Chapter 14 rules will not apply as the statutory exception to this section will be met. In an entity redemption agreement, the business entity contracts with each business owner to buy or redeem each owner's interest when specified events occur. The remaining answer choices would be true statements for a cross-purchase buy-sell agreement.

c

Which one of the following statements describing characteristics, strengths, weaknesses, advantages, or disadvantages of a durable general power of attorney is NOT correct? A) The power of attorney must be in writing. B) A disadvantage is that the authority of the agent does not survive the principal's death. C) It takes effect only after the principal is incompetent. D) An advantage is that the powers given to the agent can be as broad or as narrow as the principal wants. Explanation This is the only statement that is not true. A durable power of attorney takes effect upon its execution and delivery to the agent unless a different effective date is expressly stated in the instrument. A durable power of attorney survives the principal's incompetency, but not the principal's death.

c

Which one of the following statements regarding a charitable remainder trust (CRT) is CORRECT? A) The remainder interest in a CRT passes to a noncharitable beneficiary. B) A grantor who establishes a CRT is free to revoke the trust at any time. C) A grantor who establishes a charitable remainder unitrust (CRUT) is eligible for an income tax deduction in the year the trust is established. D) The annuity payment received from a charitable remainder annuity trust (CRAT) fluctuates each year with the value of the trust assets, providing a potential hedge against inflation. Explanation The grantor of a CRUT is eligible for an immediate income tax deduction based on the present value of the remainder interest that will pass to charity. A CRT must be irrevocable. The annuity payment from a CRAT is fixed.

c

Which one of the following statements regarding a conservatorship is not true? A) The conservator will likely have to file reports with a court. B) The conservator is subject to the jurisdiction of a court. C) The conservator has authority to decide where the ward will live. D) The conservator may have to post a bond. Explanation A conservator is almost always a fiduciary who is in charge of the ward's finances, but does not have authority over the body of the ward (e.g., deciding where the ward will live). This authority is given to a guardian. A conservator is appointed by the court which retains jurisdiction over both the conservator and guardian, requiring periodic reports from each, and may require a performance bond.

c

Which one of the following statements regarding the purpose and basic features of the generation-skipping transfer tax (GSTT) is CORRECT? A) If a transferor funds a trust that has more than one beneficiary, and at least one of the beneficiaries is a skip party, the transferor will be deemed to have made an indirect skip. B) A child of the transferor is known as a skip party in a direct skip transaction, since the child is in the generation that was skipped over in the transfer of wealth. C) The GSTT exemption allows each person to make generation-skipping transfers in the applicable amount, during life or at death, without having to pay a GSTT. D) The purpose of the GSTT is to take the place of the gift tax or estate tax with a tax at the highest estate tax rate at the time of transfer on the value of any wealth that is not transferred to the generation immediately below that of the transferor. Explanation The GSTT exemption allows a transferor to escape paying GSTT on taxable transfers up to the exemption amount. The GSTT exemption equals the estate tax applicable exclusion amount in the year of the transfer. The GSTT is in addition to (not a replacement of) the gift or estate tax that is also due on the same transfer. Any person who is in a generation that is less than two generations younger than the transferor is known as a nonskip party. The transaction will be deemed to be an indirect skip only if at least one current beneficiary of the trust is a nonskip party (not a skip party).

c

Which statement regarding bank accounts owned jointly with right of survivorship (JTWROS) is CORRECT? A) There is a gift whenever a person uses personal funds to open a joint bank account. B) A gift is made upon the creation of the donee's interest. C) A gift is made when the noncontributing joint owner (the donee) makes withdrawals. D) There are no gift tax consequences with JTWROS accounts. Explanation There is no gift when a person uses personal funds to open a joint bank account. There is a gift when the other joint owner (the donee) makes a withdrawal—the gift is the amount withdrawn.

c

You are a CFP® Certificant. A client has come to you for assistance. You should inform the client of which of the following? You can provide the client with "fill-in-the-blank" durable power of attorney forms for both finances and health care, as well as living wills, and help the client complete and execute these documents. You can be involved in data gathering, identifying goals, and identifying possible weaknesses and problem areas in the client's current situation. Your primary role will be working with and coordinating other financial planning professionals. You can review the client's current documents to interpret the contents and indicate what the legal implications of the document are for the client. A) I only B) II, III, and IV C) II and III D) III only Explanation Both II and III are legitimate and accepted roles of an educated financial planner (such as a CFP®) in the process. A financial planner is not legally or ethically prohibited from assisting a client with his or her estate plan but cannot advise a client regarding the content of the documents mentioned in Statement I. Interpreting the contents of a client's documents and informing him/her of the legal implications of those documents is not part of the financial planner's role because it usually involves some interpretation of state law, which is considered the unauthorized practice of law.

c

You are a CFP® certificant with ABC Financial Solutions. A client has come to you for estate planning assistance. You should inform the client of which of the following? You cannot ethically provide the client with any estate planning assistance and must refer the case in its entirety to an attorney. You can be involved in data gathering, identifying estate planning goals, and identifying possible weaknesses and problem areas in the client's current situation. Your role will be working with and coordinating specialists such as attorneys, accountants, and trust officers whose expertise will be necessary to analyze tax and legal implications of suggested actions and to draft needed documents. You can review the client's current estate planning documents to interpret the contents and indicate what the legal implications of the document are for the client. A) II, III, and IV B) III only C) II and III D) I only Explanation Both statements II and III are legitimate and accepted roles of a non-attorney financial planner in the estate planning process. The other answers are incorrect for various reasons. A financial planner is not legally or ethically prohibited from assisting a client with his or her estate plan. Interpreting the contents of a client's estate planning documents and informing him or her of the legal implications of those documents is not part of the non-attorney financial planner's role in estate planning since it usually involves interpretation of state law, which is considered the unauthorized practice of law.

c

You are meeting with your client, Beatrice, to review her property and value it to estimate her potential gross estate. Included in her list of property owned are the following assets: Fifty shares of AT&T stock A whole life policy with her spouse as the insured and Beatrice as the primary beneficiary, which is still in premium pay status A term life policy provided by her employer, with Beatrice as the insured and her spouse as the primary beneficiary Five hundred shares of Rob Roy, Inc., a closely held corporation Of the following statements to make to Beatrice about the valuation of property includible in her gross estate, which would be CORRECT? The AT&T stock will be entitled to a minority discount. The whole life insurance policy will be valued at its replacement cost, as measured by its interpolated terminal reserve plus any unearned premiums. The term life policy will be valued at the cost of a policy from the issuing company based on her age at the time of her death. The closely held stock may qualify for a lack of marketability discount. A) II only B) I, II, and IV C) II and IV D) I and III Explanation Option I is false because only closely held, not publicly traded, stock can qualify for a minority discount. Option III states the gift tax valuation rule for a paid-up single premium policy; a term life policy on the decedent's life will be valued at the death benefit since the insured is deemed to have died.

c

Your client, a widow, has made the following lifetime gifts to her son in an effort to reduce the size of her gross estate: YearGiftTaxable Gift2012$323,000$310,0002016$254,000$240,0002020$465,000$450,000 If she used her applicable credit to offset gift tax liability on the gifts, what amount of applicable credit remains available to your client for gifts in 2020? A) $0 B) $345,800 C) $4,232,000 D) $4,505,800 Explanation The total taxable gifts come to $1,000,000 and tax on the first $1,000,000 is $345,800. Subtract that from the 2020 applicable credit of $4,577,800 and the result is $4,232,000.

c

Your client, age 65, has a gross estate valued at $18.7 million. He and his second wife have two teenage children. In addition, your client has two children from his first marriage who are in their mid-30s. His objectives are to: leave an income stream and a portion of his estate to his current wife; leave a portion of his estate in trust for the teenage children from his current marriage; ensure that the children from his first marriage receive a portion of his estate; and reduce his federal estate tax liability. Which one of the following transfers is most appropriate for achieving the client's objectives? A) Family bypass trust equal to the applicable exclusion amount for the teenage children, with the remainder in a marital trust for his current wife B) Outright gift of two-thirds of the estate to his current wife, with the remainder passing to the teenage children in a nonmarital trust C) Combination A-B-C (power of appointment, family bypass, and QTIP) trust arrangement D) Outright transfer of the entire estate into a power of appointment trust Explanation This is the only option that achieves all of the client's goals. The family bypass trust and the outright gift options leave out his children from his first marriage. There is no guarantee that whomever has the power of appointment in a power of appointment trust will leave anything to the other parties.

c

A grandfather is considering making gifts to his grandchildren in 2020. Assuming the grandfather has made no previous generation-skipping transfers, the generation-skipping transfer tax (GSTT) lifetime exemption amount available to the grandfather in 2020 is A) $1,000,000. B) $15,000. C) $4,577,800. D) $11,580,000. Explanation The GSTT lifetime exemption amount for 2020 is $11,580,000.

d

A married couple owns a small business and would like to purchase a life insurance policy to ensure they have liquidity to pay any estate administration expenses and continue to operate the business when one of them dies. They would like to spend the least amount of money on premium payments while meeting their goals for the policy. Which of the following life insurance policies would meet their goals? A) A second-to-die policy B) A survivorship life policy C) An individual policy on each spouse D) A first-to-die policy Explanation A first-to-die policy will meet their goals because a single first-to-die policy will be less expensive than two individual policies, and it will provide cash upon the first death. A second-to-die policy and a survivorship policy are essentially the same thing and do not pay until the second death.

d

A married couple resides in a community property state. Their community property consists of real property with an adjusted basis of $90,000 and a fair market value of $300,000, as well as other property with an adjusted basis of $50,000 and a fair market value of $20,000. Spouse 1 died this year and left his entire estate to Spouse 2. What is Spouse 2's adjusted basis in the real property and other property after Spouse 1's death? RealOther A) $195,000$50,000 B) $195,000$35,000 C) $90,000$50,000 D) $300,000$20,000 Explanation Both the decedent's and surviving spouse's shares of the community property receive a new basis equal to the fair market value on the date of the decedent's death. The real property's basis is stepped up. Notice that the other property's basis is less than its original adjusted basis, so getting the date of death basis actually results in a stepped-down basis for the other property.

d

Alice Greenlee, a widow, would like to reduce the value of her probate estate. She is retired and needs to retain all the income from her assets. She wants all of her estate to go to her daughter, Bonnie, and son, Charles, in equal shares, while minimizing the legal and administrative costs. She currently has the following property: a personal residence, valued at $130,000 rental real estate, valued at $105,000 a stock portfolio, valued at $70,000 pension benefits, valued at $270,000 Given Alice's situation and objectives, which one of the following will substitutes would not be appropriate? A) Placing the stock portfolio in a living revocable trust with Bonnie and Charles as remainder beneficiaries B) Naming Bonnie and Charles equal beneficiaries of her pension benefits at her death C) A retained life estate in the personal residence, with the remainder to Bonnie and Charles D) Changing the title of the rental real estate so that Alice, Bonnie, and Charles are joint tenants Explanation All answers place the corresponding asset in a will substitute that will avoid probate of the asset. Alice needs to retain the income from the rental real estate. Placing the real estate in joint tenancy would avoid probate because Bonnie and Charles would receive the property by right of survivorship, but Alice would be legally entitled to only one-third of the income. A better solution would be either to transfer the real estate to a living revocable trust or to retain a life estate in it, with the remainder to the children. In either case, the value would be in Alice's gross estate (but not in her probate estate).

d

Alice is interested in making deductible charitable contributions, but she does not want to incur the expense of drafting a private trust agreement. Which of the following planning techniques will meet her needs? Charitable remainder trust Charitable gift annuity Charitable lead trust Pooled income fund A) I and IV B) I, II, and IV C) II and III D) II and IV Explanation Charitable gift annuities and pooled income funds do not involve the use of private trust agreements. Statements I and III are incorrect because charitable remainder trusts and charitable lead trusts involve the use of private trust agreements.

d

All of the following are correct pairings of an intrafamily planning technique with one of its characteristics except A) gift-leaseback: the donor leases the property back from the donee at a reasonable rental value. B) sale-leaseback: the transferor continues to have the right to use business-related property that she previously owned. C) installment sale: reporting gain on an installment basis is automatic if a payment is made in any year other than the year of the sale, unless the seller elects not to have it apply. D) private annuity: payments from the transferee to the annuitants are paid for a stated term certain. Explanation The answer is private annuity: payments from the transferee to the annuitants are paid for a stated term certain. It is the only wrong pairing of an intrafamily planning technique to a characteristic. Annuity payments from a private annuity are made for the lifetime of the annuitant regardless of the number of years involved. The transferor is immediately taxed on the difference between the present value of the promised payments and her basis in the transferred property.

d

All of the following statements regarding joint tenancies with right of survivorship are CORRECT except A) in joint tenancy the ownership percentages must be equal. B) joint tenancy with right of survivorship is similar to tenancies in common in that there may be two or more joint tenants who may or may not be related to each other. C) jointly held property passes to the surviving joint owners when one of the joint owner dies. D) jointly held property can be transferred by will. Explanation Jointly held property cannot be transferred by will. It passes to the surviving joint tenants by operation of law outside of the will. While there is no legal restriction that joint tenancy with right of survivorship (JTWROS) owners must be related, it would almost never be correct for people with no feelings of long-term connection to choose to be JTWROS because the property would pass to the other joint tenant upon death.

d

All of the following statements regarding self-canceling installment notes (SCINs) are correct except A) if the self-canceling provisions of a SCIN never become operative, the tax consequences of a SCIN are identical to those of a regular installment sale. B) the purchaser's basis in the asset purchased is equal to the purchase price if a SCIN premium has been paid. C) if the purchaser has paid a premium to obtain the cancelation provision in the SCIN, the seller may not have to include any payments canceled at death in his gross estate. D) the purchaser's basis in the asset purchased is limited to the payments that are actually made to the seller before death. Explanation The purchaser's basis in the asset purchased is limited to the payments that are actually made to the seller before death. The purchaser's basis is the agreed upon purchase price even if some of the anticipated payments are never made because of the cancelation provision.

d

An estate elects to value property using the alternate valuation date. Subsequently, some property in the estate is sold after the date of death and before the alternate valuation date. Which of the following statements correctly identifies the valuation date for that property? A) The property is valued as of the alternate valuation date. B) The property is valued as of the date of death. C) The property is valued as of the last day of the month that the decedent died. D) The property is valued as of the date of sale. Explanation If the estate elects to use the alternate valuation date and property is disposed of within the six-month period after the decedent's death, the valuation date is the date of disposition.

d

Andrew and Alicia are husband and wife who live in a community property state. Soon after their marriage they began establishing an emergency fund using money that each earned from their respective jobs. This fund was used to meet unexpected expenses as they arose. Three years ago, Alicia liquidated a bond fund that she had purchased prior to their marriage, and placed the proceeds in the emergency fund. There have been many deposits and withdrawals from the fund since that time. Last year, Andrew filed for divorce. Alicia is seeking to recover the full value of the bond fund proceeds that she placed in the emergency fund as her sole and separate property, and half of the remaining emergency fund. Andrew claims he is entitled to half of the entire emergency fund. Which one of the following statements is CORRECT regarding Andrew's and Alicia's rights in the emergency fund? A) The bond fund proceeds are community property, and Alicia and Andrew are each entitled to a percentage of the total emergency fund equal to their respective contributions. B) The entire emergency fund is separate property, and Alicia and Andrew are each entitled to a percentage of the total emergency fund equal to their respective contributions. C) The stock proceeds are Alicia's separate property, and she should be entitled to recover these funds in full as well as one-half of the remaining emergency fund. D) The entire emergency fund is community property, and Alicia and Andrew are each entitled to one-half of the total emergency fund. Explanation The answer is the entire emergency fund is community property, and Alicia and Andrew are each entitled to one-half of the total emergency fund.

d

Anne purchased property and titled it as joint tenancy with right of survivorship with her spouse. If she dies, how much of the property will be included in her gross estate? A) 100% B) 0% C) 25% D) 50% Explanation Property held by spouses as joint tenancies with right of survivorship or as tenants by the entirety is not valued under the percentage-of-contribution formula. Instead, half is included in the estate of the first spouse to die, regardless of who made the contribution.

d

Arthur and Tasha are a married couple, and they have three children. If they begin making gifts to their children in 2020, which of the following gifts would require the filing of a gift tax return for tax year 2020, knowing the gift tax annual exclusion remains at $15,000 for 2020? Arthur gives each child $10,000, for a total of $30,000 in gifts. Arthur gives each child $10,000, for a total of $30,000 in gifts, and he and Tasha elect gift splitting. Tasha gives a future interest gift worth $5,000 to one child. A) I, II, and III B) III only C) I and II D) II and III Explanation Statements II and III are correct. A gift tax return must be filed whenever a married couple elects gift splitting and whenever a gift of a future interest is made. Statement I is incorrect; a gift tax return is not required in this case because no gift to any donee exceeds the annual exclusion amount.

d

Arthur owns a life insurance policy on himself with a face value of $1.75 million and a cash value of $250,000. His wife and four children are beneficiaries of the policy. If Arthur died today, what amount of life insurance proceeds would be included in his probate estate? A) $1.75 million B) $250,000 C) $50,000 D) $0 Explanation Because of the beneficiary designations, the proceeds would pass by contract to the beneficiaries he named. None of the proceeds would be included in Arthur's probate estate.

d

Assume that Michael and Marie own their personal residence as a joint tenancy with right of survivorship (JTWROS) and that they originally purchased the residence for $100,000 several years ago. If Michael dies in 2020, which of the following statements is(are) CORRECT? The residence will pass through Michael's probate estate. The residence will pass to Marie under Michael's will. Marie's basis in the residence after Michael's death will be $175,000. A) I only B) II and III C) I and II D) III only Explanation Statements I and II are incorrect because the residence passes to Marie by operation of law. It is not included in Michael's probate estate, and it does not pass under his will. Statement III is correct. Marie receives a stepped-up basis in one-half of the residence ($125,000), which is added to her existing basis of $50,000 in the other half of the property.

d

Bill Martian would like to benefit the University of Florida at Orlando (the University), his alma mater. Due to a temporary cash flow problem, the only property Bill can use is a vacant lot (which he purchased six years ago for $5,000) located next to a site on which the University plans to build a new central library. The University would like to acquire this land for a parking lot at a reduced price. The present fair market value (FMV) of the lot is $25,000. Due to his cash flow problem, Bill would not only like to get any income tax benefits from a charitable contribution but also would like to receive some cash as soon as possible. Given Bill's objectives and the situation stated above, which one of the following is the most appropriate charitable giving technique for Bill to use? A) A charitable lead trust B) A charitable remainder annuity trust (CRAT) C) A charitable remainder unitrust (CRUT) D) A charitable bargain sale Explanation A charitable bargain sale will allow Bill to (1) benefit the University, (2) get a charitable income tax deduction, and (3) obtain immediate cash to help his cash flow by selling the lot to the University at below current FMV, which would make the transaction part sale and part charitable gift. None of the other alternatives would get cash to Bill as quickly, and the lead trust would get him no cash at all.

d

Bob Dickson established a Uniform Transfers to Minors Act (UTMA) account for each of his 10 children in 2019. He funded each account with $580,000. Bob had made $200,000 of taxable gifts in prior years. The gift tax due on these prior gifts was paid by his gift tax credit amount. Bob did not make any other gifts to his children in 2020. Which of the following is the net federal gift tax due for the 2020 gifts? A) $185,400 B) $67,525 C) $232,000 D) $0 Explanation Transfers to an UTMA are eligible for an annual exclusion. Therefore, $565,000 of each transfer is taxable, for a total of $5,650,000. Bob's prior taxable gifts must be added to this amount to determine total taxable gifts of $5,850,000 ($5,650,000 + $200,000). This figure is taken to the gift tax rate table to compute a tax due of $2,285,800 ($4,850,000 × 40% = $1,940,000 + $345,800 [tax on $1 million]). From this amount, the tax on $200,000 (the prior taxable gifts) is deducted. The tax on $200,000 is $54,800. Therefore, $2,285,800 - $54,800 = $2,231,000. Because Bob has used $54,800 of his $4,577,800 gift tax credit amount on prior gifts, he has $4,523,000 of this credit left to apply to current gifts. Therefore, Bob's net federal gift tax due for the current transfers is $0 because he can use a portion of his remaining applicable credit amount.

d

Davis died in 2020 and was survived by his spouse and two children. At the time of his death, he owned the following property interests: Solely owned property valued at $6 million Property owned in joint tenancy with right of survivorship (JTWROS) with his spouse, with his share valued at $2 million Davis's will made no charitable bequests and provided that his entire probate estate go equally to his surviving children. Other pertinent facts include the following: Davis made $1 million in post-1976 taxable gifts. Davis's estate had $170,000 in allowable debts. Davis's estate had funeral expenses of $80,000. Davis's estate had administrative expenses of $100,000. Davis's estate paid $50,000 in state death taxes. Which of the following amounts most closely approximates Davis's federal estate tax base (also known as his tentative tax base, which is the amount with which he enters the estate tax table)? A) $8,400,000 B) $0 C) $7,150,000 D) $6,600,000 Explanation The answer is calculated as follows: Gross estate of $7 million ($6 million in solely owned property plus $1 million as his half of the JTWROS property), less $400,000 in deductions for expenses and debts, less $1 million for the marital deduction of the JTWROS property, equals a taxable estate of $5.6 million. To that, add $1 million for post-1976 gifts to arrive at $6.6 million.

d

Ernesto, a married individual, died in 2020 with a gross estate with a fair market value of $30 million. His marginal estate tax bracket is 40%. The majority of his estate consists of personal use assets and publicly traded stock, which has rapidly declined in value since his death. Ernesto appointed his wife, Davila, as personal representative and beneficiary of one-half of his estate. He gave the remainder of his estate to his nephew. During the last year of his life, Ernesto incurred medical expenses of $50,000, all of which were reimbursed through health insurance. Two years before his death, he gifted $20,000 to his nephew and filed a gift tax return without Davila's consent to split gifts. Ernesto and Davila were in a 24% marginal income tax bracket immediately prior to his death. Which one of the following is a postmortem election that will minimize tax liability the most for Ernesto's estate or its beneficiaries? A) An election to defer and make installment payments of estate taxes B) A claim of medical expenses on his final income tax return C) The filing of a gift tax return with Davila's consent to split gifts D) The use of the alternate valuation date election for estate assets Explanation The stock has declined in value, and most personal use assets also tend to decline in value over time. Therefore, use of the alternate valuation date will probably meet its two qualifying criteria: (1) that the value of the gross estate be reduced and (2) that the estate tax due be reduced. The part of the estate not given to Davila will be taxable, but this tax could be reduced or even eliminated if the gross estate is reduced enough. The medical expenses were reimbursed by insurance. No election to split gifts may be made if a gift tax return has been filed for that year. An election to defer and make installment payments of estate taxes can be made only if more than 35% of a decedent's adjusted gross estate consists of a closely held business interest. There is no such business interest in Ernesto's estate.

d

Estates that contain closely held businesses may elect certain postmortem tax treatments. Which of the following statements about these treatments is CORRECT? The Section 6166 installment payment of estate tax requires that the decedent's closely held business interests constitute at least 50% of the decedent's adjusted gross estate. If property that has received special use valuation tax treatment loses its status as qualified property within 10 years of the decedent's death, a recapture tax is applied. A Section 303 stock redemption allows a partnership to make a distribution to redeem a portion of the stock of a decedent while avoiding dividend treatment on the amount paid for the stock. The estate of a deceased owner of a closely held business that leases all of its property cannot qualify for special use valuation. A) I and IV B) I, II, and III C) III only D) II and IV Explanation Section 6166, which provides for deferral and installment payment of the estate tax, can be elected if more than 35% of the decedent's adjusted gross estate is attributable to the value of a closely held business or businesses. A Section 303 stock redemption cannot be used for a partnership, as a partnership does not have any stock. A closely held business that leases all of its property would not be able to take the special use valuation.

d

For purposes of the generation-skipping transfer tax (GSTT) all of the following are considered skip persons except A) a trust in which all of the beneficiaries are two or more generations below the transferor. B) the transferor's grandchild. C) an unrelated person who is younger than the transferor by 37½ years or more. D) the transferor's ex-spouse, who is 40 years younger than the transferor. Explanation A transferor's spouse or ex-spouse is not a skip person, regardless of age. Also, any children of a spouse or ex-spouse are only one generation below the spouse(s). For example, if a 70-year-old married a 30-year-old who already has a 3-year-old child from a different relationship, the 30-year-old spouse is legally considered to be of the same generation as the 70-year-old and the stepchild is only one generation below the 70-year-old.

d

Fred and Ethel live in a community property state. They acquired property during their marriage and classified it as separate property pursuant to a formal legal agreement. What is the effect of the formal legal agreement? A) The property is separate property so long as the formal legal agreement is valid (i.e., recognized by local and federal law and entered into with the requisite intent). B) The property is community property. Property classification cannot be changed pursuant to a formal legal agreement. C) The property is community property. In community property states all property acquired during a marriage is considered community property. D) The property is separate property so long as the formal legal agreement is valid (i.e., recognized by local law and entered into with the requisite intent). Explanation A formal legal agreement will prevail if it is valid (i.e., recognized by local law and entered into with the requisite intent).

d

Gary and Georgeann have the following objectives: If Gary predeceases Georgeann, to provide her exclusively with a mandatory stream of income from the assets included in his gross estate To ensure that Gary's children from his prior marriage will ultimately receive the income-producing assets upon Georgeann's death To prevent assets used to provide income to Georgeann from being included in her gross estate Which of the following estate planning techniques would accomplish the couple's second and third objectives simultaneously? A family bypass (B) trust A power of appointment (A) trust A QTIP (C) trust, with an election A QTIP (C) trust, without an election A) I only B) I, II, and III C) II and III D) I and IV Explanation To accomplish the couple's second and third objectives, the property must be placed in a trust that will not qualify for the marital deduction. A family bypass trust and a QTIP trust without an election do not qualify for the marital deduction. If the estate of the first spouse to die takes a marital deduction, the estate of the surviving spouse will always have to include the trust assets in his gross estate.

d

George has a gross estate valued at $3.7 million. His estate consists almost entirely of publicly held stock owned solely by him. He owes no debts. George's only living relative is a nephew whom he hasn't seen or heard from for 30 years. George has not executed a valid will. If George were to die in the current year without change in any of the related facts, which one of the following is a disadvantage of the probate process for him? A) It will not allow George's estate to claim a marital deduction to reduce the taxable estate. B) It will not allow payment of a personal representative's fee to reduce his estate tax so that it can be covered by the applicable credit amount. C) It will not allow George's estate to be subject to court supervision regarding payment of claims and distribution. D) It will not allow distribution of his estate without incurring considerable cost in attempting to locate his nephew. Explanation The marital deduction has nothing to do with probate. Payment of a personal representative's fee would reduce his estate tax, but his estate is small enough that his applicable credit amount will certainly cover any taxable estate. Probate requires court supervision regarding payment of claims and distribution.

d

Grace Grubbs, your 76-year-old client, has the following objectives: Shifting some of the future appreciation in her portfolio of marketable securities to other members of her family Spreading income from the portfolio among her family without any preferential rights to income Maintaining control over the entire portfolio for her lifetime Reducing the size of her gross estate Grace is considering several techniques. Which one of the following would be most appropriate for accomplishing Grace's objectives? A) An installment sale B) A 20-year GRUT (grantor retained unitrust) C) A regular (C) corporation in which she retains all of the voting shares, and distributes nonvoting shares to family members D) A subchapter S corporation in which she retains most of the shares and distributes the remaining shares to family members Explanation This is the correct answer because it is the technique that will accomplish all of Grace's goals at the least tax cost. By retaining most of the shares, she would retain control of the corporation and thus control of the securities. She can reduce her gross estate by gifting the remaining shares to her family. These shares will be entitled to income on the same basis as Grace's shares, and she can spread the income among the family members who are now owners of the corporation. A gift of these shares will not be subject to the Chapter 14 rules because they are of the same type as those retained by Grace. Some of the future appreciation of the subchapter S corporation will accrue to the holders of these shares. Other options would require Grace to surrender lifetime control of the securities. In addition, an installment sale would require Grace to recognize all gain in the securities, as installment reporting of gain is not available for a sale of marketable securities. If Grace were to die during the 20-year term of the GRUT (which is likely), the fair market value of the shares at death would be included in her gross estate, and thus, her gross estate would not be reduced. The regular (C) corporation could achieve all of Grace's goals, but at a greater tax cost. A transfer of shares in such a corporation would be subject to the Chapter 14 rules because Grace's shares would be of a different type than those gifted to family members. Grace would not have retained a qualified right in the corporation for Chapter 14 purposes, and thus she would have to pay a larger gift tax. A C corporation is also potentially subject to the personal holding company tax, whereas a subchapter S corporation is not.

d

Greg died in 2020 and was survived by his wife and five children. At the time of his death, he owned the following property interests: Solely owned property valued at $16,000,000 Property owned in joint tenancy with right of survivorship (JTWROS) with his spouse, with his share valued at $1,000,000 Greg's will made no charitable bequests and provided that his entire estate go equally to his surviving children due to his wife having a large estate of her own. Other pertinent facts are: Greg made $1,000,000 in post-1976 taxable gifts. Greg's estate had $350,000 in allowable debts. Greg's estate had funeral and administrative expenses of $150,000. Greg's estate paid $60,000 in state death taxes. Which one of the following amounts most closely approximates Greg's net federal estate tax due? Use the Unified Federal Estate and Gift Tax Rates table. A) $4,577,800 B) $1,008,000 C) $6,521,800 D) $1,944,000 Explanation Gross estate is $17,000,000. Subtract $560,000 in debts, expenses, and taxes to get $16,440,000. The marital deduction can be used for the JTWROS property, so subtract $1,000,000. Then you have to add $1,000,000 for adjusted taxable gifts leaving the tax base at $16,440,000. Tax on this number is $6,521,800 [($15,440,000 × 40%) + $345,800]. Subtracting the applicable credit amount of $4,577,800 from $6,521,800 equals $1,944,000.

d

Herbert, who lives in a common-law state, has a will that gives his entire probate estate in equal shares to his three children. All of Herbert's $16,800,000 gross estate is owned in his sole name except for his residence, which is owned as joint tenants with right of survivorship with his wife. Herbert's interest in this residence is valued at $500,000. Despite having been married for 30 years, Herbert's wife has no substantial estate of her own. Herbert has made $400,000 in adjusted taxable gifts since 1976. Herbert's wife is named personal representative (PR) of his estate. Assuming Herbert is survived by his wife and children, which one of the following is a disadvantage of the probate process for Herbert? A) Herbert's estate will have to pay estate tax because the marital deduction will be too small to eliminate all tax. B) Herbert's wife will not be allowed to elect against the will unless she disclaims her right to receive Herbert's interest in the house. C) The probate process will not allow Herbert's children to disclaim any part of his estate so that Herbert's wife can receive more of his estate. D) Herbert's wife will have the right to elect against the will even though she is named as the PR of the estate. Explanation With no substantial estate of her own, Herbert's wife will be tempted to elect against the will to get part of the probate estate. While the amount she can get by this election is controlled by state law, and this amount may be reduced by the value of his interest in the residence, most states will allow a surviving spouse (especially in a marriage of this duration) to receive a substantial portion of the probate estate. While option a. states a correct consequence of the way that Herbert has elected to dispose of his property, this consequence is not due to the probate process, and thus, is not a disadvantage of that process. The probate process does not restrict the right to disclaim. A spouse can elect against the will and retain what is given to him or her by will substitute. The amount given by will substitute is simply deducted from the amount given by making the election, and the spouse receives the difference from the probate estate.

d

If Arthur died today, which of the following estate tax features would NOT apply to his estate? A) The portability of unused exemption lifetime amount between spouses (subject to some restrictions) B) The 40% top estate tax rate C) The $11,580,000 exemption amount D) The carryover basis for most inherited assets Explanation For decedents dying in 2020, the estate tax lifetime exemption amount is $11,580,000, the top estate tax rate is 40%, and portability of the unused exemption amount is available between spouses. Inherited assets—other than income in respect of a decedent—generally receive a stepped-up basis.

d

In 2020, George decided to begin a program of lifetime giving to his five grandchildren and three great- grandchildren. He wants to control the amount of annual gifts to avoid the imposition of federal gift tax, and he does not desire to use any of his or his spouse's applicable credit amount. However, his spouse is willing to split each gift over a period of 10 years. Over the 10-year period, George can give a total amount of gifts (ignoring future indexing of the annual exclusion), including the gift splitting, of A) $150,000. B) $1,200,000. C) $300,000. D) $2,400,000. Explanation Calculate as follows: $15,000 × 8 × 10 × 2 = $2,400,000. Each spouse can gift $15,000 (for 2020) per donee without resulting in a taxable gift. Using gift splitting over the 10-year period, they can gift a total of $2,400,000.

d

Jacob, who is not married, owns the following assets: (1) investment accounts worth $6 million; (2) a life insurance policy worth $5 million; (3) a house worth $2.5 million; (4) retirement accounts worth $2 million; and (5) tangible personal property worth $1 million. Jacob created a revocable living trust that names his three children as the beneficiaries, and he funded the trust with all of his assets except the retirement accounts. Which one of the following is a CORRECT statement regarding the effect of this trust on the potential liquidity of Jacob's estate at his death? A) The trust will shield Jacob's estate from an estate tax liability, and therefore is sufficient to meet estate liquidity needs. B) The trust represents neither a potential cash requirement, nor a potential source of liquidity. C) The trust does not represent a potential cash requirement, but does represent a potential source of liquidity. D) The trust assets will be included in Jacob's gross estate and will be available to meet estate liquidity needs. Explanation The estate is already below the exclusion amount so the trust won't have any effect on estate tax.

d

Jane has a gross estate estimated at $12 million. Approximately 75% of her estate is attributable to the value of personal property and collectible items. Jane is married but has no children. Her spouse does not have a large estate, as he spends money freely and foolishly. Since Jane is much older than her spouse, she would like for him to benefit from her wealth after her death without giving him control over the principal either while he is alive or at his death. Jane wants as little of her estate assets as possible to go toward payment of estate taxes. She currently has no will but has come to you for advice regarding provisions she should put in a will. Which provision, if placed in her will, would be the best to increase the liquidity of her estate and accomplish her other goals? A) Establish a qualified terminable interest property trust naming her spouse as the income beneficiary and her children as the remainder beneficiary B) Establish a power of appointment trust naming her spouse as the income beneficiary and a qualified charity as the remainder beneficiary C) Establish a testamentary trust naming her spouse as the income beneficiary and trustee D) Establish a charitable remainder trust naming her spouse as the income beneficiary and a qualified charity as the remainder beneficiary Explanation Jane should establish a charitable remainder trust naming her husband as the income beneficiary and a qualified charity as the remainder beneficiary. This way, her spouse benefits from the income that the charitable remainder trust generates and yet removes from his control the ability to sell the asset. Further, the charitable gift will reduce her estate, and therefore, any estate taxes that may be owed. LO 6.3.2

d

Jane has a gross estate estimated at $18 million. Approximately 75% of her estate is attributable to the value of personal property and collectible items. Jane is married but has no children. Her husband does not have a large estate because he spends money freely and foolishly. Because she is much older than her husband, Jane would like for him to benefit from her wealth after her death without giving him control over the principal either while he is alive or at his death. Jane wants as little of her estate assets as possible to go toward payment of estate taxes on either of their estates. She currently has no will but has come to you for advice regarding provisions she should put in a will. Which provision, if placed in her will, would be best to increase the liquidity of her estate and accomplish her other goals? A) Establish a power of appointment trust naming her husband as the income beneficiary and a qualified charity as the remainder beneficiary B) Establish a qualified terminable interest property (QTIP) trust naming her husband as the income beneficiary and a friend or other relative as the remainder beneficiary C) Establish an testamentary trust naming her husband as the sole beneficiary and trustee D) Establish a charitable remainder trust naming her husband as the income beneficiary and a qualified charity as the remainder beneficiary Explanation A QTIP wouldn't change the liquidity situation. A power of appointment trust gives the husband access to trust principal she does not want him to have while she is alive and the testamentary trust allows the same access upon her death.

d

Jasmine dies owning 1,000 acres of farmland, which is used as a family farm. The value of the acreage as farmland is $3,000 per acre. Many adjoining farms have been sold to commercial developers for as much as $20,000 an acre. Jasmine's will leaves the acreage to her son, who intends to use it as farmland for the foreseeable future. Which of the following postmortem estate planning techniques will be most useful to Jasmine's estate? A) Election against the will B) QTIP election C) Alternate valuation date (AVD) D) Special use valuation (Section 2032A) Explanation Special use valuation under Section 2032A will be most useful to Jasmine's estate because it permits the acreage to be valued at its current use as farmland ($3,000 per acre) rather than at its highest and best use as commercial development ($20,000 per acre). In 2020, the maximum reduction in value under Section 2032A is $1.18 million. In this case, the farmland is worth $3 million as a farm, but $20 million as a commercial development. The Section 2032A special use valuation would reduce the amount in the gross estate to $18,820,000 ($20 million - $1.18 million). The estate tax savings would be $472,000 ($1.18 million × 40%).

d

Jason wants to contribute $10 million to a charitable trust. He expects to receive a large inheritance in a few years, so he wants to receive an income interest from the trust for only 15 years and not for life. Which of the following charitable trusts will meet his needs? Pooled income fund Charitable remainder annuity trust (CRAT) Charitable remainder unitrust (CRUT) A) I, II, and III B) III only C) I only D) II and III Explanation Statement I is incorrect because in a pooled income fund, the donor (or one or more named beneficiaries) must retain a life income interest. Retained term interests are not allowed. Statements II and III are correct.

d

Jean died in a common-law state in 2020 and was survived by her spouse, Loren, and three adult children. Jean's gross estate, all of which was owned solely in her name, was composed of the following assets and date of death fair market values: AssetsValuesCommon stock$ 2,900,000Residence750,000Personal property60,000IRAs1,450,000Hummel figurines175,000Total Assets$ 5,335,000 Jean's only liabilities, together with their date of death balance, were as follows: LiabilitiesBalanceMortgage on residence$175,000Car loan8,000Total$183,000 The following is a list of all of the gratuitous transfers that Jean made during her lifetime: 2000: Placed the common stock previously listed in an irrevocable trust in which she retained the right to a 5% distribution of the trust account revalued annually for 25 years, with the remainder to her children at her death. The date of gift fair market value of the stock was $190,000, and the value of Jean's retained interest on the date the gift became complete was $90,000. 2001: Gave a cash gift to her niece of $30,000, which Loren agreed to split 2005: Paid the University of Iowa $11,000 for her youngest child's tuition 2020: Gave her brother $70,000 in cash, which Loren agreed to split Jean's will, executed in 2000, gave the residence and personal property to Loren and the Hummel figurines to a qualified charity. Jean's three children were designated as equal beneficiaries of her IRAs. Each beneficiary of Jean's estate was to pay any transfer tax due on the portions of the estate received. Her will also stipulated that any real property was to be received subject to the mortgage. Further, her will stipulated that funeral and administrative expenses were to be paid equally by her spouse and three children. These funeral and administrative expenses amounted to $60,000 for Jean's estate. Jean's estate paid state death taxes in the amount of $4,184 and will pay off the car loan. Which of the following amounts most closely approximates the marital deduction available to Jean's estate? A) $1,042,500 B) $635,000 C) $810,000 D) $620,000 Explanation The answer is $620,000. The marital deduction is granted only for the net amount of assets transferred to the decedent's surviving spouse in a qualifying manner. The facts state that the only assets that are given to Loren are the residence ($750,000) and personal property ($60,000). However, this total must be reduced by the $15,000 in funeral and administrative expenses that Loren must pay, as well as the $175,000 mortgage on the residence that he must pay as required by the will. Therefore, the net amount of assets that will pass to Loren is $620,000.

d

Joe made a gift of property to his niece in the amount of $50,000 on the condition that his niece pay any gift tax due. Joe has previously made prior taxable gifts in the amount of the applicable exclusion amount. His niece, however, has never made a taxable gift. Which of the following are CORRECT statements about the tax implications of making this gift? The net amount of the gift (value of the gift minus gift taxes paid) minus the maximum annual exclusion amount will be included in Joe's estate tax calculation as an adjusted taxable gift. Joe will have taxable income to the extent that the gift tax paid by his niece exceeds Joe's adjusted basis in the property. Joe's niece will have a basis in the property equal to Joe's basis in the property as adjusted for gift taxes paid by her. If Joe's niece accepts this gift, she will have to pay a gift tax out of pocket. A) I and IV B) I and III C) II and III D) I, II, III, and IV Explanation All of these statements are correct statements regarding the tax implications of making this gift.

d

Jorge is in the highest current marginal income tax bracket. He owns several thousand shares of rapidly appreciating growth stock that he wants to transfer to his three minor children. Income will not be used for legal support obligations. He wants to have some control over the distribution of income from this stock while affording himself the gift tax annual exclusion for the total value of this and subsequent transfers to the maximum amount allowed. He also wants this stock to be available to his children when they reach age 21. Jorge does not want income from the stock to be taxed to him. Which one of the following is the most appropriate lifetime transfer technique for Jorge to use to best achieve his objectives? A) Net gifts to each child B) Outright gifts of equal amounts to the children C) Irrevocable trusts structured under the provisions of Internal Revenue Code Section 2503(b) D) Irrevocable trusts structured under the provisions of Internal Revenue Code Section 2503(c) Explanation A 2503(c) trust accomplishes all of Jorge's goals: maintaining control until the children reach 21, getting the annual gift tax exclusion, and giving the corpus to the children at age 21. Outright gifts cannot do all of that. A 2503(b) trust does not require the corpus to be distributed at age 21 and net gifts leave gift tax payable by the children which is definitely not one of Jorge's goals.

d

Katie, who is unmarried, made the following gifts in 2020: Cash to her sister, Kellie, in the amount of $16,000 Stocks valued at $26,000 to her cousin, Lawrence An automobile (valued at $17,000) to her uncle, Burt $24,000 to the Compassion Center, a charitable organization Certificates of deposit worth $10,000 to her childhood friend, Marjorie What is the amount of Katie's taxable gifts in 2020? A) $93,000 B) $69,000 C) $45,000 D) $14,000 Explanation The total of Katie's taxable gifts is $14,000 ($1,000 to Kellie, $11,000 to Lawrence, and $2,000 to Burt). Note that with the exception of the gift to the Compassion Center, the $15,000 annual exclusion is deducted from each gift to arrive at the taxable gift. The $24,000 given to the Compassion Center is a charitable gift and is fully deductible under gift tax law, so there is no gift tax liability.

d

Lauren and Roger are spouses. Lauren has assets with a market value of $50 million titled in her name alone. Roger has assets valued at less than $1 million. Lauren drafts a will making an outright bequest of all of her assets to Roger. Which of the following are potential disadvantages of Lauren's approach? Lauren will not use her estate tax applicable exclusion amount when she dies. Roger may become legally incapacitated and not be able to manage the property. When Roger dies, the property must be included in Roger's gross estate to the extent Roger has not spent it or consumed it during his lifetime. The DSUE amount may not be available in some circumstances. A) I and IV B) II and III C) I and II D) I, II, III, and IV Explanation All of these are potential disadvantages of leaving all property outright to the surviving spouse.

d

Lionel Trane has made the following lifetime transfers: Gave his spouse a remainder interest worth $56,000 in a parcel of real estate after his death Funded a Section 2503(c) trust for the benefit of his daughter with $60,000 of common stock; his spouse did not split this gift. Paid his mother's medical bill to the community hospital in the amount of $15,000 Established a revocable trust for his only grandchild with $8,000 in cash Which of the following statements describe the tax impact of Lionel's lifetime transfers on subsequent lifetime transfers that Lionel may wish to make? The gift to his spouse will reduce the amount of future lifetime taxable transfers that he can make without having to pay the gift tax out of pocket. Establishing and funding the trust for his daughter will reduce the amount of future lifetime taxable transfers that he can make by the value of the gift, minus one annual exclusion, without having to pay the transfer tax out of pocket. Paying his mother's hospital bill will have no effect on subsequent lifetime transfers. Establishing the revocable trust will have no effect on subsequent lifetime transfers. A) I, II, and IV only B) I and III only C) II only D) II, III, and IV only Explanation The gift to his spouse will have no effect because no part of it will be taxable. The entire amount will be covered by the unlimited marital deduction because the gift of a vested remainder is not a terminable interest. Also, since it is a future interest gift, the remainder interest is not entitled to an annual exclusion. The gift to the daughter's trust is entitled to an annual exclusion by the terms of Section 2503(c) even though it technically is not a present interest gift. Direct payment of medical expenses to the provider is exempt from gift tax. A revocable trust is revocable and therefore is not a completed gift.

d

Mark wants to establish a grantor-retained trust that will provide him with fixed annual payments for the rest of his life. Which of the following trusts will meet his needs? GRAT GRUT A) Neither I nor II B) Both I and II C) II only D) I only Explanation A GRAT provides the grantor with a fixed annuity payment. Statement II is incorrect because in a GRUT, the income interest payable to the grantor is in the form of a fixed percentage of the fair market value of trust assets, as revalued annually.

d

Matt gives Jim securities in the current year. Matt's adjusted basis for the securities is $48,000, and the fair market value (FMV) is $40,000. Matt pays gift tax of $16,000. What is Jim's basis in the stock for gain and for loss? A) $50,000 for gain and $42,000 for loss B) $40,000 for gain and $40,000 for loss C) $0 for gain and $0 for loss D) $48,000 for gain and $40,000 for loss Explanation On the date of the gift, the FMV of the property was less than the donor's basis. Therefore, the double basis rule applies. The donee's gain basis for the property received is the same as the donor's basis. The donee's loss basis is the lesser of the donor's adjusted basis or the FMV on the date of the gift. If the FMV of the property is less than the adjusted basis at date of gift, no basis adjustment is made for gift tax paid.

d

Meredith has completed a preferred stock recapitalization of a business that had been solely owned by her. She retained all preferred shares in the new business entity and gifted all nonvoting common shares to her three children in equal shares. Her preferred shares have a fixed liquidation value and a cumulative right to a fixed amount of income. Which of the following are CORRECT statements regarding the effect of this transaction upon the liquidity of Meredith's estate? She has reduced the cash needs of her estate because she has reduced the size of her gross estate. She has increased the potential cash resources of her estate by establishing a market for the shares in the corporation owned at death. She has decreased the cash needs of her estate by completely eliminating the business as an asset of her probate estate. If her estate will owe transfer taxes, she has increased the potential cash needs of her estate by using some of her applicable credit amount to pay the gift tax for the gifts to her children. A) II, III, and IV B) III and IV C) I and II D) I, II, and IV Explanation Meredith's gross estate has been reduced immediately by the value of the shares she gave to her children (option I). It will be reduced further in the future because of any business income that goes to her children, and by the appreciation in the value of the business that will also go to the children because her shares have a fixed liquidation value. One purpose of a transaction such as this one is to create a pool of motivated buyers for the portion of the business interest that is not gifted to them (option II). The gifts to the children will cause Meredith to use her applicable credit amount (option IV). To the extent it is used, it will not be available to cover any estate tax due at her death. The business will remain an asset of her probate estate because of the preferred shares that she has retained in her name (option III)

d

Mike and Jane, a married couple, bought a condominium 15 years ago for $200,000 as joint tenants with right of survivorship (JTWROS). When Jane died, the condominium was valued at $500,000. Four years after Jane's death, Mike sells the condominium for $600,000. What is the amount of Mike's capital gain? A) $0 B) $100,000 C) $300,000 D) $250,000 Explanation Mike received a stepped-up basis in one-half of the condominium at Jane's death. He retains his original basis of $100,000 in the other half. His total basis at the time of the sale is $350,000, so his gain is $250,000.

d

One premortem liquidity planning technique is to reduce the client's potential gross estate. Which of the following actions may reduce the client's gross estate? Placing assets in a grantor-retained annuity trust (GRAT) with a 10-year term that names the client's child as remainderman Placing assets in a revocable living trust that disperses its assets to the client's child at the client's death Placing assets in an irrevocable trust in which the client is neither a beneficiary nor trustee Placing assets in an irrevocable trust in which the client as sole trustee has discretion to distribute income A) II and IV B) II, III, and IV C) I, III, and IV D) I and III Explanation The assets in the GRAT will be removed from the grantor's gross estate if he survives the trust term. If the grantor of a trust retains no beneficial interest in his trust, or control over it by virtue of being trustee, no part of the trust assets will be included in his gross estate (unless the three-year inclusionary rule applies). Assets in a revocable trust (option II) are always included in the grantor's gross estate. If the grantor as trustee has discretion to distribute income (option IV), the grantor is deemed to have retained the power to determine beneficial enjoyment of the trust assets, and those assets are included in his gross estate.

d

Paulo and Francisca own a piece of property as tenants in common. Paulo owns an undivided 45% interest in the property, and Francisca owns an undivided 55% interest. The property has a fair market value of $1 million. If Francisca dies, what amount will be included in her gross estate for federal estate tax purposes? A) $500,000 B) $0 C) $1,000,000 D) $550,000 Explanation When property is owned as a tenancy in common, the amount included in the gross estate is based on the decedent's percentage of ownership. Francisca owns a 55% interest in the property, so her gross estate will include $550,000 ($1,000,000 × 55% = $550,000).

d

Preventing a surviving spouse from being left destitute upon the death of his spouse is NOT a purpose of which of the following? A) Laws of intestate succession B) Election against the will statutes C) Community property laws D) Advancement statutes Explanation Advancement statutes deal with the issue of whether a lifetime transfer to a beneficiary should reduce what he gets from the transferor at death. Community property laws provide a surviving spouse whose name is not on the title to property with a vested half interest in certain of the decedent spouse's property acquired during the marriage. Laws of intestate succession always provide for a surviving spouse first. An irrevocable trust can be used to transfer assets both while the grantor is alive (inter vivos) and when the grantor is dead (testamentary).

d

Roy died in 2020, and his estate consists of the following assets: 350 shares of Black & Blue Inc, a closely held corporation (100% of outstandanding shares)$5,250,000Real estate (rapidly appreciating)$2,100,000Remainder of estate1,300,000Gross estate$8,650,000Administrative expenses (includes PR fee)$(55,000)Debts of the decendent(90,000)(145,000)Adjusted gross estate$8,505,000 This year, before he died, Roy made a cash gift of $200,000 to his son, Edward. This is the only taxable gift that Roy ever made. Roy's wife, Thelma, was named personal representative (PR) of his estate in his will, which left the Black & Blue Inc. stock to Edward and the remainder of his estate to Thelma. Thelma is in the lowest income tax bracket. Which one of the following is the postmortem planning technique that is available to and advisable for Thelma and/or Roy's estate? A) Election of a Section 303 stock redemption B) Election by Thelma and the estate to split the gift to Edward C) Election to use the alternate valuation date D) Election by Thelma to waive her PR fee of $15,000 Explanation The alternate valuation date is incorrect because the gross estate would most likely increase due to the rapidly appreciating real estate, and since there is no estate tax due using fair market value, the alternate valuation date would not decrease an estate tax liability. While waiving the PR fee will increase the adjusted gross estate, there will be no increased estate taxes since the tax base is less than the applicable exclusion amount. Also, the fee would then pass to Thelma income tax free as a distribution of estate assets. Splitting the gift would unnecessarily waste Thelma's applicable credit amount without any benefit to the estate. A Section 303 stock redemption is not available to the estate or its beneficiaries (to pay funeral and administrative expenses) because the business doesn't exceed 35% of the adjusted gross estate when the transfer within three years of death is considered.

d

Rupert's gross estate is valued at $13.6 million using fair market valuations. His largest asset is a chicken breeding business, with real estate valued at $10.81 million and equipment valued at $1.4 million. Which of the following facts would Rupert's executor need to know to determine whether the real estate used in the chicken breeding business will qualify for special use valuation under Code Section 2032(a)? To whom the real estate will go according to Rupert's will How long Rupert or a member of his family has owned the real estate and how it has been used in the business The amount and types of gifts Rupert made within three years of his death The amount of all secured debts on the business property A) II, III, and IV B) I and II C) I and IV D) I, II, III, and IV Explanation All cited factors have the potential to impact the property's qualification for special use valuation. Option I is relevant to the requirement that the real estate must pass to a qualified heir. Option II is necessary to determine material participation and how long the real estate has been operated in a qualified use. Options III and IV are necessary to determine if the percentage tests can be met.

d

Ruth established a Section 2503(c) minor's trust for her 5-year-old granddaughter, Frances, and funded it with $20,000 of income-producing real estate. Ruth named herself as trustee. Which one of the following statements is CORRECT regarding this transfer? A) This trust will allow Ruth to determine when Frances will be entitled to receive the corpus. B) This transfer will assure Ruth that the trust property will not be included in her gross estate. C) This type of trust cannot be funded with real estate. D) This trust will allow Ruth to determine when Frances will be entitled to receive income from the trust. Explanation The corpus of 2503(c) trusts must be distributed by age 21. As long as Ruth has control of trust distributions, it remains in her estate. A 2503(c) trust can be funded with real estate. Income-producing real estate would be a good way to generate the income desired by such a trust.

d

Svetlana has a large estate. She and her spouse, Igor, are childless. Svetlana has promised her favorite brother, Max, a portion of her wealth. Svetlana recently updated her will. Which of the following provisions of Svetlana's will qualify assets placed in the cited trusts for the marital deduction? A trust that must pay all income to Igor at least annually during his lifetime. He has a lifetime, or testamentary, right to designate anyone, including himself, as an appointee of trust property. A trust in which all income is distributed annually to Igor for his life. The corpus passes to a qualified charity at his death. A trust that gives the trustee the discretion to pay income to Igor and Max until one of them dies. Then, the trust assets are to be distributed to the survivor. A) I, II, and III B) I and II C) II and III D) I only Explanation Option I is a power of appointment trust, which qualifies for the marital deduction automatically. The trust described in option II is not a qualified charitable remainder trust since it pays James neither an annuity nor a unitrust amount. Thus, this testamentary trust is not a CRAT or a CRUT. Also, a testamentary trust cannot be a pooled income fund (PIF) because PIFs are established by public charities (50% organizations), not by individuals in any circumstance. Therefore, this trust does not qualify for the marital deduction automatically. The trust described in option III does not qualify for three reasons. First, the income interest is not paid exclusively to Igor. Second, Igor's right to the remainder is contingent on his survival of Max. Finally, to receive a marital deduction, the arrangement cannot make a spouse survive for more than six months after the death of the first spouse. In this case, not only could her brother outlive her spouse, but even if her spouse outlived her brother, her brother could easily survive for more than six months. Thus, her spouse could be forced to wait longer than six months from her death to have exclusive benefits.

d

When Ed died, his will left all of his assets to his best friend, Mark, and nothing to his wife, Martha. However, a state law allowed Martha to claim one-third of Ed's estate even though his will left her nothing. This is an example of A) filing a qualified disclaimer. B) the family allowance. C) a homestead exemption. D) electing against the will. Explanation Many states have laws providing that a surviving spouse cannot be totally disinherited if she is married to the decedent at the date of death. These laws allow a disinherited spouse to make an election against the will and receive a specified share, such as one-third, of the decedent's probate estate.

d

When Marsha dies, her will leaves her entire estate to her surviving spouse, Daniel. Marsha's will provides that if Daniel does not survive Marsha by 90 days the estate will pass to their adult children. Daniel has sufficient assets of his own and would prefer that Marsha's property pass directly to their children now, instead of to him, but he wants to avoid making a taxable gift to them. Which of the following postmortem estate planning techniques would enable Daniel to achieve his objective? A) Partial qualified terminable interest property (QTIP) election B) Election against the will C) Reverse QTIP election D) Qualified disclaimer Explanation A qualified disclaimer will achieve Daniel's objective because it constitutes a refusal by Daniel to accept the property and allows the property to pass directly to her children without any gift tax consequences. A partial QTIP election is used to qualify a terminable interest for the marital deduction, and a reverse QTIP election is used to permit the decedent's estate to use the generation-skipping transfer tax exemption for QTIP property. An election against the will allows a surviving spouse who has been disinherited under the deceased spouse's will to claim a share of the deceased spouse's estate.

d

When she died, Roberta and her spouse, Patrick, owned a condo as joint tenants with right of survivorship (JTWROS). Their basis in the condo was $500,000, and the fair market value on the date of Roberta's death was $1,200,000. What is Patrick's basis in the home following Roberta's death? A) $1,200,000 B) $250,000 C) $600,000 D) $850,000 Explanation Because Roberta and Patrick were spouses, Patrick receives a stepped-up basis of $600,000 in the 50% of the condo that is included in Roberta's gross estate. He retains his original basis of $250,000 in the 50% that is not included in Roberta's gross estate. Patrick's new basis is $850,000 ($250,000 + $600,000).

d

Which of the following are CORRECT regarding the estate tax marital deduction? The marital deduction is for property that is included in a decedent's gross estate that passes, or has passed, to the surviving spouse in a qualifying manner. The marital deduction is not allowable for property bequeathed by the decedent to the spouse if the spouse does not survive the decedent. Generally, there is no marital deduction for an interest in property passing from the decedent to the surviving spouse when an interest in the same property also passes to some other person without full consideration, and that person or his heirs may enjoy the property after the surviving spouse's interest terminates. If the surviving spouse is the only noncharitable beneficiary of a qualified CRAT or CRUT, the marital deduction is allowed for the interest passing to the spouse. The marital deduction is never available for property passing outright to a surviving spouse who is a noncitizen. A) I and III B) II, III, and V C) II and IV D) I, II, III, and IV Explanation Only option V is incorrect—the marital deduction is not available for property passing to a surviving spouse who is a noncitizen unless the surviving spouse either becomes a U.S. citizen before the date on which the decedent's estate tax return is filed or the property passes to a QDOT (qualified domestic trust) prior to that date. Option I is the classic definition of the estate tax marital deduction. Option II is true because in these circumstances the property will not pass to the decedent's spouse. Option III is correct because it describes a terminable interest. Option IV is correct because such a situation is an exception to the terminable interest rule.

d

Which of the following are characteristics of the unified tax system that are common to both testamentary transfers and lifetime gifts? Availability of the marital deduction Use of adjusted taxable gifts in calculation of the tax Availability of the annual exclusion amount Availability of an applicable credit amount A) I, II, and III B) I, II, and IV C) III and IV D) I and IV Explanation The annual exclusion (option III) can be used only for lifetime gifts. Only the estate tax uses adjusted table gifts (option II) in making the tax calculation. Gift taxation uses taxable gifts, not adjusted taxable gifts. Adjusted taxable gifts start out as cumulative lifetime taxable gifts and then subtracts the taxable gifts made for property that actually ends up in the gross estate. For example, a donor placed his home in a qualified personal residence trust (QPRT), resulting in $120,000 of taxable gifts. The QPRT was scheduled to last for 10 years; however, the donor died after four years, so the value of the home was in his estate. To avoid double taxation, his taxable gift of $120,000 from when the QPRT was established is subtracted from his cumulative lifetime gifts. Although the estate and gift tax applicable credit amounts (option IV) were different in the recent past, each tax does have an applicable credit amount.

d

Which of the following are examples of gift giving that are likely to result in favorable tax consequences? An advantage of giving property with a current value that is less than its basis (loss property) is that when the recipient sells the property the loss is available to offset any gains. Elderly taxpayers should give highly appreciated, low basis property in preference to cash. Making net gifts is a technique for clients who do not have very much in liquid assets and who want to make taxable gifts. The donee can depreciate depreciable property based on its value for gift tax purposes. A) I and IV B) II, III, and IV C) I, III and IV D) III only Explanation Statement I is incorrect; the double basis rule would not permit this favorable treatment. Statement II is incorrect; at the date of death, the property basis would be adjusted to fair market value. Statement III is correct. Statement IV is incorrect; the property would be depreciated on the basis of its adjusted basis, not its fair market value.

d

Which of the following are interests that must go through probate? A residence that the decedent held in tenancy by the entirety with his spouse An automobile that passes according to state intestacy laws Securities held by the decedent with a T.O.D. designation Real property, located in the state of the decedent's domicile, that is held solely in the decedent's name but that is considered to be community property A) I, III, and IV B) I and III C) II, III, and IV D) II and IV Explanation The interest in option I will not go through probate because tenancy by the entirety has a right of survivorship feature. Option III will not go through probate because the T.O.D. (transfer on death) designation is a type of will substitute. The decedent's share of the real property in option IV will be subject to probate in the state in which it is located, as community property must pass through probate.

d

Which of the following are the tax implications of a 10-year term charitable lead trust with the donor's children as the remainder beneficiaries? The donor's charitable gift tax deduction is determined by the present value of the charity's right to receive trust assets at the end of the 10-year term. The donor is liable for gift tax based on the entire value of the gift to the children as discounted to the date of the gift. The entire value of the assets gifted to the trust will be removed from the donor's gross estate only if he or she outlives the 10-year term. Each year, as the trust pays income to the charity, the donor receives a charitable income tax deduction for that amount. A) I and III B) I and IV C) I, II, and III D) II only Explanation In a charitable lead trust, the charity receives the income with the remainder to a noncharitable beneficiary. The present value of this remainder interest is taxable. Therefore, the charitable gift tax deduction I is the present value of the income interest. III is incorrect as the entire value of the trust assets is removed from the grantor's gross estate no matter when the grantor dies. IV is incorrect as the amount of the charitable income tax deduction is based on the value of the gift to the charity in the year the gift is made, not on income earned.

d

Which of the following are valid concerns when an applicant for long-term nursing home benefits under Medicaid transfers title to his or her personal residence to his or her community spouse? The possibility the parties may divorce Whom the community spouse may leave the residence to when he or she dies The community spouse's creditors The terms of an existing mortgage on the residence A) I, III, and IV only B) I and IV only C) II and III only D) I, II, III, and IV Explanation All of these are valid concerns when considering Medicaid as an option to pay for long-term care.

d

Which of the following assets must be listed at their date of death value on the federal estate tax return, even if the estate elects the alternate valuation date (AVD)? A) Mutual funds B) Publicly traded securities C) Real estate D) Joint and survivor annuities Explanation The answer is joint and survivor annuities. Wasting assets, such as installment notes and joint and survivor annuities, must be listed at their date of death value, even if the estate elects the AVD.

d

Which of the following correctly state a premortem purpose of selling an illiquid asset? Illiquid assets may be sold to increase estate liquidity. Illiquid assets may be sold to ensure fair market value is received for the property. Illiquid assets may be sold to reduce estate shrinkage. Illiquid assets may be sold to reduce estate administration expenses. A) I and IV B) I, II, III, and IV C) II and IV D) I and III Explanation Statement II is false because illiquid assets are often very difficult to sell for full value. Statement IV is false because this type of sale will have no effect on estate administrative expenses.

d

Which of the following decedent's estates might be eligible to elect the alternate valuation date (AVD)? Jeanine, whose gross estate is $50 million. She leaves her entire estate outright to her surviving spouse. Kim, whose gross estate is $6.5 million. She leaves her entire estate to charity. A) Both I and II B) I only C) II only D) Neither I nor II Explanation Neither estate can elect the AVD because the AVD is not available when there is no estate tax due. Jeanine's entire estate qualifies for the marital deduction, and Kim's entire estate qualifies for the charitable deduction.

d

Which of the following is a characteristic of an installment sale? It requires the receipt of at least one payment in a taxable year following the year of sale. Reporting gain on an installment basis is automatic for qualifying sales unless the taxpayer elects not to have it apply. A) I only B) Neither I nor II C) II only D) Both I and II Explanation Both of these statements represent characteristics of an installment sale.

d

Which of the following is a nontax-related financial goal? A) Freezing or reducing the value of assets B) Leveraging the use of exclusions C) Shifting the receipt of income D) Maximizing flexibility Explanation The remaining choices are tax-related financial goals.

d

Which of the following is a part-gift, part-sale of an asset for less than full and adequate consideration? A) Reverse gift B) Outright gift C) Net gift D) Bargain sale Explanation A bargain sale is a sale for less than full and adequate consideration. It is considered to be part sale and part gift.

d

Which of the following is an advantage of owning property as joint tenants with right of survivorship (JTWROS)? When one tenant dies, the property passes directly to the surviving joint tenant. Each joint tenant with a right of survivorship has a right to sever or partition the property without the consent of the joint tenant. JTWROS is convenient for certain types of assets, such as bank accounts, because either tenant has access to the account. A) I only. B) II and III. C) I and II. D) I, II, and III. Explanation All of these are advantages of owning property as JTWROS.

d

Which of the following is an effective way for a client to remove the value of an asset from his estate? A) Placing the asset in his irrevocable trust in which he retains an income interest B) Placing the asset in his revocable trust C) Placing the asset in a grantor trust D) Gifting the asset to another individual Explanation The asset is no longer in the possession, and therefore no longer in the gross estate, of the donor. LO 5.2.1

d

Which of the following is an estate liquidity need that cannot be completely eliminated by proper premortem planning? A) Death taxes B) Cash bequests C) Miscellaneous cash needs, such as cash to pay the family allowance or to keep a business operating during the period of estate administration D) Administrative expenses Explanation Administrative expenses is the correct answer because all estates, whether large or small, have certain administrative expenses that are virtually impossible to eliminate completely. Examples of such costs include court costs and professional fees—the estate's attorney, accountant, appraiser, and so forth rarely, if ever, waive their fees. Thus, administrative expenses can be diminished by proper premortem planning but can never be completely eliminated.

d

Which of the following is an example of unauthorized practice of law? A) A financial planner explaining how federal estate, gift, and income tax liability affect an estate plan, and which estate planning documents could be used to achieve desired objectives. B) A financial planner explaining to his clients the dangers of using legal forms obtained from the internet. C) A financial planner advising her clients that their current will does not reflect their goals. D) A financial planner advising a client to place solely owned property into joint tenancy with right of survivorship. Explanation The financial planner is advising the client to give up his or her legal rights in the property, and, therefore, is practicing law. The remaining options are merely providing advice, which is exactly what a planner is expected to do.

d

Which of the following is considered to be a gift for federal gift tax purposes that qualifies for the gift tax annual exclusion? A) A transfer of property to fund a revocable trust B) A gift to a political organization for its use C) Property settlement transfers pursuant to a written divorce agreement D) A payment of cash to a favorite nephew to assist with his college tuition Explanation Because it's not made directly to an educational institution (which would be excludible as a qualified transfer), the payment of cash to a favorite nephew to assist with tuition is both a gift and one that qualifies as a present interest for purposes of the gift tax annual exclusion. The gift to a political organization is a gift, but there is no gift tax liability. The transfer of property to fund a revocable trust is an incomplete transfer; gift taxes do not apply. A property settlement transfer that is a part of a written divorce agreement or decree between divorcing spouses is not considered a gift.

d

Which of the following is not a tax goal related to income tax? A) Obtaining a stepped-up basis B) Deferring the recognition of income and gain C) Shifting receipt of income D) Freezing or reducing the value of assets subject to tax Explanation Freezing or reducing the value of assets subject to tax is a tax goal related to transfer taxes, not income taxes.

d

Which of the following objectives can be accomplished through the use of a buy-sell agreement? Fix the value of the business for estate tax purposes Provide liquidity for the decedent businessowner's estate Guarantee there will be a market or buyer for the business at the decedent's death Provide for business continuation and goodwill among existing customers A) I and II B) IV only C) II and III D) I, II, III, and IV Explanation The use of a buy-sell agreement can accomplish all of these objectives.

d

Which of the following people is(are) legally incapacitated? George is 82 years old. He is confined to a wheelchair but is mentally alert. Edna is 78 years old. She suffers from advanced dementia and is no longer able to make reasoned decisions. A) Neither I nor II B) Both I and II C) I only D) II only Explanation Only statement II is correct. Edna's dementia renders her legally incapable of acting, so she is legally incapacitated. Statement I is incorrect. George is disabled because he is no longer able to walk, but he is not a minor and there is no indication he is otherwise legally incapacitated.

d

Which of the following property transfers between family members are subject to the special zero valuation rules under Chapter 14? Corporate recapitalizations Partnership capital freezes Buy-sell agreements A) III only B) II and III C) I and II D) I, II, and III Explanation All of these property transfers between family members are subject to the special zero valuation rules under Chapter 14. In other words, these types of transfers must comply with the rules under Chapter 14 of the Tax Code or the transfer will have negative gift tax consequences.

d

Which of the following regarding state property law elections and allowances are CORRECT? Family settlement agreements may require court approval. An election against the will can be made by the deceased's surviving children. A homestead exemption prevents surviving family members of the decedent from losing certain property due to the claim of an unsecured creditor. A) II and III B) I, II, and III C) I and II D) I and III Explanation Statement II is incorrect. An election against the will protects a surviving spouse, not the surviving children, from potentially being disinherited.

d

Which of the following situations involve an indirect generation-skipping transfer? A grandfather gives his daughter a life estate in his mansion, with the remainder to his only grandchild A transfer of $1 million to an irrevocable trust in which the grantor retains the right to the income of the trust for his life with the remainder to the grantor's grandchild A transfer of $1 million to an irrevocable trust in which the income beneficiaries are the grantor's grandchildren, and the remainder beneficiaries are the grantor's great-grandchildren A transfer of $1 million to an irrevocable trust in which the income beneficiaries are the grantor's children and grandchildren, but the grantor also gives a grandchild a general power of appointment over half the trust assets A) I and II B) II and III C) I and IV D) I, II, and IV Explanation Option III is a direct skip because all the beneficiaries of the trust are skip parties (persons who are two or more generations younger than the transferor). In all other options, there is at least one skip party and one nonskip party who have an interest in the property—the definition of an indirect skip. In option II the nonskip party is the grantor, as a nonskip party is anyone with a current interest in the transferred assets who is not a skip party.

d

Which of the following statements about how qualification for special use valuation increases liquidity for the estate of a decedent is CORRECT? A) It allows the estate to take a credit against the tentative tax. B) It provides the estate with a deduction from the value of the gross estate. C) Special use allows the estate tax attributable to certain estate assets to be paid in installments. D) It can lower the value of certain estate assets. Explanation If the property meets the criteria for Section 2032(A), a significant reduction from the fair market value is available; this can reduce taxes owed, and therefore, increase liquidity. LO 6.3.1

d

Which of the following statements about intestacy is CORRECT? A) Intestacy allows a client to achieve the goals of controlling and disposing of assets according to his or her desires. B) Intestacy does not allow a client to achieve the goal of controlling assets while alive and does not allow for disposing of assets according to his or her desires at death. C) Intestacy does not allow a client to achieve the goal of controlling assets, but does allow disposing of assets according to his or her desires at death. D) Intestacy allows a client to achieve the goal of controlling assets during life, but not disposing of assets, according to his or her desires at death. Explanation At death, the state intestacy statutes govern to whom the asset will be disposed.

d

Which of the following statements about the importance of properly arranging the ownership and beneficiary designation are CORRECT? The identity of the beneficiary can affect whether the proceeds are available for the intended purpose. Life insurance could be a source of income replacement for a surviving spouse. A) I only B) II only C) Neither I nor II D) Both I and II Explanation Both statements are true.

d

Which of the following statements are correct about gratuitous transfers between spouses where one or both of the spouses is a non-U.S. citizen? If a spouse who is a resident alien makes a gratuitous transfer of property to his or her spouse who is a U.S. citizen, the rules for deciding whether the transfer is entitled to a marital deduction are the same as if both spouses were U.S. citizens. If a spouse who is a nonresident alien makes a gratuitous transfer of property located in the United States to his or her spouse who is a U.S. citizen, the rules for deciding whether the transfer is entitled to a marital deduction are the same as if both spouses were U.S. citizens. If a spouse who is a U.S. citizen makes a gratuitous transfer of property to his or her spouse who is a nonresident alien, the rules for deciding whether the transfer is entitled to a marital deduction are the same as if both spouses were U.S. citizens. If a spouse who is a U.S. citizen makes a gratuitous transfer of property to his or her spouse who is a resident alien, the rules for deciding whether the transfer is entitled to a marital deduction are the same as if both spouses were U.S. citizens. A) I and IV B) II and III C) III and IV D) I and II Explanation When discussing the marital deduction, the citizenship and residency status of the decedent/donor is irrelevant. Even when the donor/decedent is a nonresident alien, the unlimited gift and estate tax marital deduction is available for qualifying transfers to a citizen spouse. When the donee/transferee spouse is an alien, it does not matter whether the donee/transferee is a resident or not, since special rules regarding the marital deduction apply in either case. There is no gift tax marital deduction for gifts to an alien spouse, but there is a super annual exclusion if the transfer is of a present interest and is not of a terminable interest. If the transfer is made at death to an alien spouse, the marital deduction is available only if a qualified domestic trust (QDOT) is used.

d

Which of the following statements concerning the transfer of a life insurance policy is correct? The matured death benefit is always excluded from gross income. The matured death benefit may be only partially excludible from gross income and partially includible in gross income. A) Both I and II B) I only C) II only D) Neither I nor II Explanation If a life insurance policy has been transferred during the life of the insured, the death proceeds may be partially taxed under the transfer for value rules assuming there are no exceptions available.

d

Which of the following statements correctly identify estate planning activities that can be performed by a financial planner who is not also a licensed attorney? Advise a client as to who would receive property under the state intestacy statutes Estimate a client's potential federal gift tax liability Advise a client that he or she needs a new will Draft a living will for a client to execute A) I and III B) II, III, and IV C) I and IV D) II and III Explanation Statements I and IV are actions only a licensed attorney can perform. Planners can certainly estimate gift tax liability and determine if a current will meets client estate planning goals.

d

Which of the following statements regarding Medicaid planning for long-term care is CORRECT? Medicaid planning is best done with the advice of an experienced Medicaid planning advisor or a qualified elder care attorney. Each state has its own asset and income levels for qualifying for benefits. Assets transferred to others are subject to a 12-month lookback period. If a client is married, the couple's residence is a countable asset. A) II, III, and IV B) I, II, III, and IV C) IV only D) I and II Explanation Statement III is incorrect. The lookback period is 60 months. Statement IV is incorrect. If a client is married, the couple's residence is not a countable asset. Real estate other than the couple's primary estate is countable.

d

Which of the following statements regarding a QTIP election is CORRECT? If the decedent's executor makes a QTIP election, the property that is the subject of the election is excluded from the surviving spouse's gross estate when the surviving spouse dies. In determining whether to make a QTIP election, the executor should determine the best overall estate tax result for both the decedent's and the surviving spouse's gross estate. A) Neither I nor II B) I only C) Both I and II D) II only Explanation Statement I is incorrect because if the executor makes a QTIP election, the QTIP property must be included in the surviving spouse's gross estate, to the extent it has not been spent or otherwise consumed during the surviving spouse's lifetime.

d

Which of the following statements regarding a cash flow plan for an estate is CORRECT? A cash flow plan for an estate should be flexible enough to account for unexpected expenses. A cash flow plan for an estate should indicate when the estate's cash inflows and outflows are expected to occur. In developing a cash flow plan, an executor should explore possible ways of reducing the estate's cash needs. A) II and III B) I and III C) I and II D) I, II, and III Explanation All of these statements are correct.

d

Which of the following statements regarding an estate's need for liquidity is NOT correct? A) Selling illiquid assets to increase estate liquidity can be undesirable. B) Savings and checking accounts owned by the decedent at death are possible sources of estate liquidity. C) Liquid assets are assets that can be quickly converted to cash without a significant loss of principal. D) Real estate is a liquid asset. Explanation Real estate is actually an illiquid asset.

d

Which of the following statements regarding estate liquidity is CORRECT? An estate typically needs substantial amounts of cash or other liquid assets to meet its obligations following the decedent's death. An estate can manage its cash flows by developing a cash flow plan indicating when its cash inflows and outflows are expected to occur. An estate that lacks sufficient liquidity to pay its obligations when due may be forced to sell illiquid assets or borrow money at unfavorable terms. A) I only B) I and III C) II only D) I, II, and III Explanation All of these statements are correct.

d

Which of the following statements regarding gift splitting is NOT correct? A) If a donor elects to split gifts, the election must be applied to all gifts made during that calendar year. B) The gift tax annual exclusion may be doubled if gift splitting is elected. C) Gift splitting is available only to spouses. D) Gifts of community property require a gift splitting election. Explanation Gifts of community property or JTWROS property do not require a gift splitting election. All of the other statements are correct.

d

Which of the following statements regarding the development of a cash flow plan to maintain an estate's liquidity is NOT correct? A) The timing of some of the estate's cash outflows will be fairly predictable. B) The executor should anticipate that there may be a delay in receiving life insurance proceeds on the decedent's life. C) The cash flow plan should be flexible enough to account for the possibility of unexpected expenses. D) In developing a cash flow plan for an estate, it is generally not possible to reduce the estate's cash needs. Explanation It may be possible to reduce the estate's cash needs by using special elections available under the estate tax laws—for example, Section 2032A, special use valuation for farm property; Section 303, stock redemption from a closely held corporation; or Section 6166, installment payment of estate taxes. In fact, gifting assets while alive can reduce the gross estate, and thus, estate taxes. Further, the strategic use of gifting can also help an estate qualify for the special elections just noted. For example, gifting nonbusiness assets increases the percentage of the estate held as a farm or business. It also may be possible to address the estate's cash needs by using life insurance trusts and reducing debts over time while the client is alive.

d

Which of the following transactions made in 2020 require the donor to file a gift tax return? The donor makes a gift of a future interest valued at $5,000 to his son. The donor and spouse use gift splitting and give their son $12,500 for the son's birthday. The donor transfers $15,000 to a revocable inter vivos trust for the son, who is both the income and remainder beneficiary. The donor and his spouse gift community property worth $14,500 to their daughter for her birthday. A) I, II, III, and IV B) II, III, and IV C) I and III D) I and II Explanation Statements I and II require filing. Statement III is not a completed gift, and Statement IV is a split gift of community property and is less than the annual exclusion; therefore, no filing is required.

d

Which of the following transfers made last year by 70-year-old Vaughn involve a generation-skipping transfer for purposes of the federal generation-skipping transfer tax (GSTT)? A gift of $16,000 to a Uniform Transfers to Minors Act (UTMA) custodial account established by Vaughn's son, Goddard, for Goddard's 9-year-old daughter, Tiffany A transfer of $100,000 to an irrevocable trust in which he retained the right to income for 10 years with the remainder to his 27-year-old grandnephew A gift of a $28,000 remainder interest in rental real estate to his 22-year-old grandson, Max, whose parents were killed in an airplane accident two years ago A gift of $20,000 cash to his 30-year-old personal secretary A transfer of $120,000 to a revocable trust that gives Vaughn's son, Goddard, the right to the income for life and then gives that right to his granddaughter, Tiffany, for her life; Tiffany is given a testamentary power of appointment over the trust corpus A) I only B) II, III, and V C) III and IV D) I, II, and IV Explanation Options III and V do not involve generation-skipping transfers. The GSTT applies to transfers to, or for the benefit of, a skip person. A skip person is either a lineal or collateral descendant (or the spouse of such a descendant) of the grandparents of the transferor or transferor's spouse or former spouse who is two or more generations below that of the transferor, or an individual (other than the transferor's spouse or former spouse) who is not such a descendant but who is more than 37½ years younger than the transferor. Therefore, transfers I and IV are generation-skipping transfers. However, transfer III is not a GST because, for the purposes of direct skips, if the child of a transferor is deceased at the time of the transfer, the lineal descendants of that child move up one generation. Transfer II involves a GST, which is indirect because there is an intervening interest that must expire before the grandnephew can receive the remainder interest. Finally, since the GSTT follows gift tax rules for lifetime transfers, transfer V does not involve a GST because there is no completed gift—the trust is revocable.

d

Which one of the following individuals is the holder of a "legal" interest? A) The income and remainder beneficiary of a trust B) The remainder beneficiary of a trust C) The income beneficiary of a trust D) The trustee of a trust Explanation The trustee of a trust holds a legal interest and has no right to enjoy or consume the trust property unless he or she is also a beneficiary of the trust.

d

Your client currently has a large gross estate that includes the following assets: A payable on death (P.O.D.) bank account in favor of client's son at death A residence owned as joint tenants with right of survivorship with his spouse Stocks held in a revocable trust that continues after his death for the benefit of his surviving spouse and family Which one of the following is a CORRECT statement regarding the advantages or disadvantages for this client of using these will substitutes? A) An advantage of owning the residence in joint tenancy with right of survivorship is that the client can control disposition of this asset at death. B) A disadvantage of the way the bank account is titled is that the client no longer has exclusive control of the bank account. C) A disadvantage of the revocable trust is that its assets will not avoid probate because it is revocable. D) An advantage of the way the bank account is titled is that it is more easily and inexpensively created than a revocable trust. Explanation P.O.D. accounts are created simply by titling the account with a named payee at the owner's death, whereas an attorney usually drafts a revocable trust agreement. The trustor (client) maintains total control of a P.O.D. account trust until death. The disposition of property held in joint tenancy cannot be controlled at death because of the right of survivorship feature. The assets in a revocable trust cannot avoid being included in the grantor's gross estate, but they will avoid inclusion in her probate estate.

d

Your client has an estate valued at $4 million. Two months ago, his wife died. He and his now deceased wife did not have any children together, but she had two children from a prior marriage. His will, drafted in 2012, leaves everything to his wife. No contingent beneficiary is named in the will, and it does not contain a residuary clause. Included in the client's estate are real estate holdings in three other states. He wants to retain lifetime ownership of these properties because of the income they provide him. He would like the real estate holdings to pass to his wife's children in equal shares upon his death. He would like the remainder of his estate to go to his brother. Which of the following are serious estate planning pitfalls that can be avoided if your client amends his will to carry out his objectives? Having the estate pass under the laws of intestacy Having the estate assets distributed through probate Having the estate pay any estate tax Having part of the estate pass to unintended beneficiaries A) I and II B) II and III C) III and IV D) I and IV Explanation Statement II is false because probate, especially from CFP Board's perspective, should be avoided and wills go through probate. Statement III is false because amending a will won't have any effect on the estate tax calculation, and the estate isn't even close to large enough to worry about estate taxes.

d

Your client has made the following lifetime cash gifts: Total Gifts to DaughterTotal Gifts to Son2018$35,000$75,0002020$90,000$55,000 How much of your client's applicable credit is gone on December 31, 2020? A) $38,800 B) $14,400 C) $0 D) $53,200 Explanation Remember the cumulative nature of the gift tax and the annual exclusion amount for present value gifts. The total gifts to the daughter and the son in 2018 are reduced by the $15,000 annual exclusion to arrive at a net taxable gift total of $80,000 ($35,000 − $15,000 + $75,000 − $15,000). The same applies to the 2020 gifts resulting in a net gift totaling $$115,000 ($90,000 − $15,000 + $55,000 − $15,000). To calculate the tax you need to first figure the tax on the total of the gifts from both 2018 and 2020 ($80,000 + $115,000 = $195,000). Tax on $195,000 would be $38,800 + 32% of $45,000 (the amount over $150,000) which is $14,400. $14,400 + $38,800 = $53,200.

d

Your client, Cora Kortz, a 77-year-old widow, has heard about living revocable trusts. She has asked you to summarize some of their advantages and disadvantages. You would tell her all of the following except: A) the trust will provide management of her property during periods of incompetency. B) a trust provides greater confidentiality than a will regarding what assets she owns and who will receive them when she dies. C) the entire value of assets in the trust at her death will be includible in her gross estate for federal estate tax purposes. D) the trust is cumbersome and expensive because each year she will have to file two separate income tax returns—her personal return and a return for the trust. Explanation The answer is the trust is cumbersome and expensive because each year she will have to file two separate income tax returns—her personal return and a return for the trust because it is the only false statement. Income earned by assets in a living revocable trust is passed through and reported on the grantor's own personal return because of the grantor trust rules.

d

Your client, Jonathan, is the sole owner of an apple orchard. Land values are beginning to increase rapidly. He is planning to retitle the property as equal joint tenants with right of survivorship (JTWROS) with his three adult children. Which of the following are CORRECT advantages and disadvantages of changing the title to JTWROS with his children? The property will avoid probate by being automatically transferred to the surviving tenants. Jonathan's children will have no rights regarding the property while Jonathan is alive. Some of the future appreciation of the property can be shifted to his children in order to minimize transfer taxes. Future appreciation of the property cannot be shifted to his children because, when he dies, 100% of the value of the orchard will be part of his gross estate because he is the only tenant who has made a contribution for his or her interest. A) IV only B) I, II, and III C) II and III D) I and IV Explanation The other options, II and III, describe features of a tenancy in common, not features of JTWROS among nonspouse tenants. Unless the applicable state law permits, JTWROS does not provide a right of testamentary control, the ability to create unequal interests among the tenants, or the ability to include property in a tenant's federal gross estate based on the fractional interest owned, rather than on the tenant's contribution (when held among nonspouses).

d

Your client, Rafer, owns a vacation home in another state. Rafer recently married for the second time and wants to include his new wife, Edna, on the title to the vacation home. At your last client meeting, he stated that his primary concern is that this property be left to Edna outside probate at his death while restricting her disposition of the property prior to his death without his consent. Rafer revoked his old will upon his marriage to Edna, but has not yet executed a new will. You are researching property ownership to identify the most appropriate form of titling for the vacation home in preparation for your next meeting with your client and his attorney. Which one of the following statements presents the most appropriate form of titling for the vacation home? A) Tenancy in common with Edna will eliminate the need for ancillary probate. B) Joint tenancy with right of survivorship between Rafer and Edna will allow the writing of a new will to control the disposition of the property. C) Sole ownership will enable Rafer to leave the home to Edna outside probate. D) Tenancy by the entirety will prevent lifetime disposition without Rafer's consent. Explanation Tenancy by the entirety has a survivorship feature, which will pass the property to Edna outside of probate and will not allow Edna to transfer her interest while Rafer is alive without his consent. Joint tenancy would allow Edna to transfer her interest without Rafer's consent, and because the form of property ownership does not affect either owner's right to make a will. Tenancy in common would neither eliminate the need for probate nor prevent Edna from selling her interest. Sole ownership by Rafer would require probate to transfer the property to Edna, plus she would have no interest in the property until Rafer died.

d

Your data gathering meeting with Colin indicated the following about his property interests: He has the right to decide, without limitation, who will receive the entire corpus of his uncle's trust. Colin's will does not exercise this right. His wife owns a paid-up life insurance policy she purchased that insures Colin's life; his wife, Lois, is the primary beneficiary. Five years ago, he created a revocable trust for his children and funded it with $80,000 worth of securities; two years ago, when the trust fund was worth $130,000, he made it irrevocable. Two years ago, he had cumulative taxable transfers that exceeded the applicable exclusion amount and paid $32,000 in gift tax to the federal government. Which one of the following estate assets would NOT be included in Colin's taxable estate if he died in the current year? A) The date-of-death value of the irrevocable trust B) The value of the corpus of his uncle's trust C) The $32,000 in gift taxes paid D) The replacement cost of the life insurance policy Explanation Because Colin's wife is the owner and beneficiary of the life insurance policy, it would seem that the death benefit will qualify for a marital deduction. However, because the death benefit is not included in Colin's gross estate, it will not transfer from Colin to his wife, and therefore no part of the policy will be in his taxable estate. Because Colin did not specify in his will that the assets in his uncle's trust were to go to his spouse (marital deduction) or to a qualified charity (charitable deduction), the assets in this trust will also be part of his taxable estate. Because the assets in the irrevocable trust go to Colin's children, they also are not entitled to either a marital or charitable deduction and thus will be included in his taxable estate. Colin cannot transfer the gift taxes because these taxes are, in effect, a phantom asset. Therefore, they cannot qualify for any estate tax deduction that would keep them from being in his taxable estate.

d

Your data-gathering meeting with Colin indicated the following about his property interests: He has the right to decide, without limitation, who will receive the entire corpus of his uncle's trust. Colin's will does not exercise this right. His spouse owns a paid-up life insurance policy that insures Colin's life, and his spouse, Lois, is the primary beneficiary. Five years ago, he created the revocable CG Trust for his children and funded it with $80,000 worth of securities; two years ago, when the trust fund was worth $130,000, he made it irrevocable. Two years ago, he had cumulative taxable transfers that exceeded the applicable exclusion amount and paid $32,000 in gift tax to the federal government. If Colin died in the current year, which of the following would be excluded from his gross estate? A) The value of the corpus of his uncle's trust B) The date of death value of the CG Trust C) The $32,000 in gift taxes paid D) The death benefit of the life insurance policy Explanation Colin's right in his uncle's trust is a general power of appointment, which would cause the maximum value that could be appointed at death to be in the holder's (Colin's) gross estate. He has no incidents of ownership in the life insurance policy on his own life, which would cause inclusion of the death benefit in his gross estate, nor is his estate the beneficiary. There is no indication that Colin assigned ownership of this policy to his spouse. If a right to alter, amend, or revoke is given up within three years of death, as it was when the CG Trust was made irrevocable, the value of the assets over which the right was retained must be included in the grantor's gross estate. Finally, any gift taxes paid out of pocket (tax liability beyond the gift tax applicable credit a

d


Related study sets

B101 Previous Quizzes Study Guide

View Set

Quick Recall Arts & Humanities Questions

View Set

SOC 110 Ch. 6 Knowledge Development

View Set

What do you enjoy doing in your free time? How much time do you have each week for doing these things? Why do you like doing these activities? How did you start doing this activity at first? Is there some other hobby or sport you would like to try?

View Set

Week 1- Practice Exercises ( Exercise 2 )

View Set